CAT1999 SEC-MATHS

Instructions
1. The test comprises of 55 questions. You should complete the test within 40 minutes.
2. There is only one correct answer to each question.
3. All questions carry four marks each.
4. Each wrong answer will attract a penalty of one mark.

1. The number of positive integer valued pairs (x, y), satisfying 4x - 17 y = I and x < 1000 is:

  • a. 59
  • b. 57
  • c. 55
  • d. 58
  • e.Not Attempted

2. Let a, b, c be distinct digits. Consider a two digit number 'ab' and a three digit number 'ccb', both defined under the usual decimal number system. If (ab)2 = ccb and ccb > 300 then the value of b is

  • a. 1
  • b. 0
  • c. 5
  • d. 6
  • e.Not Attempted

3. The remainder when 784 is divided by 342 is

  • a. 0
  • b. 1
  • c. 49
  • d. 341
  • e.Not Attempted

4. Ten points are marked on a straight line and eleven points are marked on another straight line. How many triangles can be constructed with vertices from among the above points?

  • a.495
  • b.550
  • c.1045
  • d.2475
  • e.Not Attempted

5. For a scholarship, at most n candidates out of 2n + I can be selected. If the number of different ways of selection of at least one candidate is 63, the maximum number of candidates that can be selected for the scholarship is:

  • a. 3
  • b. 4
  • c. 2
  • d. 5
  • e.Not Attempted

6. The speed of a railway engine is 42 Km per hour when no compartment is attached, and the reduction in speed is directly proportional to the square root of the number of compartments attached. If the speed of the train carried by this engine is 24 Km per hour when 9 compartments are attached, the maximum number of compartments that can be carried by the engine is:

  • a.49
  • b. 48
  • c. 46
  • d. 47
  • e.Not Attempted

7. Total expenses of a boarding house are partly fixed and partly varying linearly with tile number of boarders. The average expense per boarder is Rs. 700 when there are 25 boarders and Rs. 600 when there are 50 boarders. What is the average expense per boarder when there are 100 boarders?

  • a. 550
  • b. 560
  • c. 540
  • d. 570
  • e.Not Attempted

8. Forty percent of the employees of a certain company are men, and 75 percent of the men more than Rs. 25,000 per year. If 45 percent of the company's employees earn more than Rs. 25,000 per year, what fraction of the women employed by the company earn Rs. 25,000 year or less'?

  • a. 2/11
  • b. 1/4
  • c. 1/3
  • d. 3/4
  • e.Not Attempted

9. If | r - 6 | = 11 and | 2q - 12 | = 8, what is the minimum possible value of q / r?

  • a. -2/5
  • b. 2/17
  • c. 10/17
  • d. None of these
  • e.Not Attempted

10.If n = 1 + x, where x is the product of four consecutive positive integers, then which of the following is/are true?
A. n is odd
B. n is prime
C. n is a perfect square

  • a. A and C only
  • b. A and B only
  • c. A only
  • d. None of these
  • e.Not Attempted

11. In a survey of political preference, 78% of those asked were in favor of at least one of the proposals: I, II and III. 50% of those asked favored proposal I, 30% favored proposal II, and 20% favored proposal III. If 5% of those asked favored all three of the proposals, what percentage of those asked favored more than one of the 3 proposals.

  • a. 10
  • b. 12
  • c. 17
  • d. 22
  • e.Not Attempted

12. For two positive integers a and b define the function h(a,b) as the greatest common factor (gdf) of a, b. Let A be a set of n positive integers. G( A), the gcf of the elements of set A is computed by repeatedly using the function h. The minimum number of times h is required to be used to compute G is:

  • a. (1/2)n
  • b. (n - 1)
  • c. n
  • d. none of these
  • e.Not Attempted

13. The figure below shows two concentric circles with centre 0. PQRS is a square, inscribed in the outer circle. It also circumscribes the inner circle, touching it at points B, C, D and A. What is the ratio of the perimeter of the outer circle to that of polygon ABCD?

  • a. π/4
  • b. 3π/2
  • c. π/2
  • d. π
  • e.Not Attempted

14. Three labeled boxes containing red and white cricket balls are all mislabeled. It is known that one of the boxes contains only white balls and one only red balls. The third contains a mixture of red and white balls. You are required to correctly label the boxes with the labels red, white and red and white by picking a sample of one ball from only one box. What is the label on the box you should sample?

  • a. White
  • b. Red
  • c. Red and White
  • d. Not possible to determine from a sample of one ball
  • e.Not Attempted

15. If n2 = 123456787654321, what is n?

  • a. 12344321
  • b. 1235789
  • c. 11111111
  • d. 1111111
  • e. Not Attempted

16. Abraham, Border, Charlie, Dennis and Elmer and their respective wives recently dined together and were seated at a circular table. The seats were so arranged that men and women alternated and each woman was three places distant from her husband. Mrs. Charlie sat to the left of Mr. Abraham. Mrs. Elmer sat two places to the right of Mrs. Border. Who sat to the right of Mr. Abraham?

  • a. Mrs. Dennis
  • b. Mrs. Elmer
  • c. Mrs. Border
  • d. Mrs. Border or Mrs. Dennis
  • e.Not Attempted

17. Navjivan Express from Ahmedabad to Chennai leaves Ahmedabad at 6:30 am and travels at 50km per hour towards Baroda situated 100 kms away. At 7:00 am Howrah - Ahmedabad express leaves Baroda towards Ahmedabad and travels at 40 km per hour. At 7:30 Mr. Shah, the traffic controller at Baroda realises that both the trains are running on the same track. How much time does he have to avert a head-on collision between the two trains?

  • a. 15 minutes
  • b. 20 minutes
  • c. 25 minutes
  • d. 30 minutes
  • e.Not Attempted

18. There is a circle of radius 1 cm. Each member of a sequence of regular polygons S1(n), n = 4,5,6,... , where n is the number of sides of the polygon, is circumscribing the circle; and each member of the sequence of regular polygons S2(n), n = 4,5,6.... where n is the number of sides of the polygon, is inscribed in the circle. Let L1(n) and L2(n) denote the perimeters of the corresponding polygons of S1(n) and S2(n). Then {L1(13) + 2π} / L2(17) is

  • a. greater than π/ 4 and less than 1
  • b. greater than 1 and less than 2
  • c. greater than 2
  • d. less than π/ 4
  • e.Not Attempted

19.There is a square field with each side 500 metres long. It has a compound wall along its perimeter. At one of its comers, a triangular area of the field is to be cordoned off by erecting a straight line fence. The compound wall and the fence will form its borders. If the length of the fence is 100 metres, what is the maximum area in square metres that can be cordoned off?

  • a. 2,500
  • b. 10,000
  • c. 5,000
  • d. 20,000
  • e.Not Attempted

DIRECTIONS for questions 75 to 77:
These questions are based on the situation given below:
Ten coins are distributed among four people P, Q, R, S such that one of them gets one coin, another gets two coins, the third gets three coins and the fourth gets four coins. It is known that Q gets more coins than P, and S gets fewer coins than R.

20. If the number of coins distributed to Q is twice the number distributed to P then which one of the following is necessarily true?

  • a. R gets an even number of coins.
  • b. R gets an odd number of coins.
  • c. S gets an even number of coins
  • d. S gets an odd number of coins.
  • e.Not Attempted

21. If R gets at least two more coins than S, then which one of the following is necessarily true?

  • a. Q gets at least two more coins than S.
  • b. Q gets more coins than P.
  • c. P gets more coins than S.
  • d. P and Q together get at least five coins.
  • e.Not Attempted

22. If Q gets fewer coins than R, then which one of the following is not necessarily true?

  • a. P and Q together get at least four coins.
  • b. Q and S together get at least four coins.
  • c. R and S together get at least five coins.
  • d. P and R together get at least five coins.
  • e.Not Attempted

DIRECTIONS for questions 78 to 80:
These questions are based on the situation given below:
A young girl Roopa leaves home with x flowers, goes to the bank of a nearby river. On the bank of the river, there are four places of worship, standing in a row. She dips all the x flowers into the river. The number of flowers doubles. Then she enters the first place of worship, offers y flowers to the deity. She dips the remaining flowers into the river, and again the number of flowers doubles. She goes to the second place of worship, offers y flowers to the deity. She dips the remaining flowers into the river, and again the number of flowers doubles. She goes to the third place of worship, offers y flowers to the deity. She dips the remaining flowers into the river, and again the number of flowers doubles. She goes to the fourth place of worship, offers y flowers to the deity. Now she is left with no flowers in hand.

23. If Roopa leaves home with 30 flowers, the number of flowers she offers to each deity is:

  • a. 30
  • b. 31
  • c. 32
  • d. 33
  • e.Not Attempted

24. The minimum number of flowers that could be offered to each deity is:

  • a. 0
  • b. 15
  • c. 16
  • d. Cannot be determined
  • e.Not Attempted

25. The minimum number of flowers with which Roopa leaves home is:

  • a. 16
  • b. 15
  • c. 0
  • d. Cannot be determined
  • e.Not Attempted

DIRECTIONS for questions 26 and 27:
The following table presents the sweetness of different items relative to sucrose, whose sweetness is taken to be 1.00.

Lactose 0.163
Maltose 0.32
Glucose 0.74
Sucrose 1.00
Fructose 1.70
Saccharin 675.00

26. What is the minimum amount of sucrose (to the nearest gram) that must be added to one-gram of saccharin to make a mixture that will be at least I 00 times as sweet as glucose?

  • a. 7
  • b. 8
  • c. 9
  • d. 10
  • e.Not Attempted

27. Approximately how many times sweeter than sucrose is a mixture consisting of glucose, sucrose and fructose in the ratio of 1: 2: 3?

  • a. 1.3
  • b. 1
  • c. 0.6
  • d. 2.3
  • e.Not Attempted

DIRECTIONS for questions 28 and 29:
These questions are based on the situation given below:

A, B, C, D, E and F are a group of friends from a club. There are two housewives, one lecturer, one architect, one accountant and one lawyer in the group. There are two married couples in the group. The lawyer is married to D who is a housewife. No lady in the group is either an architect or an accountant. C, the accountant, is married to F who is a lecturer. A is married to D and E is not a housewife.

28. What is E?

  • a. Lawyer
  • b. Architect
  • c. Lecturer
  • d. Accountant
  • e.Not Attempted

29. How many members of the group are male?

  • a. 2
  • b. 3
  • c. 4
  • d. None of these
  • e.Not Attempted

DIRECTIONS for questions 30 and 31:
These questions are based on the situation given below:

Seven university cricket players are to be honored at a special luncheon. The players will be seated on the dais along one side of a single rectangular table.
A and G have to leave the luncheon early and must be seated at the extreme right end of the table, which is closest to the exit.
B will receive the Man of the Match award and must be in the center chair.
C and D who are bitter rivals for the position of wicket keeper, dislike one another and should be seated as far apart as possible.
E and F are best friends and want to sit together.

30. Which of the following may not be seated at either end of the table?

  • a. C
  • b. D
  • c. G
  • d. F
  • e.Not Attempted

31. Which of the following pairs may not be seated together?

  • a. E & A
  • b. B & D
  • c. C & F
  • d. G & D
  • e.Not Attempted

DIRECTIONS for questions 32 and 33:
These questions are based on the situation given below:

A rectangle PRSU, is divided into two smaller rectangles PQTU, and QRST by the line TQ. PQ=10cm, QR = 5 cm and RS = 10 cm. Points A, B, F are within rectangle PQTU, and points C, D, E are within the rectangle QRST. The closest pair of points among the pairs (A, C), (A, D), (A, E), (F, C), (F, D), (F, E), (B, C), (B, D), (B, E) are 10√3 cm apart.

32. Which of the following statements is necessarily true?

  • a. The closest pair of points among the six given points cannot be (F, C)
  • b. Distance between A and B is greater than that between F and C
  • c. The closest pair of points among the six given points is (C, D), (D, E), or (C, E)
  • d. None of the above
  • e.Not Attempted

33. AB > AF > BF; CD > DE > CE; and BF = 6√5 cm. Which is the closest pair of points among all the six given points?

  • a.B, F
  • b.C, D
  • c.A, B
  • d. None of these
  • e.Not Attempted

DIRECTIONS for questions 34 to 37:
These questions are based on the situation given below
In each of the questions 89 to 92 a pair of graphs F(x) and F1(x) is given. These are composed of straight-line segments, shown as solid lines, in the domain x Î (-2, 2).
If F1(x) = - F(x) choose the answer as a;
if F1(x) = F(- x) choose the answer as b;
if F1(x) = - F(- x) choose the answer as c;
and if none of the above is true, choose the answer as d;

34.

  • a. A
  • b. B
  • c. C
  • d. D
  • e.Not Attempted

35.

  • a. A
  • b. B
  • c. C
  • d. D
  • e. Not Attempted

36.

  • a. A
  • b. B
  • c. C
  • d. D
  • e.Not Attempted

37.

  • a. A
  • b. B
  • c. C
  • d. D
  • e.Not Attempted

DIRECTIONS for questions 93 and 94:
These questions are based on the situation given below:

There are in blue vessels with known volumes V1, V2 , ...., Vm, arranged in ascending order of volume, where v1 > 0.5 litre, and vm < 1 litre. Each of these is full of water initially. The water from each of these is emptied into a minimum number of empty white vessels, each having volume 1 litre. The water from a blue vessel is not emptied into a white vessel unless the white vessel has enough empty volume to hold all the water of the blue vessel. The number of white vessels required to empty all the blue vessels according to the above rules was n.

38. Among the four values given below, which is the least upper bound on e, where e is the total empty volume in the n white vessels at the end of the above process?

  • a. mvm
  • b. m(1 - vm)
  • c. mv1
  • d. m(1 - v1)
  • e.Not Attempted

39.Let the number of white vessels needed be n1 for the emptying process described above, if the volume of each white vessel is 2 liters. Among the following values, which is the least upper bound on n1?

  • a. m/4
  • b. smallest integer greater than or equal to (n/2)
  • c. n
  • d. greatest integer less than or equal to (n/2)
  • e.Not Attempted

DIRECTIONS for questions 40 to 42:
These questions are based on the situation given below:

There are fifty integers a1, a2,...,a50, not all of them necessarily different. Let the greatest integer of these fifty integers be referred to as G, and the smallest integer be referred to as L. The integers a1 through a24 form sequence S1, and the rest form sequence S2. Each member of S1 is less than or equal to each member of S2.

40. All values in S1 are changed in sign, while those in S2 remain unchanged. Which of the following statements is true?

  • a.Every member of S1 is greater than or equal to every member of S2.
  • b. G is in S1
  • c. If all numbers originally in S1 and S2 had the same sign, then after the change of sign, the largest number of S1 and S2 is in S1.
  • d. None of the above.
  • e.Not Attempted

41. Elements of S1 are in ascending order, and those of S2 are in descending order. a24 and a25 are interchanged. Then, which of the following statements is true ?

  • a. S1 continues to be in ascending order
  • b. S2 continues to be in descending order
  • c. S1 continues to be in ascending order and S2 in descending order
  • d. None of the above
  • e.Not Attempted

42. Every element of S1 is made greater than or equal to every element of S2 by adding to each element of S1 an integer x. Then x cannot be less than:

  • a. 210
  • b. The smallest value of S2
  • c. The largest value of S2
  • d. ( G-L )
  • e.Not Attempted

DIRECTIONS for questions 43 to 45:
These questions are based on the situation given below:

Let x and y be real numbers and let f(x, y) = | x + y |, F(f(x, y)) = -f(x, y) and G(f(x, y)) = -F(f(x, y))

43. Which of the following statements is true?

  • a. F(f(x, y)) . G(f(x, y)) = -F(f(x, y)) . G(f(x, y))
  • b. F(f(x, y)) . G(f(x, y)) > -F(f(x, y)) . G(f(x, y))
  • c. F(f(x, y)) . G(f(x, y)) G(f(x, y)) . F(f(.x, y))
  • d. F(f(x,y)) + G(f(x, y)) + f(x, y) = f(-x, -y)
  • e.Not Attempted

44. What is the value of f(G(f(1, 0)), f(F(f(1, 2)), G(f(1, 2))))?

  • a. 3
  • b. 2
  • c. 1
  • d. 0
  • e.Not Attempted

45. Which of the following expressions yields x2 as its result?

  • a. F(f(x, -x)).G(f(x, -x))
  • b. F(f(x, x)).G(f(x, x)).4
  • c. -F(f(x, x).G(f(x, x)) ÷ log2 16
  • d. f(x, x).f(x, x)
  • e.Not Attempted

DIRECTIONS for questions 46 and 47:
These questions are based on the situation given below:

A robot moves on a graph sheet with x and y-axes. The robot is moved by feeding it with a sequence of instructions. The different instructions that can be used in moving it, and their meanings are:
Instruction Meaning GOTO(x,y) move to point with coordinates (x, y) no matter where you are currently
WALKX(P) Move parallel to the x-axis through a distance of p, in the positive direction if p is positive, and in the negative direction if p is negative
WALKY(P) Move parallel to the y-axis through a distance of p, in the positive direction if p is positive, and in the negative direction if p is negative.

46. The robot reaches point (6, 6) when a sequence of three instructions is executed, the first of which is a GOTO(x, y) instruction, the second is WALKX(2) and the third is WALKY(4). What are the values of x and y?

  • a. 2, 4
  • b. 0, 0
  • c. 4, 2
  • d. 2, 2
  • e.Not Attempted

47. The robot is initially at (x, y), x > 0 and y < 0. The minimum number of instructions needed to be executed to bring it to the origin (0,0) if you are prohibited from using the GOTO instruction is: 1. 2

  • a. 2
  • b. 1
  • c. x + y
  • d. 0
  • e.Not Attempted

DIRECTIONS for questions 48 to 50:
These questions are based on the situation given below: A road network (shown in the figure below) connects cities A, B, C and D. All road segments are straight lines. D is the midpoint on the road connecting A and C. Roads AB and BC are at right angles to each other with BC shorter than AB. The segment AB is 100 km long. Ms. X and Mr. Y leave A at 8:00 am, take different routes to city C and reach at the same time. X takes the highway from A to B to C and travels at an average speed of 61.875 km per hour. Y takes the direct route AC and travels at 45 km per hour on segment AD. Y's speed on segment DC is 55 km per hour.

48. What is the average speed of Y in km per hour?

  • a. 47.5
  • b. 49.5
  • c. 50
  • d. 52
  • e.Not Attempted

49. The total distance traveled by Y during the journey is approximately

  • a. 105 km
  • b. 150 km
  • c. 130 km
  • d. Cannot be determined
  • e.Not Attempted

50. What is the length of the road segment BD?

  • a. 50 km
  • b. 52.5 km
  • c. 55 km
  • d. Cannot be determined
  • e.Not Attempted

DIRECTIONS for questions 51 and 52:
These questions are based on the situation given below:

Rajiv reaches city B from city A in 4 hours, driving at the speed of 35 km per hour for the first 2 hours and at 45 km per hour for the next two hours. Aditi follows the same route, but drives at three different speeds: 30, 40 and 50 km per hour, covering an equal distance in each speed segment. The two cars are similar with petrol consumption characteristics (km per litre) shown in the figure below.

51. The amount of petrol consumed by Aditi for the journey is

  • a. 8.3 litres
  • b. 8.6 litres
  • c. 8.9 litres
  • d. 9.2 litres
  • e.Not Attempted

52. Zoheb would like to drive Aditi's car over the same route from A to B and minimize the petrol consumption for the trip. The amount of petrol required by him is

  • a. 6.67 litres
  • b. 7 litres
  • c. 6.33 litres
  • d. 6.0 litres
  • e.Not Attempted

DIRECTIONS for questions 108 to 110:
These questions are based on the situation given below:

Recently, Ghosh Babu spent his winter vacation on Kyakya Island. During the vacation, he visited the local casino where he came across a new card game. Two players, using a normal deck of 52 playing cards, play this game. One player is called the Dealer and the other is called the Player. First, the Player picks a card at random from the deck. This is called the base card. The amount in rupees equal to the face value of the base card is called the base amount. The face values of Ace, King, Queen and Jack are ten. For other cards, the face value is the number on the card. Once, the Player picks a card from the deck, the Dealer pays him the base amount. Then the dealer picks a card from the deck and this card is called the top card. If the top card is of the same suit as the base card, the Player pays twice the base amount to the Dealer. If the top card is of the same colour as the base card (but not the same suit) then the Player pays the base amount to the Dealer. If the top card happens to be of a different colour than the base card, the Dealer pays the base amount to the Player. Ghosh Babu played the game 4 times. First time he picked eight of clubs and the Dealer picked queen of clubs. Second time, he picked ten of hearts and the dealer picked two of spades. Next time, Ghosh Babu picked six of diamonds and the dealer picked ace of hearts. Lastly, he picked eight of spades and the dealer picked jack of spades. Answer the following questions based on these four games.

53. If Ghosh Babu stopped playing the game when his gain would be maximized, the gain in Rs. would have been

  • a. 12
  • b. 20
  • c. 16
  • d. 4
  • e.Not Attempted

54. The initial money Ghosh Babu had (before the beginning of the game sessions) was Rs. X. At no point did he have to borrow any money. What is the minimum possible value of X?

  • a. 16
  • b. 8
  • c. 100
  • d. 24
  • e.Not Attempted

55. If the final amount of money that Ghosh Babu had with him was Rs. 100, what was the initial amount he had with him?

  • a. 120
  • b. 8
  • c. 4
  • d. 96
  • e.Not Attempted

CAT99 SEC-DI

Instructions
1. The test comprises of 55 questions. You should complete the test within 40 minutes.
2. There is only one correct answer to each question.
3. All questions carry four marks each.
4. Each wrong answer will attract a penalty of one mark.

DIRECTIONS for questions 1 to 10:
Each question consists of five statements followed by options consisting of three statements put together in a specific order. Choose the option which indicates a valid argument, that is, where the third statement is a conclusion drawn from the preceding two statements.

Example: All cigarettes are hazardous to health.
Brand X is a cigarette.
Brand X is hazardous to health.
ABC is a valid option, where statement C can be concluded from statements A and B

1. A. All software companies employ knowledge workers. B. Tara Tech employs knowledge workers.
C. Tara Tech is a software company.
D. Some software companies employ knowledge workers.
E. Tara Tech employs only knowledge workers.

  • a. ABC
  • b. ACB
  • c. CDB
  • d. ACE
  • e.Not Attempted

2. A. Traffic congestion increases carbon monoxide in the environment.
B. Increase in carbon monoxide is hazardous to health.
C. Traffic congestion is hazardous to health.
D. Some traffic congestion does not cause increased carbon monoxide.
E. Some traffic congestion is not hazardous to health.

  • a. CBA
  • b. BDE
  • c. CDE
  • d. BAC
  • e.Not Attempted

3. A. Apples are not sweets.
B. Some apples are sweet.
C. All sweets are tasty.
D. Some apples are not tasty.
E. No apple is tasty.

  • a. CEA
  • b. BDC
  • c. CBD
  • d. EAC
  • e.Not Attempted

4. A. Some towns in India are polluted.
B. All polluted towns should be destroyed.
C. Town Meghana should be destroyed.
D. Town Meghana is polluted.
E. Some towns in India should be destroyed.

  • a.BDE
  • b.BAE
  • c. ADE
  • d.CDB
  • e.Not Attempted

5. A. No patriot is a criminal.
B. Bundledas is not a criminal.
C. Bundledas is a patriot.
D. Bogusdas is not a patriot.
E. Bogusdas is a criminal.

  • a. ACB
  • b. ABC
  • c. ADE
  • d. ABE
  • e.Not Attempted

6. A. Ant eaters like ants.
B. Boys are ant eaters.
C. Balaram is an ant eater.
D. Balaram likes ants.
E. Balaram may eat ants.

  • a.DCA
  • b.ADC
  • c.ABE
  • d.ACD
  • e.Not Attempted

7. A. All actors are handsome.
B. Some actors are popular.
C. Ram is handsome.
D. Ram is a popular actor.
E. Some popular people are handsome.

  • a. ACD
  • b. ABE
  • c. DCA
  • d. EDC
  • e.Not Attempted

8. A. Modern industry is technology driven.
B. BTI is a modern industry.
C. BTI is technology driven.
D. BTI may be technology driven.
E. Technology driven industry is modem.

  • a. ABC
  • b. ABD
  • c. BCA
  • d. EBC
  • e.Not Attempted

9. A. All Golmal islanders are blue coloured people.
B. Some smart people are not blue coloured people.
C. Some babies are blue coloured.
D. Some babies are smart.
E. Some smart people are not Golmal islanders.

  • a. BCD
  • b. ABE
  • c. CBD
  • d. None of the above
  • e.Not Attempted

10.A. MBAs are in great demand.
B. Ram and Sita are in great demand.
C. Ram is in great demand.
D. Sita is in great demand.
E. Ram and Sita are MBAs.

  • a. ABE
  • b. ECD
  • c. AEB
  • d. EBA
  • e.Not Attempted

DIRECTIONS for questions 121 to 124:

Each question has a main statement followed by four statements labelled A, B, C and D. Choose the ordered pair of statements where the first statement implies the second, and the two statements are logically consistent with the main statement.

11. Either the orangutan is not angry, or he frowns upon the world.
A. The orangutan frowns upon the world.
B. The orangutan is not angry.
C. The orangutan does not frown upon the world.
D. The orangutan is angry.

  • a. CB only
  • b. DA only
  • c. AB only
  • d. CB and DA
  • e.Not Attempted

12. Either Ravana is a demon, or he is a hero.
A. Ravana is a hero.
B. Ravana is a demon.
C. Ravana is not a demon.
D. Ravana is not a hero.

  • a. CD only
  • b. BA only
  • c. CD and BA
  • d. DB and CA
  • e.Not Attempted

13. Whenever Rajeev uses the internet, he dreams about spiders.
A. Rajeev did not dream about spiders.
B. Rajeev used the internet.
C. Rajeev dreamt about spiders.
D. Rajeev did not use the internet.

  • a. AD
  • b. DC
  • c. CB
  • d. DA
  • e.Not Attempted

14. If I talk to my professors, then I do not need to take a pill for headache.
A. I talked to my professors.
B. I did not need to take a pill for headache.
C. I needed to take a pill for headache.
D. I did not talk to my professors.

  • a. AB only
  • b. DC only
  • c. CD only
  • d. AB and CD
  • e.Not Attempted

DIRECTIONS for questions 15 to 24:
Each question has a set of four statements. Each statement has three segments. Choose the alternative where the third segment in the statement can be logically deduced using both the preceding two, but not just from one of them.

15. A. No cowboys laugh. Some who laugh are sphinxes. Some sphinxes are not cowboys.
B. All ghosts are fluorescent. Some ghosts do not sing. Some singers are not fluorescent.
C. Cricketers indulge in swearing. Those who swear are hanged. Some who are hanged are not cricketers.
D. Some crazy people are pianists. All crazy people are whistlers. Some whistlers are pianists.

  • a. A and B
  • b. C only
  • c. A and D
  • d. D only
  • e. Not Attempted

16. A. All good people are knights. All warriors are good people. All knights are warriors.
B. No footballers are ministers. All footballers are tough. Some ministers are players.
C. All pizzas are snacks. Some meals are pizzas. Some meals are snacks.
D. Some barkers are musk-deer. All barkers are sloth bears. Some sloth bears are musk-deer.

  • a. C and D
  • b. B and C
  • c. A only
  • d. C only
  • e.Not Attempted

17. A. Dinosaurs are pre-historic creatures. Water-buffaloes are not dinosaurs. Water-buffaloes are not pre- historic creatures.
B. All politicians are frank. No frank people are crocodiles. No crocodiles are politicians.
C. No diamond is quartz. No opal is quartz. Diamonds are opals.
D. All monkeys like bananas. Some GI Joes like bananas. Some GI Joes are monkeys.

  • a. C only
  • b. B onlys
  • c. A and D
  • d. B and C
  • e.Not Attempted

18. A. All earthquakes cause havoc. Some landslides cause havoc. Some earthquakes cause landslides.
B. All glass things are transparent. Some curios are glass things. Some curios are transparent.
C. All clay objects are brittle. All XY are clay objects. Some XY are brittle.
D. No criminal is a patriot. Ram is not a patriot. Ram is a criminal.

  • a. D only
  • b. B only
  • c. C and B
  • d. A only
  • e.Not Attempted

19.A. MD is an actor. Some actors are pretty. MD is pretty.
B. Some men are cops. All cops are brave. Some brave people are cops.
C. All cops are brave. Some men are cops. Some men are brave.
D. All actors are pretty; MD is not an actor; MD is not pretty.

  • a. D only
  • b. C only
  • c. A only
  • d. B and C
  • e.Not Attempted

20. A. All IIMs are in India. No BIMs are in India. No IIMs are BIMs.
B. All IIMs are in India. No BIMs are in India. No BIMs are IIMs.
C. Some IIMs are not in India. Some BIMs are not in India. Some IIMs are BIMs.
D. Some IIMs are not in India. Some BIMs are not in India. Some BIMs are IIMs.

  • a. A and B
  • b. C and D
  • c. A only
  • d. B only
  • e.Not Attempted

21. A. Citizens of Yes Islands speak only the truth. Citizens of Yes Islands are young people. Young people speak only the truth.
B. Citizens of Yes Islands speak only the truth. Some Yes Islands are in the Atlantic. Some citizens of Yes Islands are in the Atlantic.
C. Citizens of Yes Islands speak only the truth. Some young people are citizens of Yes Islands. Some young people speak only the truth.
D. Some people speak only the truth. Some citizens of Yes Islands speak only the truth. Some people who speak only the truth are citizens of Yes Islands.

  • a.A only
  • b.B only
  • c. C only
  • d. D only
  • e.Not Attempted

22. A. All mammals are viviparous. Some fish are viviparous. Some fish are mammals.
B. All birds are oviparous. Some fish are not oviparous. Some fish are birds.
C. No mammal is oviparous. Some creatures are oviparous and some are not. Some creatures are not mammals.
D. Some creatures are mammals. Some creatures are viviparous. Some mammals are viviparous.

  • a. A only
  • b. B only
  • c. C only
  • d. D only
  • e.Not Attempted

23. A. Many singers are not writers. All poets are singers. Some poets are not writers.
B. Giants climb beanstalks. Some chicken do not climb beanstalks. Some chicken are not giants.
C. All explorers live in snowdrifts. Some penguins live in snowdrifts. Some penguins are explorers.
D. Amar is taller than Akbar. Anthony is shorter than Amar. Akbar is shorter than Anthony.

  • a. A only
  • b. B only
  • c. B and C
  • d. D only
  • e.Not Attempted

24. A. A few farmers are rocket scientists. Some rocket scientists catch snakes. A few farmers catch snakes.
B. Poonam is a kangaroo. Some kangaroos are made of teak. Poonam is made of teak.
C. No bulls eat grass. All matadors eat grass. No matadors are bulls.
D. Some skunks drive Cadillacs. All skunks are polar bears. Some polar bears drive Cadillacs.

  • a. B only
  • b. A and C
  • c. C only
  • d. C and D
  • e.Not Attempted

DIRECTIONS for questions 25 to 28:<>br> These questions are based on the situation given below:

The figure below presents sales and net profit, in Rs. Crores, of IVP Ltd for the five years from 199495 to 1998-99. During this period, the sales increased from Rs.100 Crores to Rs. 680 Crores. Correspondingly, the net profit increased from Rs. 2.2 Crores to Rs. 12 Crores. Net profit is defined as the excess of sales over total costs.

25. The highest percentage of growth in sales, relative to the previous year, occurred in

  • a. 1995-96
  • b. 1996-97
  • c. 1997-98
  • d. 1998-99
  • e.Not Attempted

26. The highest percentage growth in net profit, relative to the previous year, was achieved in

  • a. 1998-99
  • b. 1997-98
  • c. 1996-97
  • d. 1995-96
  • e.Not Attempted

27. Defining profitability as the ratio of net profit to sales, IVP Ltd. recorded the highest profitability in

  • a. 1998-99
  • b. 1997-98
  • c. 1994-95
  • d. 1996-97
  • e.Not Attempted

28. With profitability as defined in question 27, it can be concluded that

  • a. Profitability is non-decreasing during the five years from 1994-95 to 1998-99
  • b. Profitability is non-increasing during the five years from 1994-95 to 1998-99.
  • c. Profitability remained constant during the five years from 1994 - 95 to 1998 -99.
  • d. None of the above
  • e.Not Attempted

DIRECTIONS for questions 29 to 34:

Consider the information provided in the figure below relating to India's foreign trade in 1997-98 and the first eight months of 1998-99. Total trade with a region is defined as the sum of exports to and imports from that region. Trade deficit is defined as the excess of imports over exports. Trade deficit may be negative A: U.S.A.
B: Germany
C: Other E.U.
D: U.K.
E: Japan
F: Russia
G: Other East Europe
H: OPEC
I: Asia
J: Other L.D.Cs
K: Others

29. What is the region with which India had the highest total trade in 1997-98?

  • a.USA
  • b. Other E. U.
  • c. OPEC
  • d. Others
  • e.Not Attempted

30. In 1997-98 the amount of Indian exports, in millions US $, to the region with which India had the lowest total trade, is approximately

  • a. 750
  • b. 340
  • c. 220
  • d. 440
  • e.Not Attempted

31. In 1997-98, the trade deficit with respect to India, in billions of US $, for the region with the highest trade deficit with respect to India, is approximately equal to

  • a. 6.0
  • b. 3.0
  • c. 4.5
  • d. 7.5
  • e.Not Attempted

32. What is the region with the lowest trade deficit with India in 1997-98?

  • a. USA
  • b. Asia
  • c. Others
  • d. Other E. U.
  • e.Not Attempted

ADDITIONAL DIRECTIONS for questions 143 and 144:
These questions are based on the situation give below:

Assume that the average monthly exports from India and imports to India during the remaining four months of 1998-99 would be the same as that for the first eight months of the year.

33. What is the region to which Indian exports registered the highest percentage growth between 1997-98 and 1998-99?

  • a. Other East Europe
  • b. USA
  • c. Asia
  • d. Exports have declined, no growth
  • e.Not Attempted

34. What is the percentage growth rate in India's total trade deficit between 1997-98 and 1998-99?

  • a. 43
  • b. 47
  • c. 50
  • d. 40
  • e.Not Attempted

DIRECTIONS for questions 35 to 38:
These questions are based on the price fluctuations of four commodities - arhar, pepper, sugar and gold during February - July 1999 as described in the figures below:

35. Price change of a commodity is defined as the absolute difference in ending and beginning prices expressed as a percentage of the beginning. What is the commodity with the highest price change?

  • a. Arhar
  • b. Pepper
  • c. Sugar
  • d. Gold
  • e. Not Attempted

36. Price volatility (PV) of a commodity is defined as follows:
PV = (highest price during the period – lowest price during the period) / average price during the period.
What is the commodity with the lowest price volatility?

  • a. Arhar
  • b. Pepper
  • c. Sugar
  • d. Gold
  • e.Not Attempted

37. Mr. X, a funds manager with an investment company invested 25% of his funds in each of the four commodities at the beginning of the period. He sold the commodities at the end of the period. His investments in the commodities resulted in:

  • a. 17% profit
  • b. 5.5% loss
  • c. no profit, no loss
  • d. 4.3% profit
  • e.Not Attempted

38. The price volatility of the commodity with the highest PV during the Febrauary-July period is approximately equal to:

  • a. 3%
  • b. 40%
  • c. 20%
  • d. 12%
  • e.Not Attempted

DIRECTIONS for questions 39 to 43:
These questions are based on the table below presenting data on percentage population covered by drinking water and sanitation facilities in selected Asian countries.
Country A is said to dominate B or A > B if A has higher percentage in total coverage for both drinking water and sanitation facilities, and, B is said to be dominated by A, or B < A.
A country is said to be on the coverage frontier if no other country dominates it. Similarly, a country is not on the coverage frontier if it is dominated by at least one other country.

39.What are the countries on the coverage frontier?

  • a. India and China
  • b. Sri Lanka and Indonesia
  • c. Philippines and Bangladesh
  • d. Nepal and Pakistan
  • e.Not Attempted

40. Which of the following statements are true? A. India > Pakistan and India > Indonesia
B. India > China and India > Nepal
C. Sri Lanka > China
D. China > Nepal

  • a. A and C
  • b. B and D
  • c. A, B and C
  • d. B, C and D
  • e.Not Attempted

41. Using only the data presented under Sanitation facilities columns, it can be concluded that rural population in India, as a percentage of its total population is approximately

  • a. 76
  • b. 70
  • c. 73
  • d. Cannot be determined
  • e.Not Attempted

42. Again, using only the data presented under Sanitation facilities columns, sequence China, Indonesia and Philippines in ascending order of rural population as a percentage of their respective total populations. The correct order is:

  • a. Philippines, Indonesia, China
  • b. Indonesia, China, Philippines
  • c. Indonesia, Philippines, China
  • d. China, Indonesia, Philippines
  • e.Not Attempted

43. India is not on the coverage frontier because A. it is lower than Bangladesh in terms of coverage of drinking water facilities.
B. it is lower than Sri Lanka in terms of coverage of sanitation facilities.
C. it is lower than Pakistan in terms of coverage of sanitation facilities.
D. it is dominated by Indonesia.

  • a. A and B
  • b. A and C
  • c. D
  • d. None of these
  • e.Not Attempted

DIRECTIONS for questions 154 and 155:
These relate to the above table with the additional proviso that the gap between the population coverages of sanitation facilities and drinking water facilities is a measure of disparity in coverage.

44. The country with the most disparity in coverage of rural sector is

  • a. India
  • b. Bangladesh
  • c. Nepal
  • d. None of these
  • e.Not Attempted

45. The country with the least disparity in coverage of urban sector is

  • a. India
  • b. Pakistan
  • c. Philippines
  • d. None of these
  • e.Not Attempted

DIRECTIONS for questions 46 to 55:
Each question is followed by two statements, A and B. Answer each question using the following instructions:
Choose A if the question can be answered by using one of the statements alone, but cannot be answered using the other statement alone.
Choose B if the question can be answered by using either statement alone.
Choose C if the question can be answered by using both statements together, but cannot be answered using either statement alone.
Choose D if the question cannot be answered even by using both statements together.

46. The average weight of students in a class is 50 kg. What is the number of students in the class?
A. The heaviest and the lightest members of the class weigh 60 kg and 40 kg respectively.
B. Exclusion of the heaviest and the lightest members from the class does not change the average weight of the students.

  • a. A
  • b. B
  • c. C
  • d. D
  • e.Not Attempted

47. A small storage tank is spherical in shape. What is the storage volume of the tank?
A. The wall thickness of the tank is I cm.
B. When the empty spherical tank is immersed in a large tank filled with water, 20 litres of water overflow from the large tank.

  • a. A
  • b. B
  • c. C
  • d. D
  • e.Not Attempted

48. Mr. X starts walking northwards along the boundary of a field, from point A on the boundary, and after walking for 150 metres reaches B, and then walks westwards, again along the boundary, for another 100 metres when he reaches C. What is the maximum distance between any pair of points on the boundary of the field?
A. The field is rectangular in shape.
B. The field is a polygon, with C as one of its vertices and A the mid point of a side.

  • a. A
  • b. B
  • c. C
  • d. D
  • e.Not Attempted

49. A line graph on a graph sheet shows the revenue for each year from 1990 through 1998 by points and joins the successive points by straight line segments. The point for revenue of 1990 is labelled A, that for 1991 as B, and that for 1992 as C. What is the ratio of growth in revenue between 91-92 and 90-91?
A. The angle between AB and X-axis when measured with a protractor is 40 degrees, and the angle between CB and X-axis is 80 degrees.
B. The scale of Y-axis is 1 cm = 1000 Rs.

  • a. A
  • b. B
  • c. C
  • d. D
  • e.Not Attempted

50. There is a circle with centre C at the origin and radius r cm. Two tangents are drawn from an external point D at a distance d cm from the centre. What are the angles between each tangent and the X-axis?
A. The coordinates of D are given
B. The X-axis bisects one of the tangents.

  • a. A
  • b. B
  • c. C
  • d. D
  • e.Not Attempted

51. Find a pair of real numbers x and y that satisfy the following two equations simultaneously. It is known that the values of a, b, c, d, e and f are non-zero.
ax + by = c
dx + ey = f

A. a = kd and b = ke, c = kf, k ≠ 0
B. a = b = 1, d = e = 2, f ≠ 2c

  • a. A
  • b. B
  • c. C
  • d. D
  • e.Not Attempted

52. Three professors A, B and C are separately given three sets of numbers to add. They were expected to find the answers to 1+1, 1+1+2, and 1+1 respectively. Their respective answers were 3, 3, and 2. How many of the professors are mathematicians?
A. A mathematician can never add two numbers correctly, but can always add three numbers correctly.
B. When a mathematician makes a mistake in a sum, the error is + I or -1.

  • a. A
  • b. B
  • c. C
  • d. D
  • e.Not Attempted

53. How many among the four students A, B, C and D have passed the exam'?
A. The following is a true statement: A and B passed the exam.
B. The following is a false statement: At least one among C and D has passed the exam.

  • a. A
  • b. B
  • c. C
  • d. D
  • e.Not Attempted

54. What is the distance x between two cities A and B in integral number of Kms?
A. x satisfies the equation log 2 x = √x
B. x ≤ 10 Kms

  • a. A
  • b. B
  • c. C
  • d. D
  • e.Not Attempted

55. Mr. Mendel grew one hundred flowering plants from black seeds and white seeds, each seed giving rise to one plant. A plant gives flowers of only one colour. From a black seed comes a plant giving red or blue flowers. From a white seed comes a plant giving red or white flowers. How many black seeds were used by Mr. Mendel?
A. The number of plants with white flowers was 10.
B. The number of plants with red flowers was 70.

  • a. A
  • b. B
  • c. C
  • d. D
  • e.Not Attempted

CAT99 SECTION - ENGLISH

Instructions
1. The test comprises of 55 questions. You should complete the test within 40 minutes.
2. There is only one correct answer to each question.
3. All questions carry four marks each.
4. Each wrong answer will attract a penalty of one mark.

DIRECTIONS for questions I to 5:
Sentences given in each question, when properly sequenced, form a coherent paragraph. Each sentence is labelled with a letter. Choose the most logical order of sentence from among the four given choices to construct a coherent paragraph

A. In rejecting the functionalism in positivist organization theory, either wholly or partially, there is often a move towards a political model of organization theory.
B. Thus the analysis would shift to the power resources possessed by different groups in the organization and the way they use these resources in actual power plays to shape the organizational structure.
C. At the extreme, in one set of writings, the growth of administrators in the organization is held to be completely unrelated to the work to be done and to be caused totally by the political pursuit of self- interest.
D. The political model holds that individual interests are pursued in organizational life through the exercise of power and influence.

  • a. ADBC
  • b. CBAD
  • c. DBCA
  • d. ABDC
  • e.Not Attempted

2. A. Group decision making, however, does not necessarily fully guard against arbitrariness and anarchy, for individual capriciousness can get substituted by collusion of group members.
B. Nature itself is an intricate system of checks and balances, meant to preserve the delicate balance between various environmental factors that affect our ecology.
C. In institutions also, there is a need to have in place a system of checks and balances which inhibits the concentration of power in only some individuals.
D. When human interventions alter this delicate balance, the outcomes have been seen to be disastrous.

  • a. CDAB
  • b. BCAD
  • c. CABD
  • d. BDCA
  • e.Not Attempted

3. A. He was bone-weary and soul-weary, and found himself muttering, "Either I can't manage this place, or it's unmanageable."
B. To his horror, he realized that he had become the victim of an amorphous, unwitting, unconscious conspiracy to immerse him in routine work that had no significance.
C. It was one of those nights in the office. when -the office clock was moving towards four in the morning and Bennis was still not through with the incredible mass of paper stacked before him.
D. He reached for his calendar and ran his eyes down each hour, half-hour, and quarter-hour, to see where his time had gone that day, the day before, the month before

  • a. ABCD
  • b. CADB
  • c. BDCA
  • d. DCBA
  • e.Not Attempted

4. A. With that, I swallowed the shampoo, and obtained most realistic results almost on the spot.
B. The man shuffled away into the back regions to make up a prescription, and after a moment I got through on the shop-telephone to the Consulate, intimating my location.
C. Then, while the pharmacist was wrapping up a six-ounce bottle of the mixture, I groaned and inquired whether he could give me something for acute gastric cramp.
D. I intended to stage a sharp gastric attack, and entering an old-fashioned pharmacy, I asked for a popular shampoo mixture, consisting of olive oil and flaked soap.

  • a. DCBA
  • b. DACB
  • c. BDAC
  • d. BCDA
  • e.Not Attempted

5. A. Since then, intelligence tests have been mostly used to separate dull children in school from average or bright children, so that special education can be provided to the dull.
B. In other words, intelligence tests give us a norm for each age.
C. Intelligence is expressed as Intelligence quotient, and tests are developed to indicate what an average child of a certain age can do-what a 5-year-old can answer, but a 4year-old cannot, for instance.
D. Binet developed the first set of such tests in the early 1900s to find out which children in school needed special attention.
E. Intelligence can be measured by tests.

  • a. CDABE
  • b. DECAB
  • c. EDACB
  • d. CBADE
  • e.Not Attempted

DIRECTIONS for questions 6 to 13:
6. Three airlines - IA, JA and SA - operate on the Delhi-Mumbai route. To increase the number of seats sold, SA reduced its fares and this was emulated by IA and JA immediately. The general belief was that the volume of air travel between Delhi and Mumbai would increase as a result.
Which of the following, if true, would add credence to the general belief?

  • a. Increase in profitability of the three airlines.
  • b. Extension of the discount scheme to other routes.
  • c. A study that shows that air travellers in India are price-conscious.
  • d. A study that shows that as much as 80% of air travel in India is company-sponsored.
  • e.Not Attempted

7. According to McNeill, a Brahmin priest was expected to be able to recite at least one of the Vedas. The practice was essential for several centuries when the Vedas had not yet been written down. It must have had a selective effect, since priests would have been recruited from those able or willing to memorize long passages. It must have helped in the dissemination of the work, since a memorized passage can be duplicated many times.
Which one of the following can be inferred from the above passage?

  • a. Reciting the Vedas was a Brahmin's obligation.
  • b. The Vedic priest was like a recorded audio cassette.
  • c. McNeill studied the behaviour of Brahmin priests.
  • d. Vedic hymns had not been scripted.
  • e.Not Attempted

8.Developed countries have made adequate provisions for social security for senior citizens. State insurers (as well as private ones) offer medicare and pension benefits to people who can no longer earn. In India, with the collapse of the joint family system, the traditional shelter of the elderly has disappeared. And a State faced with a financial crunch is not in a position to provide social security. So, it is advisable that the working population give serious thought to building a financial base for itself.
Which one of the following, if it were to happen, weakens the conclusion drawn in the above passage the most?

  • a. The investable income of the working population, as a proportion of its total income, will grow in the future.
  • b. The insurance sector is underdeveloped and trends indicate that it will be extensively privatized in the future.
  • c. India is on a path of development that will take it to a developed country status, with all its positive and negative implications.
  • d. If the working population builds a stronger financial base, there will be a revival of the joint family system.
  • e.Not Attempted

9. Various studies have shown that our forested and hilly regions and, in general, areas where biodiversity—as reflected in the variety of flora—is high, are the places where poverty appears to be high. And these same areas are also the ones where educational performance seems to be poor. Therefore, it may be surmised that, even disregarding poverty status, richness in biodiversity goes hand in hand with educational backwardness.
Which one of the following statements, if true, can be said to best provide supporting evidence for the surmise mentioned in the passage?

  • a. In regions where there is little variety in flora, educational performance is seen to be as good as in regions with high variety in flora, when poverty levels are high.
  • b. Regions which show high biodiversity also exhibit poor educational performance, at low levels of poverty
  • c. Regions which show high biodiversity reveal high levels of poverty and poor educational performance
  • d. In regions where there is low biodiversity, at all levels of poverty, educational performance is seen to be good.
  • e.Not Attempted

10.Cigarettes constitute a mere 20% of tobacco consumption in India, and fewer than 15% of the 200 million tobacco users consume cigarettes., Yet these 15% contribute nearly 90% of the tax revenues to the Exchequer from the tobacco sector. The punitive cigarette taxation regime has kept the tax base narrow, and reducing taxes will expand this base.
Which one of the following best bolsters the conclusion that reducing duties will expand the tax base'?

  • a. The cigarette manufacturers’ association has decided to indulge in aggressive promotion.
  • b. There is a likelihood that tobacco consumers will shift to cigarette smoking if cigarette prices were to reduce
  • c. The cigarette manufacturers are lobbying for a reduction on duties.
  • d. An increase in duties on non-cigarette tobacco may lead to a shift in favour of cigarette smoking.
  • e.Not Attempted

11.Thomas Malthus, the British clergyman turned economist, predicted that the planet would not be able to support the human population for long. His explanation was that human population grows at a geometric rate, while the food supply grows only at an arithmetic rate.
Which one of the following, if true, would not undermine the thesis offered by Malthus?

  • a. Population growth can be slowed down by the voluntary choices of individuals and not just by natural disasters.
  • b. The capacity of the planet to feed a growing human population can be enhanced through biotechnological means.
  • c. Human systems, and natural systems like food supply, follow natural laws of growth which have remained constant, and will remain unchanged
  • d. Human beings can colonize other planetary systems on a regular and on-going basis to accommodate a growing population
  • e.Not Attempted

12. The company's coffee crop for 1998-99 totalled 8079 tonnes, an all time record. The increase over the previous year's production of 5830 tonnes was 38.58%. The previous highest crop was 6089 tonnes in 1970-7 1. The company had fixed a target of 8000 tonnes to be realized by the year 2000-01, and this has been achieved two years earlier, thanks to the emphasis laid on the key areas of irrigation, replacement of unproductive coffee bushes, intensive refilling and improved agricultural practices. It is now our endeavour to reach the target of 10000 tonnes in the year 2001-02.
Which one of the following would contribute most to making the target of I 0000 tonnes in 2001-02 unrealistic?

  • a. The potential of the productivity enhancing measures implemented up to now has been exhausted.
  • b. The total company land under coffee has remained constant since 1969 when an estate in the Nilgiri Hills was acquired.
  • c. The sensitivity of the crop to climatic factors makes predictions about production uncertain.
  • d. The target-setting procedures in the company have been proved to be sound by the achievement of the 8000 tonne target.
  • e.Not Attempted

13. Animals in general are shrewd in proportion as they cultivate society. Elephants and beavers show the greatest signs of this sagacity when they are together in large numbers, but when man invades their communities they lose all their spirit of industry. Among insects, the labours of the bee and the ant have attracted the attention and admiration of naturalists, but all their sagacity seems to be lost upon separation, and a single bee or ant seems destitute of every degree of industry. It becomes the most stupid insect imaginable,, and it languishes and soon dies.
Which of the following can be inferred from the above passage?

  • a. Humankind is responsible for the destruction of the natural habitat of animals and insects.
  • b. Animals, in general, are unable to function effectively outside their normal social environment.
  • c. Naturalists have great admiration for bees and ants, despite their lack of industry upon separation.
  • d. Elephants and beavers are smarter than bees and ants in the presence of human beings
  • e.Not Attempted

DIRECTIONS for questions 14 and 15:
For each of the two questions, indicate which of the statements given, with that particular question is consistent with the description of the unseasonable man in the passage below.

Unseasonableness is a tendency to do socially permissible things at the wrong time. The unseasonable man is the sort of person who comes to confide in you when you are busy. He serenades his beloved when she is ill. He asks a man who has just lost money by paying a bill for a friend to pay a bi II for him. He invites a friend to go for a ride just after the friend has finished a long car trip. He is eager to offer services which are not wanted but which cannot be politely refused. If he is present at an arbitration, he stirs up dissension between the two parties, who were really anxious to agree. Such is the unseasonable man.

14. He tends to

  • a. entertain women.
  • b. be a successful arbitrator when dissenting parties are anxious to agree.
  • c. be helpful when solicited.
  • d. tell a long story to people who have heard it many times before.
  • e.Not Attempted

15. The unseasonable man tends to

  • a. bring a higher bidder to a salesman who has just closed a deal.
  • b. disclose confidential information to others.
  • c. sing the praises of the bride when he goes to a wedding.
  • d. sleep late and rise early
  • e. Not Attempted

DIRECTIONS for questions 16 to 23:
In each of the following sentences, a part of the sentence is underlined. Beneath each sentence, four different ways of phrasing the underlined part are indicated. Choose the best alternative from among the four.

16. It was us who had left before he arrived

  • a. we who had left before time he had arrived.
  • b. us who had went before he arrived.
  • c. us who had went before had arrived
  • d. we who had left before he arrived.
  • e.Not Attempted

17. The MP rose up to say that, in her opinion, she thought the Women's Reservation Bill should be passed on unanimously.

  • a. rose to say that she thought the Women's Reservation Bill should be passed
  • b. rose up to say that, the Women's Reservation Bill should be passed on
  • c. rose to say that, in her opinion, she thought that the Women's Reservation Bill should be passed
  • d. rose to say that, in her opinion, the Women's Reservation Bill should be passed on
  • e.Not Attempted

18. Mr. Pillai, the president of the union and who is also a member of the community group, will be in charge of the negotiations.

  • a. since he is a member of the community group
  • b. also being a member of the community group
  • c. a member of the community group
  • d. in addition, who is a member of the community group
  • e.Not Attempted

19.Since the advent of cable television, at the beginning of this decade, the entertainment industry took a giant stride forward in our country.

  • a. this decade saw the entertainment industry taking
  • b. this decade, the entertainment industry has taken
  • c. this decade, the entertainment industry had taken
  • d. this decade, the entertainment industry took
  • e.Not Attempted

20. His mother made great sacrifices to educate him, moving house on three occasions, and severing the thread on her loom's shuttle whenever Mencius neglected his lessons to make him understand the need to persevere.

  • a. severing the thread on her loom's shuttle whenever Mencius neglected his lessons to make him understand the need to persevere.
  • b. severed the thread on her loom's shuttle whenever Mencius neglected his lessons to make him understand the need to persevere
  • c. severed the thread on her loom's shuttle whenever Mencius neglected his lessons to make him understand the need for persevering
  • d. severing the thread on her loom's shuttle whenever Mencius neglected his lessons, to make them understand the need to persevere
  • e.Not Attempted

21. If you are on a three-month software design project and, in two weeks, you've put together a programme that solves part of the problem, show it to your boss without delays.

  • a. and, you've put together a programme that solves part of the problem in two weeks
  • b. and, in two weeks, you've put together a programme that solves part of the problem
  • c.and, you've put together a programme that has solved part of the problem in two weeks
  • d. and, in two weeks you put together a programme that solved only part of the problem
  • e.Not Attempted

22. Many of these environmentalists proclaim to save nothing less than the planet itself

  • a. to save nothing lesser than
  • b. that they are saving nothing lesser than
  • c. to save nothing less than
  • d. that they save nothing less than
  • e.Not Attempted

23. Bacon believes that the medical profession should be permitted to ease and quicken death where the end would otherwise only delay for a few days and at the cost of great pain.

  • a. be delayed for a few days
  • b. be delayed for a few days and
  • c. be otherwise only delayed for a few days and
  • d. otherwise only delay for a few days and
  • e.Not Attempted

DIRECTIONS for questions 24 to 50:
Each of the five passages given below is followed by questions. For each question, choose the best answer.

PASSAGE I

The World Trade Organisation (WTO) was created in the early 1990s as a component of the Uruguay Round negotiation. However, it could have been negotiated as part of the Tokyo Round of the 1970s, since that negotiation was an attempt at a 'constitutional reform' of the General Agreement on Tariffs and Trade (GATT). Or it could have been put off to the future, as the US government wanted. What factors led to the creation of the WTO in the early 1990s?

One factor was the pattern of multilateral bargaining that developed late in the Uruguay Round. Like all complex international agreements, the WTO was a product of a series of trade-offs between principal actors and groups. For the United States, which did not want a new Organisation, the dispute settlement part of the WTO package achieved its longstanding goal of a more effective and more legal dispute settlement system. For the Europeans, who by the 1990s had come to view GATT dispute settlement less in political terms and more as a regime of legal obligations, the WTO package was acceptable as a means to discipline the resort to unilateral measures by the United States. Countries like Canada and other middle and smaller trading partners were attracted by the expansion of a rulesbased system and by the symbolic value of a trade Organisation, both of which inherently support the weak against the strong. The developing countries were attracted due to the provisions banning uni 'lateral measures. Finally, and perhaps most important, many countries at the Uruguay Round came to put a higher priority on the export gains than on the import losses that the negotiation would produce, and they came to associate the WTO and a rules-based system with those gains. This reasoning - replicated in many countries - was contained in U.S. Ambassador Kantor's defence of the WTO, and it amounted to a recognition that international trade and its benefits cannot be enjoyed unless trading nations accept the discipline of a negotiated rules-based environment.

A second factor in the creation of the WTO was pressure from lawyers and the legal process. The dispute settlement system of the WTO was seen as a victory of legalists over pragmatists but the matter went deeper than that. The GATT, and the WTO, are contract organisations based on rules, and it is inevitable that an Organisation created to further rules will in turn be influenced by the legal process. Robert Hudec has written of the 'momentum of legal development', but what is this precisely? Legal development can be defined as promotion of the technical legal values of consistency, clarity (or, certainty) and effectiveness; these are values that those responsible for administering any legal system will seek to maximise. As it played out in the WTO, consistency meant integrating under one roof the whole lot of separate agreements signed under GATT auspices; clarity meant removing ambiguities about the powers of contracting parties to make certain decisions or to undertake waivers; and effectiveness meant eliminating exceptions arising out of grandfather-rights and resolving defects in dispute settlement procedures and institutional provisions. Concern for these values is inherent in any rules-based system of co-operation, since without these values rules would be meaningless in the first place. Rules, therefore, create their own incentive for fulfilment.

The momentum of legal development has occurred in other institutions besides the GATT, most notably in the European Union (EU). Over the past two decades the European Court of Justice (ECJ) has consistently rendered decisions that have expanded incrementally the EU's internal market, in which the doctrine of 'mutual recognition' handed down in the case Cassis de Dijon in 1979 was a key turning point. The Court is now widely recognised as a major player in European integration, even though arguably such a strong role was not originally envisaged in the Treaty of Rome, which initiated the current European Union. One means the Court used to expand integration was the 'teleological method of interpretation', whereby the actions of member states were evaluated against 'the accomplishment of the most elementary community goals set forth in the Preamble to the [Rome] treaty'. The teleological method represents an effort to keep current policies consistent with stated goals, and it is analogous to the effort in GATT to keep contracting party trade practices consistent with stated rules. In both cases legal concerns and procedures are an independent force for further cooperation.

In large part the WTO was an exercise in consolidation. In the context of a trade negotiation that created a near- revolutionary expansion of international trade rules, the formation of the WTO was a deeply conservative act needed to ensure that the benefits of the new rules would not be lost. The WTO was all about institutional structure and dispute settlement: these are the concerns of conservatives and not revolutionaries, which is why lawyers and legalists took the lead on these issues. The WTO codified the GATT institutional practice that had developed by custom over three decades, and it incorporated a new dispute settlement system that was necessary to keep both old and new rules from becoming a sham. Both the international structure and the dispute settlement system were necessary to preserve and enhance the integrity of the multilateral trade regime that had been built incrementally from the 1940s to the 1990s.

24. What could be the closest reason why the WTO was not formed in the 1970s?

  • a. The US government did not like it.
  • b.Important players did not find it in their best interest to do so.
  • c. Lawyers did not work for the dispute settlement system.
  • d. The Tokyo Round negotiation was an attempt at constitutional reform.
  • e.Not Attempted
25. The most likely reason for the acceptance of the WTO package by nations was that
  • a. it had the means to prevent the US from taking unilateral measures.
  • b. they recognized the need for a rule-based environment to protect the benefits of increased trade.
  • c. it settles disputes more legally and more effectively.
  • d. its rule-based system leads to export gains.
  • e.Not Attempted

26. According to the passage, WTO promoted the technical legal values partly through

  • a. integrating under one roof the agreements signed under GATT.
  • b. rules that create their own incentive for fulfilment.
  • c. grandfather-rights exceptions and defects in dispute settlement procedures.
  • d. ambiguities about the powers of contracting parties to make certain decisions.
  • e.Not Attempted

27. In the method of interpretation of the European Court of Justice,

  • a. current policies needed to be consistent with stated goals.
  • b.contracting party trade practices needed to be consistent with stated rules.
  • c. enunciation of the most elementary community goals needed to be emphasized.
  • d. actions of member states needed to be evaluated against the stated community goals.
  • e.Not Attempted

28. In the statement "...it amounted to a recognition that international trade and its benefits cannot be enjoyed unless trading nations accept the discipline of a negotiated rules-based environment.", ‘it' refers to:

  • a. Ambassador Kantor's defence of the WTO.
  • b. The higher priority on export gains placed by many countries at the Uruguay Round.
  • c. The export gains many countries came to associate with a rule-based system.
  • d. The provision of a rule-based system by the WTO.
  • e.Not Attempted

29. The importance of Cassis de Dijon is that it

  • a. gave a new impetus to the momentum of legal development at the European Court of Justice.
  • b. resulted in a decision that expanded incrementally the EU's internal marketOther E. U.
  • c. strengthened the role of the Court more than envisaged in the Treaty of Rome.
  • d. led to a doctrine that was a key turning point in European integration
  • e.Not Attempted

PASSAGE II

Have you ever come across a painting, by Picasso, Mondrian, Miro, or any other modem abstract painter of this century, and found yourself engulfed in a brightly coloured canvas which your senses cannot interpret? Many people would tend to denounce abstractionism as senseless trash. These people are disoriented by Miro's bright, fanciful creatures and two- dimensional canvases. They click their tongues and shake their heads at Mondrian's grid works, declaring the poor guy played too many scrabble games. They silently shake their heads in sympathy for Picasso, whose gruesome, distorted figures must be a reflection of his mental health. Then, standing in front of a work by Charlie Russell, the famous Western artist, they'll declare it a work of God. People feel more comfortable with something they can relate to and understand immediately without too much thought. This is the case with the work of Charlie Russell. Being able to recognize the elements in his paintings--trees, horses and cowboys-gives people a safety line to their world of "reality". There are some who would disagree when I say abstract art requires more creativity and artistic talent to produce a good piece than does representational art, but there are many weaknesses in their arguments.

People who look down on abstract art have several major arguments to support their beliefs. They feel that artists turn abstract because they are not capable of the technical drafting skills that appear in a Russell; therefore, such artists create an art form that anyone is capable of and that is less time consuming, and then parade it as artistic progress. Secondly, they feel that the purpose of art is to create something of beauty in an orderly, logical composition. Russell's compositions are balanced and rational, everything sits calmly on the canvas, leaving the viewer satisfied that he has seen all there is to see. The modem abstractionists, on the other hand, seem to compose their pieces irrationally. For example, upon seeing Picasso's Guernica, a friend of mine asked me, "What's the point?" Finally, many people feel that art should portray the ideal and real. The exactness of detail in Charlie Russell's work is an example of this. He has been called a great historian because his pieces depict the life style, dress, and events of the times. His subject matter is derived from his own experiences on the trail, and reproduced to the smallest detail.

I agree in part with many of these arguments, and at one time even endorsed them. But now, I believe differently. Firstly I object to the argument that abstract artists are not capable of drafting. Many abstract artists, such as Picasso, are excellent draftsmen. As his work matured, Picasso became more abstract in order to increase the expressive quality of his work. Guernica was meant as a protest against the bombing of that city by the Germans. To express the terror and suffering of the victims more vividly, he distorted the figures and presented them in a black and white journalistic manner. If he had used representational images and colour, much of the emotional content would have been lost and the piece would not have caused the demand for justice that it did. Secondly, I do not think that a piece must be logical and aesthetically pleasing to be art. The message it conveys to its viewers is more important. It should reflect the ideals and issues of its time and be true to itself, not just a flowery, glossy surface. For example, through his work, Mondrian was trying to present a system of simplicity, logic, and rational order. As a result, his pieces did end up looking like a scrabble board.

Miro created powerful, surrealistic images from his dreams and subconscious. These artists were trying to evoke a response from society through an expressionistic manner. Finally,-abstract artists and representational artists maintain different ideas about 'reality'. To the representational artist, reality is what he sees with his eyes. This is the reality he reproduces on canvas. To the abstract artist, reality is what he feels about what his eyes see. This is the reality he interprets on canvas. This can be illustrated by Mondrian's Trees series. You can actually see the progression from the early recognizable, though abstracted, Trees, to his final solution, the grid system.

A cycle of abstract and representational art began with the first scratchings of prehistoric man. From the abstractions of ancient Egypt to representational, classical Rome, returning to abstractionism in early Christian art and so on up to the present day, the cycle has been going on. But this day and age may witness its death through the camera. With film, there is no need to produce finely detailed, historical records manually; the camera does this for us more efficiently. Maybe, representational art would cease to exist. With abstractionism as the victor of the first battle, may be a different kind of cycle will be touched off. Possibly, some time in the distant future, thousands of years from now, art itself will be physically non- existent. Some artists today believe that once they have planned and constructed a piece in their mind, there is no sense in finishing it with their hands; it has already been done and can never be duplicated.

30. The author argues that many people look down upon abstract art because they feel that:

  • a. Modem abstract art does not portray what is ideal and real.
  • b. Abstract artists are unskilled in matters of technical drafting.
  • c. Abstractionists compose irrationally.
  • d. All of the above.
  • e.Not Attempted

31. The author believes that people feel comfortable with representational art because

  • a. they are not engulfed in brightly coloured canvases.
  • b. they do not have to click their tongues and shake their heads in sympathy.
  • c. they understand the art without putting too much strain on their minds
  • d. paintings like Guernica do not have a point
  • e.Not Attempted

32. In the author's opinion, Picasso's Guernica created a strong demand for justice since

  • a. it was a protest against the German bombing of Guernica.
  • b. Picasso managed to express the emotional content well with his abstract depiction.
  • c. it depicts the terror and suffering of the victims in a distorted manner
  • d. it was a mature work of Picasso's, painted when the artist's drafting skills were excellent..
  • e.Not Attempted

33.The author acknowledges that Mondrian's pieces may have ended up looking like a scrabble board because

  • a. many people declared the poor guy played too many scrabble games.
  • b. Mondrian believed in the 'grid-works' approach to abstractionist painting.
  • c. Mondrian was trying to convey the message of simplicity and rational order.
  • d. Mondrian learned from his Trees series to evolve a grid system.
  • e.Not Attempted

34. The main difference between the abstract artist and the representational artist in matters of the 'ideal' and the 'real', according to the author, is:

  • a. How each chooses to deal with 'reality' on his or her canvas.
  • b. The superiority of interpretation of reality over reproduction of reality.
  • c. The different values attached by each to being a historian.
  • d. The varying levels of drafting skills and logical thinking abilities.
  • e.Not Attempted

PASSAGE III

Each one has his reasons: for one art is a flight; for another, a means of conquering. But one can flee into a hermitage, into madness, into death. One can conquer by arms. Why does it have to be writing, why does one have to manage his escapes and conquests by writing? Because, behind the various alms of authors, there is a deeper and more immediate choice which is common to all of us. We shall try to elucidate this choice, and we shall see whether it is not in the name of this very choice of writing that the engagement of writers must be required.

Each of our perceptions is accompanied by the consciousness that human reality is a 'revealer', that is, it is through human reality that 'there is' being, or, to put it differently, that man is the means by which things are manifested. It is our presence in the world which multiplies relations. It is we who set up a relationship between this tree and that bit of sky. Thanks to us, that star which has been dead for millenia, that quarter moon, and that dark river are disclosed in the unity of a landscape. It is the speed of our auto and our airplane which organizes the great masses of the earth. With each of our acts, the world reveals to us a new face. But, if we know that we are directors of being, we also know that we are not its producers. If we turn away from this landscape, it will sink back into its dark permanence. At least, it will sink back; there is no one mad enough to think that it is going to be annihilated. It is we who shall be annihilated, and the earth will remain in its lethargy until another consciousness comes along to awaken it. Thus, to our inner certainty of being 'revealers' is added that of being inessential in relation to the thing revealed.

One of the chief motives of artistic creation is certainly the need of feeling that we are essential in relationship to the world. If I fix on canvas or in writing a certain aspect of the fields or the sea or a look on someone's face which I have disclosed, I am conscious of having produced them by condensing relationships, by introducing order where there was none, by imposing the unity of mind on the diversity of things. That is, I think myself essential in relation to my creation. But this time it is the created object which escapes me; I can not reveal and produce at the same time. The creation becomes inessential in relation to the creative activity. First of all, even if it appears to others as definitive, the created object always seems to us in a state of suspension; we can always change this line, that shade, that word. Thus, it never forces itself. A novice painter asked his teacher, 'When should I consider my painting finished?' And the teacher answered, 'When you can look at it in amazement and say to yourself "I'm the one who did that!...

Which amounts to saying 'never'. For it is virtually considering one's work with someone else's eyes and revealing what has been created. But it is self-evident that we are proportionally less conscious of the thing produced and more conscious of our productive activity. When it is a matter of poetry or carpentry, we work according to traditional nonns, with tools whose usage is codified; it is Heidegger's famous 'they' who are working with our hands. In this case, the result can seem to us sufficiently strange to preserve its objectivity in our eyes. But if we ourselves produce the rules of production, the measures, the criteria, and if our creative drive comes from the very depths of our heart, then we never find anything but ourselves in our work. It is we who have invented the laws by which we judge it. It is our history, our love, our gaiety that we recognize in it. Even if we should regard it without touching it any further, we never receive from it that gaiety or love. We put them into it. The results which we have obtained on canvas or paper never seem to us objective. We are too familiar with the processes of which they are the effects. These processes remain a subjective discovery; they are ourselves, our inspiration, our ruse, and when we seek to perceive our work, we create it again, we repeat mentally the operations which produced it; each of its aspects appears as a result. Thus, in the perception, the object is given as the essential thing and the subject as the inessential. The latter seeks essentiality in the creation and obtains it, but then it is the object which becomes the inessential.

The dialectic is nowhere more apparent than in the art of writing, for the literary object is a peculiar top which exists only in movement. To make it come into view a concrete act called reading is necessary, and it lasts only as long as this act can last. Beyond that, there are only black marks on paper. Now, the writer can not read what be writes, whereas the shoemaker can put on the shoes he has just made if they are to his size, and the architect can live in the house he has built. In reading, one foresees; one waits. He foresees the end of the sentence, the following sentence, the next page. He waits for them to confirm or disappoint his foresights. The reading is composed of a host of hypotheses, followed by awakenings, of hopes and deceptions, Readers are always ahead of the sentence they are reading in a merely probable future which partly collapses and partly comes together in proportion as they progress, which withdraws from one page to the next and forms the moving horizon of the literary object. Without waiting, without a future, without ignorance, there is no objectivity.

35. The author holds that:

  • a. There is an objective reality and a subjective reality.
  • b. Nature is the sum total of disparate elements.
  • c. It is human action that reveals the various facets of nature.
  • d. Apparently disconnected elements in nature are unified in a fundamental sense.
  • e. Not Attempted

36.It is the author's contention that:

  • a. Artistic creations are results of human consciousness.
  • b. The very act of artistic creation leads to the escape of the created object
  • c. Man can produce and reveal at the same time.
  • d. An act of creation forces itself on our consciousness leaving us full of amazement.
  • e.Not Attempted

37. The passage makes a distinction between perception and creation in terms of

  • a. Objectivity and subjectivity.
  • b. Revelation and action.
  • c. Objective reality and perceived reality.
  • d.Essentiality and non-essentiality of objects and subjects.
  • e.Not Attempted

38. The art of writing manifests the dialectic of perception and creation because

  • a. reading reveals the writing till the act of reading lasts.
  • b. writing to be meaningful needs the concrete act of reading.
  • c. this art is anticipated and progresses on a series of hypotheses.
  • d. this literary object has a moving horizon brought about by the very act of creation.
  • e.Not Attempted

39.A writer, as an artist,

  • a. reveals the essentiality of revelation.
  • b. makes us feel essential vis-d-vis nature.
  • c. creates reality.
  • d. reveals nature in its permanence.
  • e.Not Attempted

PASSAGE IV

Since World War II, the nation-state has been regarded with approval by every political system and every ideology. In the name of modernisation in the West, of socialism in the Eastern bloc, and of development in the Third World, it was expected to guarantee the happiness of individuals as citizens and of peoples as societies. However, the state today appears to have broken down in many parts of the world. It has failed to guarantee either security or social justice, and has been unable to prevent either international wars or civil wars. Disturbed by the claims of communities within it, the nation-state tries to repress their demands and to proclaim itself as the only guarantor of security of all. In the name of national unity, territorial integrity, equality of all its citizens and non-partisan secularism, the state can use its powerful resources to reject the demands of the communities; it may even go so far as genocide to ensure that order prevails.

As one observes the awakening of communities in different parts of the world, one cannot ignore the context in which identity issues arise. It is no longer a context of sealed frontiers and isolated regions but is one of integrated global systems. In a reaction to this trend towards globalisation, individuals and communities everywhere are voicing their desire to exist, to use their power of creation and to play an active part in national and international life.

There are two ways in which the current upsurge in demands for the recognition of identities can be looked at. On the positive side, the efforts by certain population groups to assert their identity can be regarded as "liberation movements", challenging oppression and injustice. What these groups are doing - proclaiming that they are different, rediscovering the roots of their culture or strengthening group solidarity - may accordingly be seen as legitimate attempts to escape from their state of subjugation and enjoy a certain measure of dignity. On the downside, however, militant action for recognition tends to make such groups more deeply entrenched in their attitude and to make their cultural compartments even more watertight. The assertion of identity then starts turning into self-absorption and isolation, and is liable to slide into intolerance of others and towards ideas of "ethnic cleansing", xenophobia and violence.

Whereas continuous variations among peoples prevent drawing of clear dividing lines between the groups, those militating for recognition of their group's identity arbitrarily choose a limited number of criteria such as religion, language, skin colour, and place of origin so that their members recognise themselves primarily in terms of the labels attached to the group whose existence is being asserted. This distinction between the group in question and other groups is established by simplifying the feature selected. Simplification also works by transforming groups into essences, abstractions endowed with the capacity to remain unchanged through time. In some cases, people actually act as though the group has remained unchanged and talk, for example, about the history of nations and communities as if these entities survived for centuries without changing, with the same ways of acting and thinking, the same desires, anxieties, and aspirations.

Paradoxically, precisely because identity represents a simplifying fiction, creating uniform groups out of disparate people, that identity performs a cognitive function. It enables us to put names to ourselves and others, form some idea of who we are and who others are, and ascertain the place we occupy along with the others in the world and society. The current upsurge to assert the identity of groups can thus be partly explained by the cognitive function performed by identity. However, that said, people would not go along as they do, often in large numbers, with the propositions put to them, in spite of the sacrifices they entail, if there was not a very strong feeling of need for identity, a need to take stock of things and know "who we are", "where we come from", and "where we are going".

Identity is thus a necessity in a constantly changing world, but it can also be a potent source of' violence and disruption. How can these two contradictory aspects of identity be reconciled? First, we must bear the arbitrary nature of identity categories in mind, not with a view to eliminating all forms of identification—which would be unrealistic since identity is a cognitive necessity—but simply to remind ourselves that each of us has several identities at the same time. Second, since tears of nostalgia are being shed over the past, we recognise that culture is constantly being recreated by cobbling together fresh and original elements and counter-cultures. There are in our own country a large number of syncretic cults wherein modem elements are blended with traditional values or people of different communities venerate saints or divinities of particular faiths. Such cults and movements are characterised by a continual inflow and outflow of members which prevent them from taking on a self-perpetuating existence of their own and hold out hope for the future, indeed, perhaps for the only possible future. Finally, the nation-state must respond to the identity urges of its constituent communities and to their legitimate quest for security and social justice. It must do so by inventing what the French philosopher and sociologist, Raymond Aron, called "peace through law". That would guarantee justice both to the state as a whole and its parts, and respect the claims of both reason and emotions. The problem is one of reconciling nationalist demands with the exercise of democracy.

40. According to the author, happiness of individuals was expected to be guaranteed in the name of:

  • a. Development in the Third world
  • b. Socialism in the Third world.
  • c. Development in the West.
  • d. Modernisation in the Eastern Bloc.
  • e.Not Attempted

41. Demands for recognition of identities can be viewed:

  • a. Positively and negatively.
  • b. As liberation movements and militant action.
  • c. As efforts to rediscover cultural roots which can slide towards intolerance of others.
  • d. All of the above.
  • e.Not Attempted

42. Going by the author's exposition of the nature of identity, which of the following statements is untrue?

  • a. Identity represents creating uniform groups out of disparate people.
  • b. Identity is a necessity in the changing world.
  • c. Identity is a cognitive necessity.
  • d. None of the above.
  • e.Not Attempted

43. According to the author, the nation-state

  • a. has fulfilled its potential.
  • b. is willing to do anything to preserve order.
  • c. generates security for all its citizens.
  • d. has been a major force in preventing civil and international wars.
  • e.Not Attempted

44. Which of the following views of the nation-state cannot be attributed to the author?

  • a. It has not guaranteed peace and security.
  • b. It may go as far as genocide for self-preservation.
  • c. It represents the demands of communities within it.
  • d. It is unable to prevent international wars.
  • e.Not Attempted

PASSAGE V

The persistent patterns in the way nations fight reflect their cultural and historical traditions and deeply rooted attitudes that collectively make up their strategic culture. These patterns provide insights that go beyond what can be learnt just by comparing armaments and divisions. In the Vietnam War, the strategic tradition of the United States called for forcing the enemy to fight a massed battle in an open area, where superior American weapons would prevail. The United States was trying to re-fight World War II in the jungles of Southeast Asia, against an enemy with no intention of doing so.

Some British military historians describe the Asian way of war as one of indirect attacks, avoiding frontal attacks meant to overpower an opponent. This traces back to Asian history and geography: the great distances and harsh terrain have often made it difficult to execute the sort of open field clashes allowed by the flat terrain and relatively compact size of Europe. A very different strategic tradition arose in Asia.

The bow and arrow were metaphors for an Eastern way of war. By its nature, the arrow is an indirect weapon. Fired from a distance of hundreds of yards, it does not necessitate immediate physical contact with the enemy. Thus, it can be fired from hidden positions. When fired from behind a ridge, the barrage seems to come out of nowhere, taking the enemy by surprise. The tradition of this kind of fighting is captured in the classical strategic writings of the East. The 2,000 years' worth of Chinese writings on war constitutes the most subtle writings on the subject in any language. Not until Clausewitz, did the West produce a strategic theorist to match the sophistication of Sun-tzu, whose Art of War was written 2,300 years earlier.

In Sun-tzu and other Chinese writings, the highest achievement of arms is to defeat an adversary without fighting. He wrote: "To win one hundred victories in one hundred battles is not the acme of skill. To subdue the enemy without fighting is the supreme excellence." Actual combat is just one among many means towards the goal of subduing an adversary. War contains too many surprises to be a first resort. It can lead to ruinous losses, as has been seen time and again. It can have the unwanted effect of inspiring heroic efforts in an enemy, as the United States learned in Vietnam, and as the Japanese found out after Pearl Harbor.

Aware of the uncertainties of a military campaign, Sun-tzu advocated war only after the most thorough preparations. Even then it should be quick and clean. Ideally, the army is just an instrument to deal the final blow to an enemy already weakened by isolation, poor morale, and disunity. Ever since Sun-tzu, the Chinese have been seen as masters of -subtlety who take measured actions to manipulate an adversary without his knowledge. The dividing line between war and peace can be obscure. Low level violence often is the backdrop to a larger strategic campaign. The unwitting victim, focused on the day-to-day events, never realizes what's happening to him until it's too late. History holds many examples. The Viet Cong lured French and U.S. infantry deep into the jungle, weakening their morale over several years. The mobile army of the United States was designed to fight on the plains of Europe, where it could quickly move unhindered from one spot to the next. The jungle did more than make quick movement impossible; broken down into smaller units and scattered in isolated bases, US forces were deprived of the feeling of support and protection that ordinarily comes from being part of a big army.

The isolation of U.S. troops in Vietnam was not just a logistical 'detail, something that could be overcome by, for instance, bringing in reinforcements by helicopter. In a big army reinforcements are readily available. It was Napoleon who realized the extraordinary effects on morale that come from being part of a larger formation. Just the knowledge of it lowers the soldier's fear and increases his aggressiveness. In the jungle and on isolated bases, this feeling was removed. The thick vegetation slowed down the reinforcements and made it difficult to find stranded units. Soldiers felt they were on their own.

More important, by altering the way the war was fought, the Viet Cong stripped the United States of its belief in the inevitability of victory, as it had done to the French before them. Morale was high when these armies first went to Vietnam. Only after many years of debilitating and demoralizing fighting did Hanoi launch its decisive attacks, at Dienbienphu in 1954 and against Saigon in 1975. It should be recalled that in the final push to victory the North Vietnamese abandoned their jungle guerrilla tactics completely, committing their entire army of twenty divisions to pushing the South Vietnamese into collapse. This final battle, with the enemy's army all in one place, was the one that the United States had desperately wanted to fight in 1965. When it did come out into the open in 1975, Washington had already withdrawn its forces and there was no possibility of re-intervention.

The Japanese early in World War 11 used a modem form of the indirect attack, one that relied on stealth and surprise for its effect. At Pearl Harbor, in the Philippines, and in Southeast Asia, stealth and surprise were attained by sailing under radio silence so that the navy's movements could not be tracked. Moving troops aboard ships into Southeast Asia made it appear that the Japanese army was also "invisible." Attacks against Hawaii and Singapore seemed, to the American and British defenders, to come from nowhere. In Indonesia and the Philippines the Japanese attack was even faster than the German blitz against France in the West.

The greatest military surprises in American history have all been in Asia. Surely there is something going on here beyond the purely technical difficulties of detecting enemy movements. Pearl Harbor, the Chinese intervention in Korea, and the Tet offensive in Vietnam all came out of a tradition of surprise and stealth. U.S. technical intelligence – the location of enemy units and their movements was greatly improved after each surprise, but with no noticeable improvement in the American ability to foresee or prepare what would happen next. There is a cultural divide here, not just a technical one. Even when it was possible to track an army with intelligence satellites, as when Iraq invaded Kuwait or when Syria and Egypt attacked Israel, surprise was achieved. The United States was stunned by Iraq's attack on Kuwait even though it had satellite pictures of Iraqi troops massing at the border.

The exception that proves the point that cultural differences obscure the West's understanding of Asian behavior was the Soviet Union's 1979 invasion of Afghanistan. This was fully anticipated and understood in advance. There was no surprise because the United States understood Moscow's world view and thinking. It could anticipate Soviet action almost as well as the Soviets themselves, because the Soviet Union was really a Western country.

The difference between the Eastern and the Western way of war is striking. The West's great strategic writer, Clausewitz, linked war to politics, as did Sun-tzu. Both were opponents of militarism, of turning war over to the generals. But there all similarity ends. Clausewitz wrote that the way to achieve a larger political purpose is through destruction of the enemy's army. After observing Napoleon conquer Europe by smashing enemy armies to bits, Clausewitz made his famous remark in On War (1932) that combat is the continuation of politics by violent means. Morale and unity are important, but they should be harnessed for the ultimate battle. If the Eastern way of war is embodied by the stealthy archer, the metaphorical Western counterpart is the swordsman charging forward, seeking a decisive showdown, eager to administer the blow that will obliterate the enemy once and for all. In this view, war proceeds along a fixed course and occupies a finite extent of time, like a play in three acts with a beginning, a middle, and an end. The end, the final scene, decides the issue for good.

When things don't work out quite this way, the Western military mind feels tremendous frustration. Sun-tzu's great disciples, Mao Zedong and Ho Chi Minh, are respected in Asia for their clever use of indirection and deception to achieve an advantage over stronger adversaries. But in the West their approach is seen as underhanded and devious. To the American strategic mind, the Viet Cong guerrilla did not fight fairly. He should have come out into the open and fought like a man, instead of hiding in the jungle and sneaking around like a cat in the night.

45. According to the author, the main reason for the U.S. losing the Vietnam war was

  • a. the Vietnamese understood the local terrain better.
  • b. the lack of support for the war from the American people.
  • c. the failure of the U.S. to mobilize its military strength.
  • d. their inability to fight a war on terms other than those they understood well.
  • e.Not Attempted

46. Which of the following statements does not describe the 'Asian' way of war?

  • a. Indirect attacks without frontal attacks.
  • b. The swordsman charging forward to obliterate the enemy once and for all.
  • c. Manipulation of an adversary without his knowledge.
  • d. Subduing an enemy without fighting.
  • e.Not Attempted

47. Which of the following is not one of Sun-tzu's ideas?

  • a. Actual combat is the principal means of subduing an adversary.
  • b. War should be undertaken only after thorough preparation.
  • c. War is linked to politics.
  • d. War should not be left to the generals alone.
  • e.Not Attempted

48. The difference in the concepts of war of Clausewitz and Sun-tzu is best characterized by

  • a. Clausewitz's support for militarism as against Sun-tzu's opposition to it.
  • b. their relative degrees of sophistication.
  • c. their attitude to guerrilla warfare.
  • d. their differing conceptions of the structure, time and sequence of a war.
  • e.Not Attempted

49. To the Americans, the approach of the Viet Cong seemed devious because

  • a. the Viet Cong did not fight like men out in the open.
  • b. the Viet Cong allied with America's enemies.
  • c. the Viet Cong took strategic advice from Mao Zedong
  • d. the Viet Cong used bows and arrows rather than conventional weapons.
  • e.Not Attempted

50. According to the author, the greatest military surprises in American history have been in Asia because

  • a. The Americans failed to implement their military strategies many miles away from their own country.
  • b. The Americans were unable to use their technologies like intelligence satellites effectively to detect enemy movements
  • c. The Americans failed to understand the Asian culture of war that was based on stealth and surprise.
  • d. Clausewitz is inferior to Sun-tzu.
  • e.Not Attempted

DIRECTIONS for questions 51 to 55:
Arrange the sentences A, B, C and D to form a logical sequence between sentences 1 and 6.

51. 1. Making people laugh is tricky.
A. At times, the intended humour may simply not come off.
B. Making people laugh while trying to sell them something is a tougher challenge, since the commercial can fall flat on two grounds.
C. There are many advertisements which do amuse but do not even begin to set the cash ti ills ringing.
D. Again, it is rarely sufficient for an advertiser simply to amuse the target audience in order to reap the sales benefit.
6. There are indications that in substituting the hard sell for a more entertaining approach, some agencies have rather thrown out the baby with the bath water.

  • a. CDBA
  • b. ABCD
  • c. BADC
  • d. DCBA
  • e.Not Attempted

52. 1. Picture a termite colony, occupying a tall mud hump on an African plain. A. Hungry predators often invade the colony and unsettle the balance.
B. The colony flourishes only if the proportion of soldiers to workers remains roughly the same, so that the queen and workers can be protected by the soldiers, and the queen and soldiers can be serviced by the workers.
C. But its fortunes are presently restored, because the immobile queen, walled in well below ground level, lays eggs not only in large enough numbers, but also in the varying proportions required.
D. The hump is alive with worker termites and soldier termites going about their distinct kinds of business.
6. How can we account for her mysterious ability to respond like this to events on the distant surface?

  • a. BADC
  • b. DBAC
  • c. ADCB
  • d. BDCA
  • e.Not Attempted

53. 1. According to recent research, the critical period for developing language skills is between the ages of three and five and a half years.
A. The read-to child already has a large vocabulary and a sense of grammar and sentence structure.
B. Children who are read to in these years have a far better chance of reading well in school, indeed, of doing well in all their subjects.
C. And the reason is actually quite simple.
D. This correlation is far and away the highest yet found between home influences and school success.
6. Her comprehension of language is therefore very high.

  • a. DACB
  • b. ADCB
  • c. ABCD
  • d. BDCA
  • e.Not Attempted

54.1. High-powered outboard motors were considered to be one of the major threats to the survival of the Beluga whales.
A. With these, hunters could approach Belugas within hunting range and profit from its inner skin and blubber.
B. To escape an approaching motor, Belugas have learned to dive to the ocean bottom and stay there for up to 20 minutes, by which time the confused predator has left.
C. Today, however, even with much more powerful engines, it is difficult to come close, because the whales seem to disappear suddenly just when you thought you had them in your sights.
D. When the first outboard engines arrived in the early 1930s, one came across 4 and 8 HP motors.
6. Belugas seem to have used their well-known sensitivity to noise to evolve an ‘avoidance' strategy to outsmart hunters and their powerful technologies.

  • a. DACB
  • b. CDAB
  • c. ADBC
  • d. BDAC
  • e.Not Attempted

55. 1. The reconstruction of history by post-revolutionary science texts involves more than a multiplication of historical misconstructions.
A. Because they aim quickly to acquaint the student with what the contemporary scientific community thinks it knows, textbooks treat the various experiments, concepts, laws and theories of the current normal science as separately and as nearly seriatim as possible.
B. Those misconstructions render revolutions invisible; the arrangement of the still visible material in science texts implies a process that, if it existed, would deny revolutions a function.
C. But when combined with the generally unhistorical air of science writing and with the occasional systematic misconstruction, one impression is likely to follow.
D. As pedagogy this technique of presentation is unexceptionable.
6. Science has reached its present state by a series of individual discoveries and inventions that, when gathered together, constitute the modem body of technical knowledge.

  • a. BADC
  • b. ADCB
  • c. DACB
  • d. CBDA
  • e.Not Attempted

CAT2001 SECTION - MATHEMATICS

Instructions
1. The test comprises of 50 questions. You should complete the test within 40 minutes.
2. There is only one correct answer to each question.
3. All questions carry four marks each.
4. Each wrong answer will attract a penalty of one mark.

Directions from questions 1 to 5:
Each question is independent of each other.

1.A ladder leans against a vertical wall. The top of the ladder is 8m above the ground. When the bottom of the ladder is moved 2m farther away from the wall, the top of the ladder rests against the foot of the wall. What is the length of the ladder?

  • a. 10m
  • b. 15m
  • c. 20m
  • d. 17m
  • e.Not Attempted

2.Ujakar and Keshav attempted to solve a quadratic equation. Ujakar made a mistake in writing down the constant term. He ended up with the roots (4,3). Keshav made a mistake in writing down the coefficient of x. He got the roots as (3,2). What will be the exact roots of the original quadratic equation?

  • a. (6,1)
  • b. (-3, -4)
  • c. (4,3)
  • d. (-4, -3)
  • e.Not Attempted

3. A student took five papers in an examination, where the full marks were the same for each paper. His marks in these papers were in the proportion of 6:7:8:9:10. In all papers together, the candidate obtained 60% of the total marks. Then the number of papers in which he got more than 50% marks is:

  • a. 2
  • b. 3
  • c. 4
  • d. 5
  • e.Not Attempted

4.A certain city has a circular wall around it, and the wall has four gates pointing north, south, east and west. A house stands outside the city, three kms north of the north gate, and it can just be seen from a point nine kms east of the South Gate. What is the diameter of the wall that surrounds the city?

  • a. 6 km
  • b. 9 km
  • c. 12 km
  • d. None of these
  • e.Not Attempted

5. Let x, y and z be distinct integers, x and y are odd and positive, and z is even and positive. Which one of the following statements can not be true?

  • a. (x - z)2 y is even
  • b. (x - z)y2 is odd
  • c. (x - z)y is odd
  • d. (x - y)2 z is even
  • e.Not Attempted

6. A square, whose side is 2 meters, has its corners cut away so as to form an octagon with all sides equal. Then the length of each side of the octagon, in meters is:

  • a. (√2)/(√2 + 1)
  • b. (2)/(√2 + 1)
  • c. (2)/(√2 - 1)
  • d. (√2)/(√2 - 1)
  • e.Not Attempted

7. All the page numbers from a book are added, beginning at page 1. However, one page number was mistakenly added twice. The sum obtained was1000. Which page number was added twice?

  • a. 44
  • b. 45
  • c. 10
  • d. 12
  • e.Not Attempted

8. x and y are real numbers satisfying the conditions 2< x < 3 and -8 < y < -7. Which of the following expressions will have the least value?

  • a. x2y
  • b. xy2
  • c. 5xy
  • d. None of these
  • e.Not Attempted

9.In a number system the product of 44 and 11 is 1034. The number 3111 of this system, when converted to the decimal number system, becomes

  • a. 406
  • b. 1086
  • c. 213
  • d. 691
  • e.Not Attempted

10.Based on the figure below, what is the value of x, if y = 10? It is given that AD = y, AB = z, DC = x - 3, BC = x + 4. If AE is the perpendicular on BD, then AE = x - 3.

  • a. 10
  • b. 11
  • c. 12
  • d. None of these
  • e.Not Attempted

Directions for questions 11 to 12:
The petrol consumption rate of a new car 'Palto' depends on its speed and may be described by the graph below

11. Manisha makes the 200 km trip from Mumbai in Pune at a steady speed of 60 km per hour. What is the amount of petrol consumed for the journey?

  • a. 12.5 litres
  • b. 13.33 litres
  • c. 16 litres
  • d. 19.75 litres
  • e.Not Attempted

12. Manisha would like to minimise the fuel consumption for the trip by driving at the appropriate speed. How should she change the speed?

  • a. Increase the speed
  • b. Decrease the Speed
  • c. Maintain the speed at 60 km/hour
  • d. Cannot be determined
  • e.Not Attempted

DIRECTIONS for questions 13 and 14:
The batting average (BA) of a test batsman is computed from runs scored and innings played-completed innings and incomplete innings (not out) in the following manner:
r1 = number of runs scored in completed innings
n1 = number of completed innings
r2 = number of runs scored in incomplete innings
n2= number of incomplete innings
BA = (r1 + r2)/ n1
To better assess a batsman's accomplishments, the ICC is considering two other measures MBA1 and MBA2 defined as follows:
MBA1 = r1 / n1 + n2 / n1 + max [0, (r2 / n2 - r1 / n1)]
MBA2 = (r1 + r2 )/ (n1 + n2 )

13. Based on the information provided which of the following is

  • a. MBA1 ≤ BA ≤ MBA2
  • b. BA ≤ MBA2 ≤ MBA1
  • c. MBA2 ≤ BA ≤ MBA1
  • d. None of these
  • e.Not Attempted

14. An experienced cricketer with no incomplete innings has a BA of 50. The next time he bats, the innings is incomplete and he scores 45 runs. In can be inferred that

  • a. BA and MBA1 will both increase
  • b. BA well increase and MBA2 will decrease
  • c. BA will increase and not enough data is available to assess change in MBA1 and MBA2
  • d. None of these
  • e.Not Attempted

DIRECTIONS for questions 15 to 50:
Answer the questions independent of each other. Each question has a set of four statements. Each statement has three segments. Choose the alternative where the third segment

15.Raju has 128 boxes with him. He has to put at least 120 oranges in one box and 144 at the most. Then the least number of boxes which will have the same number of oranges is:

  • a. 5
  • b. 103
  • c. 3
  • d. 6
  • e. Not Attempted

16. Every ten years the Indian government counts all the people living in the country. Suppose that the director of the census has reported the following data on two neighbouring villages Chota hazri and Mota hazri
Chota hazri has 4,522 fewer males than Mota hazri
Mota hazri has 4,020 more females than males.
Chota hazri has twice as many females as males.
Chota hazri has 2,910 fewer females than Mota hazri.
What is the total number of males in Chota hazri?

  • a. 11264
  • b. 14174
  • c. 5632
  • d. 10154
  • e.Not Attempted

17. If x > 5 and y < -1, then which of the following statements is true?

  • a.(x + 4y) > 1
  • b. x > - 4y
  • c. - 4x < 5y
  • d. None of these
  • e.Not Attempted

18. The figure below shows the network connecting cities A, B, C, D, E and F. The arrows indicate permissible direction of travel. What is the number of distinct paths from A to F?

  • a. 9
  • b. 10
  • c. 11
  • d. None of these
  • e.Not Attempted

19.Three runners A, B and C run a race, with runner A finishing 12 meters ahead of runner B and 18 meters ahead of runner C, while runner B finishes 8 meters ahead of runner C. Each runner travels the entire distance at a constant speed. What was the length of the race?

  • a.36 meters
  • b. 48 meters
  • c. 60 meters
  • d. 72 meters
  • e.Not Attempted

20. Consider a triangle. Its longest side has length 20 and another of its sides has length 10. Its area is 80. What is the exact length of its third side?

  • a. √260
  • b. √250
  • c. √240
  • d. √270
  • e.Not Attempted

21. A train X departs from station A at 11.00 a.m. for station B, which is 180 km away. Another train Y departs from station B at the same time. Train X travels at an average speed of 70 km/hr and does not stop anywhere until it arrives at station B. Train Y travels at an average speed of 50 km/hr, but has to stop for 15 minutes at station C, which is 60 km away from station B enroute to station A. At what distance from A would they meet?

  • a. 112
  • b. 118
  • c. 120
  • d. None of these
  • e.Not Attempted

22. Three friends, returning from a movie, stopped to eat at a restaurant. After dinner, they paid their bill and noticed mints at the front counter. Sita took 1/3 of the mints, but returned four because she had a momentary pang of guilt. Fatima then took 1/4 of what was left but returned three for similar reasons. Eswari then took half of the reminder but threw two back into the bowl. The bowl had only 17 mints left when the raid was over. How many mints were originally in the bowl?

  • a. 38
  • b. 31
  • c. 41
  • d. None of these
  • e.Not Attempted

23. Shyam and Vyom walk up an escalator (moving stairway). The escalator moves at a constant speed. Shyam takes three steps for every two of Vyom's steps. Shyam gets to the top of the escalator after having taken 25 steps, while Vyom (because her slower pace lets the escalator do a little more of the work) takes only 20 steps to reach the top. If the escalator were turned off, how many steps would they have to take to walk up?

  • a. 40
  • b. 50
  • c. 60
  • d. 80
  • e.Not Attempted

24.If a, b, c and d are four positive real numbers such that abcd = 1, what is the minimum value of (l+a) (l+b) (l+c) (l+d)

  • a. 4
  • b. 1
  • c. 16
  • d. 18
  • e.Not Attempted

25. Anita had to do a multiplication. Instead of taking 35 as one of the multipliers, she took 53. As a result, the product went up by 540. What is the new product?

  • a. 1050
  • b. 540
  • c. 1440
  • d. 1590
  • e.Not Attempted

26. The owner of an art shop conducts his business in the following manner: Every once in a while he raises his prices by X%, then a while later he reduces all the new prices by X%. After one such up-down cycle, the price of a painting decreased by Rs 441. After a second up-down cycle the painting was sold for Rs 1944.81. What was the original price of the painting?

  • a. 2756.25
  • b. 2256.25
  • c. 2500
  • d. 2000
  • e.Not Attempted

27. A set of consecutive positive integers beginning with 1 is written on the blackboard. A student came along and erased one number. The average of the remaining numbers is 35 7/17. What was the number erased?

  • a. 7
  • b. 8
  • c. 9
  • d. None of these
  • e.Not Attempted

28. Let n be the number of different 5 digit numbers, divisible by 4 that can be formed with the digits 1,2,3,4,5 and 6, with no digit being repeated What is the value of n?

  • a. 144
  • b. 168
  • c. 192
  • d. None of the above
  • e.Not Attempted

29. Three math classes: X, Y, and Z, take an algebra test.
The average score in class X is 83.
The average score in class Y is 76
The average score in class Z is 85.
The average score of all students in classes X and Y together is 79.
The average score of all students in classes Y and Z together is 81.
What is the average for all the three classes?

  • a. 81
  • b. 81.5
  • c. 82
  • d. 84.5
  • e.Not Attempted

30. In the diagram, ABCD is a rectangle with AE= EF = FB. What is the ratio of the area of the triangle CEF and that of the rectangle?

  • a. 1/6
  • b. 1/8
  • c. 1/9
  • d. None of these
  • e.Not Attempted

31. At a certain fast food restaurant, Bakshi can buy 3 burgers, 7 shakes, and one order of fries for Rs.120. At the same place it would cost Rs.164.50 for 4 burgers, 10 shakes, and one order of fries. How much would it cost for a meal of one burger, one shake, and one order of fries?

  • a. Rs 31
  • b. Rs 41
  • c. Rs 21
  • d. Cannot be determined.
  • e.Not Attempted

32. A can complete a piece of work in 4 days. B takes double the time taken by A. C takes double that of B, and D takes double that of C to complete the same task. They are paired in groups of two each. One pair takes two- thirds the time needed by the second pair to complete the work. Which is the first pair?

  • a. A, B
  • b. A, C
  • c. B, C
  • d. A, D
  • e.Not Attempted

33. In a 4- digit number, the sum of the first two digits is equal to that of the last two digits. The sum of the first and last digits is equal to the third digit. Finally, the sum of the second and fourth digits is twice the sum of the other two digits. What is the third digit of the number?

  • a. 5
  • b. 8
  • c. 1
  • d. 4
  • e.Not Attempted

34. A college has raised 75% of the amount it needs for a new building by receiving an average donation of Rs. 600 from the people already solicited. The people already solicited represent 60% of the total people the college will ask for donations. If the college is to raise exactly the amount needed for the new building, what should be the average donation from the remaining people to be solicited?

  • a. Rs 300
  • b. Rs 250
  • c. Rs 400
  • d. Rs.500
  • e.Not Attempted

35. There's a lot of work in preparing a birthday dinner. Even after the turkey is in the oven, there's still the potatoes and gravy, yams, salad, and cranberries, not to mention setting the table.
Three friends, Asit, Arnold, and Afzal work together to get all of these chores done. The time it takes them to do the work together is six hours less than Asit would have taken working alone, one hour less than Arnold would have taken alone, and half the time Afzal would have taken working alone.
How long did it take them to do these chores working together?

  • a. 20 minutes
  • b. 30 minutes r
  • c. 40 minutes
  • d. 50 minutes
  • e. Not Attempted

36. A red light flashes 3 times per minute and a green light flashes 5 times in two minutes at regular intervals. If both lights start flashing at the same time, how many times do they flash together in each hour?

  • a. 30
  • b. 24
  • c. 20
  • d. 60
  • e.Not Attempted

37. Two sides of a plot measure 32 meters and 24 meters and the angle between them is a right angle. The other two sides measure 25 meters each and the other three angles are not right angles.
What is the area of the plot?

  • a. 768
  • b. 534
  • c. 696.5
  • d. 684
  • e.Not Attempted

38. Ashish is given Rs.158 in one rupee denominations. He has been asked to allocate them into a number of bags such that any amount required between Re.1 and Rs.158 can be given by handing out a certain number of bags without opening them. What is the minimum number of bags required?

  • a. 11
  • b. 12
  • c. 13
  • d. None of these
  • e.Not Attempted

39. In the given figure BC = AC, angle AFD = 400 and CE = CD. The value of angle BCE = ?

  • a. 140
  • b. 70
  • c. 100
  • d. None of these
  • e.Not Attempted

40. For a Fibonacci sequence, from the third term onwards, each term in the sequence is the sum of the previous two terms in that sequence. If the difference of squares of seventh and sixth terms of this sequence is 517, what is the tenth term of this sequence?

  • a. 147
  • b. 76
  • c. 123
  • d. Cannot be determined
  • e.Not Attempted

41. In some code, letters a, b, c, d and e represents numbers 2, 4, 5, 6 and 10. We don't know which letter represents which number. Consider the following relationships:
i) a + c = e ii) b - d = d and iii) e + a =b
Which statement below is true?

  • a. b = 4, d = 2
  • b. a = 4, e = 6
  • c. b = 6, e = 2
  • d. a = 4, c = 6
  • e.Not Attempted

42. At his usual rowing rate, Rohit can travel 12 miles downstream in a certain river in six hours less than it takes him to travel the same distance upstream. But if he could double his usual rowing rate for this 24 mile round trip, the downstream 12 miles would then take only one hour less than the upstream 12 miles. What is the speed of the current in miles per hour?

  • a. 7/3
  • b. 4/3
  • c. 5/3
  • d. 8/3
  • e.Not Attempted

43. Two men X and Y started working for a certain company at similar jobs on January 1,1950. X asked for an initial salary of Rs.300 with an annual increment of Rs.30. Y asked for an initial salary of Rs.200 with a rise of Rs.15 every six months. Assume that the arrangements remained unaltered till December 31,1959. Salary is paid on the last day of the month. What is the total amount paid to them as salary during the period?

  • a. Rs. 93,300
  • b. Rs.93,200
  • c. Rs.93,100
  • d. None of these
  • e.Not Attempted

44. m is the smallest positive integer such that n > m. also it is known that n3 - 7n2 + 11n - 5 is positive. Then the possible value for m is:

  • a. 5
  • b. 8
  • c. 4
  • d. None of these
  • e.Not Attempted

45. A rectangular pool 20 meters wide and 60 metres long is surrounded by a walkway of uniform width. If the total area of the walkway is 516 square meters, how wide, in metres, is the walkway?

  • a. 43
  • b. 4.3
  • c. 3
  • d. 3.5
  • e.Not Attempted

46. December 9, 2001 is Sunday. What was the day on December 9, 1971?

  • a. Thursday
  • b. Wednesday
  • c. Saturday
  • d. Sunday
  • e.Not Attempted

47. Let b be a positive integer and a = b2 - b. If b ≤ 4, then a2 - 2a is divisible by

  • a. 15
  • b. 20
  • c. 24
  • d. None of these
  • e.Not Attempted

48.Fresh grapes contain 90% water by weight while dried grapes contain 20% water by weight. What is the weight of dry grapes available from 20 kg of fresh grapes?

  • a. 2 Kg
  • b. 2.4 Kg
  • c. 2.5 Kg
  • d. None of these
  • e.Not Attempted

49. A change making machine contains 1 rupee, 2 rupee and 5 rupee coins. The total number of coins is 300. The amount is Rs.960. If the number of 1 rupee coins and the number of 2 rupee coins are interchanged, the value comes down by Rs.40. The total number of 5 rupee coins is:

  • a. 100
  • b. 140
  • c. 60
  • d. 150
  • e.Not Attempted

50. Let x, y be two positive numbers such that x + y = 1. Then, the minimum value of (x+1/x)2 +(y+1/y)2 is

  • a. A
  • b. B
  • c. C
  • d. D
  • e.Not Attempted

CAT2001 SECTION - ENGLISH

Instructions
1. The test comprises of 55 questions. You should complete the test within 40 minutes.
2. There is only one correct answer to each question.
3. All questions carry four marks each.
4. Each wrong answer will attract a penalty of one mark.

DIRECTIONS for questions 1 to 30:
Each of the six passages given below is followed by questions. Choose the best answer for each question.

PASSAGE -1

The narrative of Dersu Uzala is divided into two major sections, set in 1902 and 1907, that deal with separate expeditions which Areseniev conducts into the Ussuri region. In addition, a third time frame forms a prologue to the film. Each of the temporal frames has a difference focus, and by shifting them Kurosawa is able to describe the encroachment of settlements upon the wilderness and the consequent erosion of Dersu's way of life. As the film opens, that erosion has already begun. The first image is a long shot of a huge forest, the trees piled upon one another by the effects of the telephoto lens so that the landscape becomes an abstraction and appears like a huge curtain of green. A title informs us that the year is 1910. This is as close into the century as Kurosawa will go. After this prologue, the events of the film will transpire even farther back in time and will be presented as Arseniev's recollections.

The character of Dersu Uzala is the heart of the film, his life the example that Kurosawa wishes to affirm. Yet the formal organisation of the film works to contain, to close, to circumscribe that life by erecting a series of obstacles around it. The file itself is circular, opening and closing by Dersu's grave, thus sealing off the character from the modern world to which Kurosawa once so desperately wanted to speak. The multiple time frames also work to maintain a separation between Dersu and the contemporary world. We must go back farther even than 1910 to discover who he was. But this narrative structure has yet another implication. It safeguards Dersu's example, inoculates it from contamination with history, and protects it from contact with the industrialised, urban world. Time is organised by the narrative into a series of barriers, which enclose Dersu in a kind of vacuum chamber, protecting him from the social and historical dialectics that destroyed the other Kurosawa heroes. Within the film, Dersu does die, but the narrative structure attempts to immortalise him and his example, as Dersu passes from history into myth.

We see all this at work in the enormously evocative prologue. The camera tilts down to reveal felled trees littering the landscape and an abundance of construction. Roads and houses outline the settlement that is being built; Kurosawa cuts to a medium shot of Arseniev standing in the midst of the clearing, looking uncomfortable and disoriented. A man passing in a wagon asks him what he is doing, and the explorer says he is looking for a grave. The driver replies that no one has died here, the settlement is too recent. These words enunciate the temporal rupture that the film studies. It is the beginning of things (industrial society) and the end of things (the forest), the commencement of one world so young that no one has had time yet to die and the eclipse of another, in which Dersu has died. It is his grave for which the explorer searches. His passing symbolises the new order, the development that now surrounds Arseniev. The explorer says he buried his friend three years ago, next to huge cedar and fir trees, but now they are all gone. The man on the wagon replies they were probable chopped down when the settlement was built, and he drives off.

Arseniev walks to a barren, treeless spot next to a pile of bricks. As he moves, the camera tracks and pans to follow, revealing a line of freshly built houses and a woman hanging her laundry to dry. A distant train whistle is heard, and the sounds of construction in the clearing vie with the cries of birds and the rustle of wind in the trees. Arseniev pauses, looks around for the grave that once was, and murmurs desolately, "Dersu". The image now cuts farther into the past, to 1902, and the first section of the film commences, which describes Arseniev's meeting with Dersu and their friendship. Kurosawa defines the world of the film initially upon a void, a missing presence. The grave is gone, brushed aside by a world rushing into modernism, and now the hunter exists only in Arseniev's memories. The hallucinatory dreams and visions of Dodeskaden are succeeded by nostalgic, melancholy ruminations. Yet by exploring these ruminations, the film celebrates the timelessness of Dersu's wisdom.

The first section of the film has two purposes: to describe the magnificence and inhuman vastness of nature and to delineate the code of ethics by which Dersu lives and which permits him to survive in these conditions. When Dersu first appears, the other soldiers treat him with condescension and laughter, but Arseniev watches him closely and does not share their derisive response. Unlike them, he is capable of immediately grasping Dersu's extraordinary qualities. In camp, Kurosawa frames Arseniev by himself, sitting on the other side of the fire from his soldiers. While they sleep or joke among themselves, he writes in his diary and Kurosawa cuts in several point-of-view shots from his perspective of trees that appear animated and sinister as the fire light dances across their gnarled, leafless outlines. This reflective dimension, this sensitivity to the spirituality of nature, distinguishes him from the others and forms the basis of his receptivity to Dersu and their friendship. It makes him a fit pupil for the hunter.

1. According to the author the section of the film following the prologue:

  • a. serves to highlight the difficulties that Dersu faces that eventually kills him.
  • b. shows the difference in thinking between Arseniev and Dersu.
  • c. shows the code by which Dersu lives that allows him to survive his surroundings.
  • d. serves to criticise the lack of understanding of nature in the pre-modern era.
  • e.Not Attempted

2. Arseniev's search for Dersu's grave:

  • a. is part of the beginning of the film
  • b. symbolises the end of the industrial society.
  • c. is misguided since the settlement is too new
  • d. symbolises the rediscovery of modernity.
  • e.Not Attempted

3. In the film, Kurosawa hints at Arseniev's reflective and sensitive nature:

  • a. by showing him as not being derisive towards Dersu, unlike other soldiers.
  • b. by showing him as being aloof from other soldiers.
  • c. through shots of Arseniev writing his diary, framed by trees.
  • d. all of the above
  • e.Not Attempted

4. The film celebrates Dersu's wisdom

  • a. by exhibiting the moral vacuum of the pre-modern world.
  • b. by turning him into a mythical figure.
  • c. through hallucinatory dreams and visions.
  • d. through Arseniev's nostalgic, melancholy ruminations
  • e.Not Attempted

5. How is Kurosawa able to show the erosion of Dersu’s way of life?

  • a. by documenting the ebb and flow of modernisation.
  • b. by going back farther and farther in time
  • c. by using three different time frames and shifting them.
  • d. through his death in a distant time.
  • e.Not Attempted

6. According to the author, which of these statements about the film are correct?

  • a. The film makes its arguments circuitously.
  • b. The film highlights the insularity of Arseniev.
  • c.The film begins with the absence of its main protagonist.
  • d. None of the above.
  • e.Not Attempted

PASSAGE -II

Billie Holiday died a few weeks ago. I have been unable until now to write about her, but since she will survive many who receive longer obituaries, a short delay in one small appreciation will not harm her or us. When she died we -- the musicians, critics, all who were ever transfixed by the most heart-rending voice of the past generation -- grieved bitterly. There was no reason to. Few people pursued self-destruction more whole-heartedly, and when the pursuit was at an end, at the age of forty-four, she had turned herself into a physical and artistic wreck. Some of us tried gallantly to pretend otherwise, taking comfort in the occasional moments when she still sounded like a ravaged echo of her greatness. Other had not even the heart to see and listen any more. We preferred to stay home and, if old and lucky enough to own the incomparable records of her heyday from 1937 to 1946, many of which are not even available on British LP to recreate those coarse-textured, sinuous, sensual and unbearable sad noises which gave her a sure corner of immortality. Her physical death called, if anything, for relief rather than sorrow. What sort of middle age would she have faced without the voice to earn money for her drinks and fixes, without the looks -- and in her day she was hauntingly beautiful -- to attract the men she needed, without business sense, without anything but the disinterested worship of ageing men who had heard and seen her in her glory?

And yet, irrational though it is, our grief expressed Billie Holiday's art, that of a woman for whom one must be sorry. The great blues singers, to whom she may be justly compared, played their game from strength. Lionesses, though often wounded (did not Bessie Smith call herself "a tiger, ready to jump"?), their tragic equivalents were Cleopatra and Phaedra; Holiday's was an embittered Ophelia. She was the Puccini heroine among blues singers, or rather among jazz singers, for though she sang a cabaret version of the blues incomparably, her natural idiom was the pop song. Her unique achievement was to have twisted this into a genuine expression of the major passions by means of a total disregard of its sugary tunes, or indeed of any tune other than her own few delicately crying elongated notes, phrased like Bessie Smith or Louis Armstrong in sackcloth, sung in a thin, gritty, haunting voice whose natural mood was an unresigned and voluptuous welcome for the pains of love. Nobody has sung, or will will sing Bessie's songs as she did

Little need be said about her horrifying life, which she described with emotional, though hardly with factual, truth in her autobiography Lady Sings the Blues. After an adolescence in which self-respect was measured by a girl's insistence in picking up the coins thrown to her by clients with her hands, she was plainly beyond help. She did not lack it, for she had the flair and scrupulous honesty of John Hammond to launch her, the best musicians of the 1930s to accompany her -- notably Teddy Wilson, Frankie Newton and Lester Young -- the boundless devotion of all serious connoisseurs, and much public success. It was too late to arrest a career of systematic embittered self-immolation. To be born with both beauty and self-respect in the Negro ghetto of Baltimore in1915 was too much of a handicap, even without rape at the age of ten and drug-addiction in her teens. But while she destroyed herself, she sang, unmelodious, profound and heartbreaking. It is impossible not to weep for her, or not to hate the world which made her what she was.

7. According to the passage, Billie Holiday was fortunate in all but one of the following ways:

  • a. she was fortunate to have been picked up young by an honest producer.
  • b. she was fortunate to have the likes of Louis Armstrong and Bessie Smith accompany her.
  • c. she was fortunate to possess the looks.
  • d. she enjoyed success among the public and connoisseurs.
  • e.Not Attempted

8.According to the author, if Billie Holiday had not died in her middle age:

  • a. she would have gone on to make a further mark.
  • b. she would have become even richer than what she was when she died
  • c. she would have led a rather ravaged existence.
  • d. she would have led a rather comfortable existence.
  • e.Not Attempted

9. Why will Billie holiday survive many who receive longer obituaries?

  • a. because of her blues creations.
  • b. because she was not as self-destructive as some other blues exponents.
  • c. because of her smooth and mellow voice.
  • d. because of the expression of anger in her songs.
  • e.Not Attempted

10. Which of the following statements is not representative of the author’s opinion?
Which one of the following best bolsters the conclusion that reducing duties will expand the tax base'?

  • a.Billie Holiday had her unique brand of melody.
  • b. Billie Holiday’s voice can be compared to other singers’ in certain ways.
  • c. Billie Holiday’s voice had a ring of profound sorrow.
  • d. Billie Holiday welcomed suffering in her profession and in her life.
  • e.Not Attempted

PASSAGE -III

The union government's position vis-A-vis the United Nations conference on racial and related discrimination world-wide seems to be the following: discuss race please, not caste; caste is our very own and not at all as bad as you think. The gross hypocrisy of that position has been lucidly underscored by Kancha Ilaiah. Explicitly, the world community is to be cheated out of considering the matter on the technicality that caste is not, as a concept, tantamount to a racial category. Internally, however, allowing the issue to be put on agenda at the said conference would, we are particularly admonished, damage the country's image. Somehow, India's spiritual beliefs elbow out concrete actualities. Inverted representations, as we know, have often been deployed in human histories as balm for the forsaken -- religion being most persistent of such inversions. Yet, we would humbly submit that if globalising our markets are thought good for the "national" pocket, globalising our social inequities might not be so bad for the mass of our people. After all, racism was as uniquely institutionalised in South Africa as caste discrimination has been within our society; why then can't we permit the world community to express itself on the latter with a fraction of the zeal with which, through the years, we pronounced on the former? As to the technicality about whether or not caste is admissible into an agenda about race (that the conference is also about a related discriminations' tends to be forgotten), a reputed sociologist has recently argued that where race is a "biological" category caste is a "social" one.

Having earlier fiercely opposed implementation of the Mandal Commission Report, the said sociologist is at least to be complemented now for admitting, however tangentially, that caste discrimination is a reality, although in his view, incompatible with racial discrimination. One would like quickly to offer the hypothesis that biology, in important ways that affect the lives of many millions, is in itself perhaps a social construction. But let us look at the matter in another way. If it is agreed -- as per the position today at which anthropological and allied scientific determinations rest -- that the entire race of homo sapiens derived from an originally black African female (called "Eve") then one is hard put to understand how, on some subsequent ground, ontological distinctions are to be drawn either between races or castes. Let us also underline the distinction between the supposition that we are all God's children and the rather more substantiated argument about our descent from "Eve", lest both positions are thought to be equally diversionary. It then stands to reason that all subsequent distinctions are, in modern parlance, "constructed" ones, and, like all ideological constructions, attributable to changing equations between knowledge and power among human communities through contested histories here, there, and elsewhere.

This line of thought receives, thankfully, extremely consequential buttress from the findings of the Human Genome project. Contrary to earlier (chiefly 19th Century colonial) persuasions on the subject of race, as well as, one might add, the somewhat infamous Jensen offering in the 20th century from America, those findings deny genetic difference between races. If anything, they suggest that environmental factors impinge on gene-function, as a dialectic seems to unfold between nature and culture. It would thus seem that biology as the constitution of pigmentation enters the picture first only as a part of that dialectic. Taken together, the original mother stipulation and the Genome findings ought indeed to furnish ground for human equality across the board, as well as yield policy initiatives towards equitable material dispensations aimed at building a global order where, in Hegel's stirring formulation, only the rational constitutes the right. Such, sadly, is not the case as everyday fresh arbitrary grounds for discrimination are constructed in the interests of sectional dominance

11.According to the author, "inverted representations as balm for the forsaken":

  • a. is good for the forsaken and often deployed in human histories.
  • b. is good for the forsaken, but not often deployed historically for the oppressed.
  • c. occurs often as a means of keeping people oppressed.
  • d. occurs often to invert the status quo.
  • e.Not Attempted

12. When the author writes "globalising our social inequities", the reference is to:

  • a. going beyond an internal deliberation on social inequity.
  • b. dealing with internal poverty through the economic benefits of globalisation.
  • c. going beyond an internal delimitation of social inequity.
  • d. achieving disadvantaged people’s empowerment, globally.
  • e.Not Attempted

13. According to the author, the sociologist who argued that race is a "biological" category and caste is a "social" one:

  • a. generally shares the same orientation as the author's on many of the central issues discussed.
  • b. tangentially admits to the existence of "caste" as a category.
  • c. admits the incompatibility between the people of different race and caste.
  • d. admits indirectly that both caste-based prejudice and racial discrimination exist.
  • e.Not Attempted

14. An important message in the passage, if one accepts a dialectic between nature and culture, is that:

  • a. the result of the Human Genome Project reinforces racial differences.
  • b. race is at least partially a social construct.
  • c. discrimination is at least partially a social construct.
  • d. caste is at least partially a social construct.
  • e.Not Attempted

15. Based on the passage, which of the following unambiguously fall under the purview of the UN conference being discussed? A.Racial prejudice.
B. Racial Pride.
C. Discrimination, racial or otherwise.
D. Caste- related discrimination.
E. Race related discrimination

  • a. A, E
  • b. C, E
  • c. A, C, E
  • d. B, C, D
  • e. Not Attempted

PASSAGE -IV

In modern scientific story, light was created not once but twice. The first time was in the Big Bang, when the universe began its existence as a glowing, expanding, fireball, which cooled off into darkness after a few million years. The second time was hundreds of millions of year later, when the cold material condensed into dense nuggets under the influence of gravity, and ignited to become the first stars.

Sir Martin Rees, Britain's astronomer royal, named the long interval between these two enlightenments the cosmic "Dark Age". The name describes not only the poorly lit conditions, but also the ignorance of astronomers about that period. Nobody knows exactly when the first stars formed, or how they organised themselves into galaxies or even whether stars were the first luminous objects. They may have been preceded by quasars, which are mysterious, bright spots found at the centres of some galaxies. Now, two independent groups of astronomers, one led by Robert Becker of the University of California, and the other by George Djorgovski of Caltech, claim to have peered far enough into space with their telescopes (and therefore backwards enough in time) to observe the closing days of the Dark Age.

The main problem that plagued previous efforts to study the Dark Age was not the lack of suitable telescopes, but rather the lack of suitable things at which to point them. Because these events took place over 13 billion years ago, if astronomers are to have any hope of unravelling them they must study objects that are at least 13 billion light years away. The best prospects are quasars, because they are so bright and compact that they can be seen across vast stretches of space. The energy source that powers a quasar is unknown, although it is suspected to be the intense gravity of a giant black hole. However, at the distances required for the study of Dark Age, even quasars are extremely rare and faint.

Recently some members of Dr. Becker's team announced their discovery of the four most distant quasars known. All the new quasars are terribly faint, a challenge that both teams overcame by peering at them through one of the twin telescopes in Hawaii. These are the world's largest, and can therefore collect the most light. The new work by Dr. Becker's team analysed the light from all four quasars. Three of them appeared to be similar to ordinary, less distant quasars. However, the fourth and most distant, unlike any other quasar ever seen, showed unmistakable signs of being shrouded in a fog of hydrogen gas. This gas is leftover material from the Big Bang that did not condense into stars or quasars. It acts like fog because new-born stars and quasars emit mainly ultraviolet light, and hydrogen gas is opaque to ultraviolet. Seeing this fog had been the goal of would-be Dark Age astronomers since 1965, when James Gunn and Bruce Peterson spelled out the technique for causing quasars as backlighting beacons to observe the fog's ultraviolet shadow.

The fog prolonged the period of darkness until the heat from the first stars and quasars had the chance to ionise the hydrogen (breaking it into its constituent parts, protons and electrons). Ionised hydrogen is transparent to ultraviolet radiation, so at that moment the fog lifted and the universe became the well-lit place it is today. For this reason, the end of the Dark Age is called the "Epoch of Re-ionisation", because the ultraviolet shadow is visible only in the most distant of the four quasars. Dr. Becker's team concluded that the fog had dissipated completely by the time the universe was about 900 million years old, and one-seventh of its current size.

16. In the passage, the Dark Age refers to:

  • a. the period when the universe became cold after the Big Bang.
  • b. a period about which astronomers know very little.
  • c. the medieval period when cultural activity seemed to have come to an end.
  • d. the time that the universe took to heat up after the Big-Bang.
  • e.Not Attempted

17. Astronomers find it difficult to study the Dark Age because:

  • a. suitable telescopes are few.
  • b. the associated events took place aeons ago.
  • c. the energy source that powers a quasar is unknown.
  • d. their best chance is to study quasars, which are faint objects to begin with.
  • e.Not Attempted

18. The four most distant quasars discovered recently:

  • a. could only be seen with quasars discovered recently.
  • b. appear to be similar to other ordinary, quasars.
  • c. appear to be shrouded in a fog of hydrogen gas.
  • d. have been sought to be discovered by Dark Age astronomers since 1965.
  • e.Not Attempted

19.The fog of hydrogen gas seen through the telescopes:

  • a. is transparent to hydrogen radiation from stars and quasars in all states.
  • b. was lifted after heat from stars and quasars ionised it.
  • c. is material which eventually became stars and quasars.
  • d. is broken into constituent elements when stars and quasars are formed.
  • e.Not Attempted

PASSAGE -V

Studies of the factors governing reading development in young children have achieved a remarkable degree of consensus over the past two decades. This consensus concerns the causal role of phonological skills in young children's reading progress. Children who have good phonological skills, or good "phonological awareness", become good readers and good spellers. Children with poor phonological skills progress more poorly. In particular, those who have a specific phonological deficit are likely to be classified as dyslexic by the time that they are 9 or 10 years old.

Phonological skills in young children can be measured at a number of different levels. The term phonological awareness is a global one, and refers to a deficit in recognising smaller units of sound within spoken words. Development work has shown that this deficit can be at the level of syllables. Of onsets and rimes, or of phonemes. For example, a 4-year old child might have difficulty in recognising that a word like valentine has three syllables, suggesting a lack of syllabic awareness. A-5 year old might have difficulty in recognising that the odd word out in the set of words fan, cat, hat, mat is fan. This task requires an awareness of the sub-syllable units of the onset and the rime. The onset corresponds to any initial consonants in a syllable, and the rime corresponds to the vowel and to any following consonants. Rimes correspond to single-syllable words, and so the rime in fan differs from the rime in cat, hat, and mat. In longer words, rime and rhyme may differ. The onsets in valentine are /v/ and /t/, and the rimes correspond to the spelling patterns 'al', 'en', and 'ine'.

A 6 year-old might have difficulty in recognising that plea and may begin with the same initial sound. This is a phonemic judgement. Although the initial phoneme /p/ is shared between the two words, in plea it is part of the onset 'pr' Until children can segment the onset (or the rime), such phonemic judgements are difficult for them to make. In fact, a recent survey of different developmental studies has shown that the different level of phonological awareness appears to emerge sequentially. The awareness of syllables, onsets, and rimes appears to emerge at around the ages of 3 and 4, long before most children go to school. The awareness of phonemes, on the other hand, usually emerges at around the age of 5 or 6, when children have been taught to read for about a year. An awareness of onsets and rimes thus appears to be a precursor of reading, whereas an awareness of phonemes at every serial position in a word only appears to develop as reading is taught. The onset-rime and phonemic levels of phonological structure, however, are not distinct. Many onsets in English are single phonemes, and so are some rimes (e.g. sea, go, zoo).

The early availability of onsets and rimes is supported by studies that have compared the development of phonological awareness of onsets, rimes, and phonemes in the same subjects using the same phonological awareness tasks. For example, a study by Treiman and Zudowski used a same/different judgement task based on the beginning or the end sounds of words. In the beginning sound task, the words either began with the same onset, as in plea and plank, or shared only the initial phoneme, as in plea and pray. In the end-sound task, the words either shared the entire rime, as in spit and wit, or shared only the final phoneme, as in rat and wit. Treiman and Zudowski showed that 4 and 5 year old children found the onset-rime version of the same/different task significantly easier than the version based on phonemes. Only the 6-year-olds, who had been learning to read for about a year, were able to perform both versions of the tasks with an equal level of success.

20. The single-syllable words Rhyme and Rime are constituted by the exact same set of: A. rime(s). B. Onset(s) C. Rhyme(s). D. Phonemes(s)

  • a. A, B
  • b. A, C
  • c. A, B, C
  • d. B, C, D
  • e.Not Attempted

21. The Treiman and Zudowski experiment found evidence to support the following:

  • a. at age 6, reading instruction helps children perform, both, the same-different judgement task.
  • b. the development of onset-rime awareness precedes the development of an awareness of phonemes.
  • c.at age 4-5 children find the onset-rime version of the same/different task significantly easier.
  • d. the development of onset-rime awareness is a necessary and sufficient condition for the development of an awareness of phonemes.
  • e.Not Attempted

22. A phonological deficit in which of the following is likely to be classified as dyslexia?

  • a. Phonemic judgement
  • b. Onset judgement
  • c. Rime judgement
  • d. Any one or more of the above
  • e.Not Attempted

23. From the following statements, pick out the true statement according to the passage.

  • a. A mono-syllabic word can have only one onset.
  • b. A mono-syllabic word can have only one rhyme but more than one rime.
  • c. A mono-syllabic word can have only one phoneme.
  • d.All of the above.
  • e.Not Attempted

24. Which one of the following is likely to emerge last in the cognitive development of a child?

  • a. Rhyme
  • b. Rime
  • c. Onset.
  • d. Phoneme.
  • e.Not Attempted

PASSAGE -VI

Democracy rests on a tension between two different principles. There is, on the one hand, the principle of equality before the law, or, more generally, of equality, and, on the other, what may be described as the leadership principle. The first gives priority to rules and the second to persons. No matter how skilfully we contrive our schemes, there is a point beyond which the one principle cannot be promoted without some sacrifice of the other.

Alexis de Tocqueville, the great nineteenth century writer on democracy, maintained that the age of democracy, whose birth he was witnessing, would also be the age of mediocrity: in saying this he was thinking primarily of a regime of equality governed by impersonal rules. Despite his strong attachment to democracy, he took great pains to point out what he believed to be its negative side: a dead level plane of achievement in practically every sphere of life. The age of democracy would, in his view, be an unheroic age; there would not be room in it for either heroes of hero-worshippers.

But modern democracies have not been able to do without heroes: this too was foreseen, with much misgiving, by Tocqueville. Tocqueville viewed this with misgiving because he believed, rightly or wrongly, that unlike in aristocratic societies there was no proper place in a democracy for heroes and, hence, when they arose they would sooner or later turn into despots. Whether they require heroes or not, democracies certainly require leaders, and, in the contemporary age, bred them in great profusion; the problems is to know what to do with them.

In a world preoccupied with scientific rationality the advantages of a system based on an impersonal rule of law should be a recommendation with everybody. There is something orderly and predictable about such a system. When life is lived mainly in small, self-contained communities, men are able to take finer personal distinctions into account in dealing with their fellow men. They are unable to do this in a large and amorphous society, and organised living would be impossible without a system of impersonal rules. Above all, such a system guarantees a kind of equality to the extent that everybody, no matter in what station of life, is bound by the same explicit, often written, rules, and nobody is above them.

But a system governed solely by impersonal rules can at best ensure order and stability; it cannot create any shining vision of a future in which mere formal equality will be replaced by real equality and fellowship. A world governed by impersonal rules cannot easily change itself, or when it does, the change is so gradual as to make the basic and fundamental feature of society appear unchanged. For any kind of basic or fundamental change, a push is needed from within, a kind of individual initiative which will create new rules, new terms and conditions of life.

The issue of leadership thus acquires crucial significance in the context of change. If the modern age is preoccupied with scientific rationality, it is no less preoccupied with change. To accept what exists on its own terms is traditional, not modern, and it may be all very well to appreciate tradition in music, dance and drama, but for society as a whole the choice has already been made in favour of modernisation and development. Moreover, in some countries the gap between ideal and reality has become so great that the argument for development and change is now irresistible.

In these countries no argument for development has greater appeal or urgency than the one which shows development to be the condition for the mitigation, if not the elimination, of inequality. There is something contradictory about the very present of large inequalities in a society which professes to be democratic. It does not take people too long to realise that democracy by itself can guarantee only formal equality; beyond this, it can only whet people's appetite for real or substantive equality. From this arises their continued preoccupation with plans and schemes that will help to bridge the gap between the ideal of equality and the reality which is so contrary to it. When pre-existing rules give no clear directions of change, leadership comes into its own. Every democracy invests its leadership with a measure of charisma, and expects from it a corresponding measure of energy and vitality. Now, the greater the urge for change in a society the stronger the appeal of a dynamic leadership in it. A dynamic leadership seeks to free itself from the constraints of existing rules; in a sense that is the test of its dynamism. In this process it may take a turn at which it ceases to regard itself as being bound by these rules, placing itself above them. There is always a tension between 'charisma' and 'discipline' in the case of a democratic leadership, and when this leadership puts forward revolutionary claims, the tension tends to be resolved at the expense of discipline. Characteristically, the legitimacy of such a leadership rests on its claim to be able to abolish or at least substantially reduce the existing inequalities in society. From the argument that formal equality or equality before the law is but a limited good, it is often one short step to the argument that it is a hindrance or an obstacle to the establishment of real or substantive equality. The conflict between a "progressive" executive and a "conservative" judiciary is but one aspect of this larger problem. This conflict naturally acquires added piquancy when the executive is elected and the judiciary appointed. 25. Which of the following four statements can be inferred from the above passage?
A. There is conflict between the pursuit of equality and individuality.
B. The disadvantages of impersonal rules can be overcome in small communities.
C. Despite limitations, impersonal rules are essential in large systems.
D. Inspired leadership, rather than plans and schemes, is more effective in bridging inequality.

  • a. B, D but not A, C
  • b. A, D but not C, B
  • c. A, B, but not C, D
  • d. A, C but not B, D
  • e.Not Attempted

26. What possible factor would a dynamic leader consider a "hindrance" in achieving the development goals of a nation?

  • a. Principle of equality before the law.
  • b. Judicial activism.
  • c. A conservative judiciary.
  • d. Need for discipline.
  • e.Not Attempted

27. Which of the following four statements can be inferred from the above passage?
A. Scientific rationality is an essential feature of modernity.
B. Scientific rationality results in the development of impersonal rules.
C. Modernisation and development have been chosen over traditional music, dance and drama.
D. Democracies aspire to achieve substantive equality

  • a. A, B, D but not C
  • b. A, B but not C, D
  • c. A, D but not B, C.
  • d. A, B C but not D
  • e.Not Attempted

28. A key argument the author is making is that:

  • a. in the context of extreme inequality, the issue of leadership has limited significance.
  • b. democracy is incapable of eradicating inequality.
  • c. formal equality facilitates development and change.
  • d. impersonal rules are good for avoiding instability but fall short of achieving real equality.
  • e.Not Attempted

29. Tocqueville believed that the age of democracy would be an un-heroic age because:

  • a. democratic principles do not encourage heroes.
  • b. there is no urgency for development in democratic countries.
  • c. heroes that emerged in democracies would become despots.
  • d. aristocratic society had a greater ability to produce heroes.
  • e.Not Attempted

30. Dynamic leaders are needed in democracies because:

  • a. they have adopted the principles of "formal" equality rather than "substantive" equality.
  • b. formal 'equality whets people's appetite for' substantive' equality.
  • c. systems that rely on the impersonal rules of 'formal' equality loose their ability to make large changes
  • d. of the conflict between a 'progressive' executive and a 'conservative' judiciary.
  • e.Not Attempted

DIRECTIONS for question 31 to 35:
The sentence given in each question, when properly sequenced, form a coherent paragraph. Each sentence is labelled with a letter. Choose the most logical order of sentences from among the given choices to construct a coherent paragraph.

31. A. Passivity is not, of course, universal. B. In areas where there are no lords or laws, or in frontier zones where all men go armed, the attitude of the peasantry may well be different. C. So indeed it may be on the fringe of the unsubmissive. D. However, for most of the soil-bound peasants the problem is not whether to be normally passive or active, but when to pass from one state to another. E. This depends on an assessment of the political situation.

  • a. BEDAC
  • b. CDABE
  • c. EDBAC
  • d. ABCDE
  • e.Not Attempted

32. A. But in the industrial era destroying the enemy's productive capacity means bombing the factories which are located in the cities. B. So in the agrarian era, if you need to destroy the enemy's productive capacity, what you want to do is burn his fields, or if you're really vicious, salt them. C. Now in the information era, destroying the enemy's productive capacity means destroying the information infrastructure. D. How do you do battle with your enemy? E. The idea is to destroy the enemy's productive capacity, and depending upon the economic foundation, that productive capacity is different in each case. F. With regard to defence, the purpose of the military is to defend the nation and be prepared to do battle With its enemy.

  • a. FDEBAC
  • b. FCABED
  • c. DEBACF
  • d. DFEBAC
  • e.Not Attempted

33.A. Michael Hofman, a poet and translator, accepts this sorry fact without approval or complaint. B. But thanklessness and impossibility do not daunt him. C. He acknowledges too -- in fact he returns to the point often -- that best translators of poetry always fail at Home level. D. Hofman feels passionately about his work, and this is clear from his writings E. In terms of the gap between worth and reward, translators come somewhere near nurses and street - cleaners.

  • a. EACDB
  • b. ADEBC
  • c. EACBD
  • d. DCEAB
  • e.Not Attempted

34. A. Although there are large regional variations, it is not infrequent to find a large number of people sitting here together and doing nothing. B. Once in office, they receive friends and relatives who feel free to call any time without prior appointment. C. While working, one is struck by the slow and clumsy actions and reactions, indifferent attitudes. Procedure rather than outcome orientation, and the lack of consideration for others. D. Even those who are employed often come late to the office and leave early unless they are forced to be punctual. E. Work is not intrinsically valued in India. F. Quite often people visit ailing friends and relatives or go out of their way to help them in their personal matters even during office hours.

  • a. ECADBF
  • b. EADCFB
  • c. EADBFC
  • d. ABFCBE
  • e.Not Attempted

35. A. The situations in which violence occurs and the nature of that violence tends to be clearly defined at least in theory, as in the proverbial Irishman's question: Is this a private fight or can anyone join in? B. So the actual risk to outsiders, though no doubt higher than our societies, is calculable. C. Probably the only uncontrolled applications of force are those of social superiors to social inferiors and even here there are probably some rules. D. However binding the obligation to kill, members or feuding families engaged in mutual massacre will be genuinely appalled if by some mischance a bystander or outsider is killed.

  • a. DABC
  • b. ACDB
  • c. CBAD
  • d. DBAC
  • e. Not Attempted

DIRECTIONS for questions 36 to 40:
Each of the words below, a contextual usage is provided. Pick the word from the alternatives given that is most inappropriate in the given context.

36. Disuse: Some words fall into disuse as technology makes objects obsolete

  • a. Prevalent
  • b. Discarded
  • c. Obliterated
  • d. Unfashionable
  • e.Not Attempted

37. Facetious: When I suggested that war is a method of controlling population, my father remarked that I was being facetious

  • a. Jovian
  • b. Jovial
  • c. Jocular
  • d. Joking
  • e.Not Attempted

38. Specious: A specious argument is not simply a false one but one that has the ring of truth.

  • a.Deceitful
  • b. Fallacious
  • c. Credible
  • d. Deceptive
  • e.Not Attempted

39. Parsimonious: The evidence was constructed from very parsimonious scraps of information

  • a. Frugal
  • b. Penurious
  • c. Thrifty
  • d. Altruistic
  • e.Not Attempted

40. Obviate: The new mass transit system may obviate the need for the use of personal cars:

  • a. Prevent
  • b. Forestall
  • c. Preclude
  • d. Bolster
  • e.Not Attempted

41.

Exceed
Dictionary Definition Usage
A. To extend outside of, or enlarge beyond; used chiefly in strictly physical relations E. The mercy of god exceeds our finite minds
B. To be greater than or superior to F. Their accomplishments exceeded our expectation
C. Be beyond the comprehension of G. He exceed his authority when he paid his brother's gambling debts with money from the trust
D. To go beyond a limit set by (as an authority or privilege) H. If this rain keeps up, the river will exceed its banks by morning
  • a. A-H, B-F, C-E, D-G
  • b. A-H, B-E, C-F, D-G
  • c. A-G, B-F, C-E, D-H
  • d. A-F, B-G, C-H, D-E
  • e.Not Attempted

42.

Infer
Dictionary Definition Usage
A. To derive by reasoning or implication E. We see smoke and infer fire
B. To surmise F. Given some utterance, a listener may infer from it things which the utterer never implied
C. To point out G. I waited all day to meet him, form this you can infer my zeal to see him
D. To hint H. She did not take part in the debate except to ask a question inferring that she was not interested in the debate
  • a. A-G, B-H, C-E, D-F
  • b. A-F, B-H, C-E, D-G
  • c. A-H, B-G, C-F, D-E
  • d. A-E, B-F, C-G, D-H
  • e.Not Attempted

43.

Mellow
Dictionary Definition Usage
A. Adequately and properly ages so as to be free of harshness E. He has mellowed with age
B. Freed from the rashness of youth F. The tones of the old violin were mellow
C. Of soft and loamy consistency G. Some wines are mellow
D. Rich and full but free from stridency H. Mellow soil is found in the Gangetic plains
  • a. A-E, B-G, C-F, D-H
  • b. A-E, B-F, C-G, D-H
  • c. A-G, B-E, C-H, D-F
  • d. A-H, B-G, C-F, D-E
  • e.Not Attempted

44.

Relief
Dictionary Definition Usage
A. Removal or lightening of something distressing E. A ceremony follows the relief of a sentry after the morning shift
B. Aid in the form of necessities for the indigent F. It was a relief to take off the tight shoes
C. Diversion G. The only relief I get is by playing cards
D. Release from the performance of duty H. Disaster relief was offered to the victims
  • a. A-F, B-H, C-E, D-G
  • b. A-F, B-H, C-G, D-E
  • c. A-H, B-F, C-G, D-E
  • d. A-G, B-E, C-H, D-F
  • e.Not Attempted

45.

Purge
Dictionary Definition Usage
A. Remove a stigma from the name of E. The opposition was purged after the coup
B. Make clean by removing whatever is superfluous, foreign F. The committee heard his attempt to purge himself of a charge of heresy
C. Get rid of G. Drugs that purge the bowels are often bad for the brain
D. To cause evacuation of H. It is recommended to purge water by distillation
  • a. A-E, B-G, C-F, D-H
  • b. A-F, B-E, C-H, D-G
  • c. A-H, B-F, C-G, D-E
  • d. A-F, B-H, C-E, D-G
  • e.Not Attempted

DIRECTIONS for question 46 to 50: In each of the following sentences, parts of the sentence are left blank. Beneath each sentence, four different ways of completing the sentence are indicated. Choose the best alternative from among the four.

46. But ___________ are now regularly written to describe well-established practices, organisations and institutions, not all of which seem to be ________ away.

  • a. reports, withering
  • b. stories, trading
  • c. books, dying
  • d.obituaries, fading
  • e.Not Attempted

47. The Darwin who ___________ is most remarkable for the way in which he _________the attributes of the world class thinker and head of the household.

  • a. comes, figures
  • b. arises, adds
  • c. emerges, combines
  • d. appeared, combines
  • e.Not Attempted

48. Since her face was free of __________ there was no way to __________ if she appreciated what had happened.

  • a. make-up, realise
  • b. expression, ascertain
  • c. emotion, diagnose
  • d. scars, understand
  • e.Not Attempted

49.In this context, the ___________ of the British labour movement is particularly ___________ .

  • a. affair, weird
  • b. activity, moving
  • c. experience, significant
  • d. atmosphere, gloomy
  • e.Not Attempted

50. Indian intellectuals may boast, if they are so inclined, of being __________ to the most elitist among the intellectual ___________ of the world.

  • a. subordinate, traditions
  • b. heirs, cliques
  • c. ancestors, societies
  • d. heir, traditions
  • e.Not Attempted

CAT2001 SECTION - DI

Instructions
1. The test comprises of 50 questions. You should complete the test within 40 minutes.
2. There is only one correct answer to each question.
3. All questions carry four marks each.
4. Each wrong answer will attract a penalty of one mark.

DIRECTIONS for questions 1 to 7:
Answer each of the questions independent of each other.

1. Four friends Ashok, Bashir, Chirag and Deepak are out shopping. Ashok has less money than three times the amount that Bashir has. Chirag has more money than Bashir. Deepak has an amount equal to the difference of amounts with Bashir and Chirag. Ashok has three times the money with Deepak. They each have to buy a least one shirt, or one shawl, or one sweater, or one jacket, that are priced Rs.200, Rs. 400, Rs.600 and Rs.1000 apiece, respectively. Chirag borrows Rs.300 from Ashok and buys a jacket. Bashir buts a sweater after borrowing Rs.100 from Ashok and is left with no money. Ashok buys three shirts. What is the costliest item that Deepak could buy with his own money?

  • a. A Shirt
  • b. A Shawl
  • c. A sweater
  • d. A jacket
  • e.Not Attempted

2. In a family gathering there are two males who are grandfathers and four males who are fathers. In the same gathering there are two females who are grandmothers and four females who are mothers. There is at least one grandson or a granddaughter present in this gathering. There are two husband wife pairs in this group. These can either be a grandfather and a grandmother, or a father and a mother. The single grandfather (whose wife is not present) has two grandsons and a son present. The single grandmother (whose husband is not present) has two grand daughters and a daughter present. A grandfather or a grandmother present with their spouses does not have any grandson or granddaughter present. What is the minimum number of people present in this gathering?

  • a. 10
  • b. 12
  • c. 14
  • d. 16
  • e.Not Attempted

3. Eight people carrying food baskets are going for a picnic on motorcycles. Their names are A, B, C, D, E, F, G, and H. They have four motorcycles M1, M2, M3 and M4 among them. They also have four food baskets O, P, Q and R of different size and shapes and which can be carried only on motorcycles M1, M2, M3 or M4 respectively. No more than two persons can travel on a motorcycle and no more than one basket can be carried on a motorcycle. There are two husband-wife pairs in this group of eight people and each pair will ride on a motorcycle together. C cannot travel with A or B. E cannot travel with B or F. G cannot travel with F, or H, or D. The husband-wife pairs must carry baskets O and P. Q is with A and P is with D. F travels on M1 and E travels on M2 motorcycles. G is with Q, and B cannot go with R. Who is travelling with H?

  • a. A
  • b. B
  • c. C
  • d. D
  • e.Not Attempted

4.I have a total of Rs.1000. Item A costs Rs.110, item B costs Rs.90, item C costs Rs.70, item D costs Rs.40 and item E costs Rs.45. For every item D that I purchase, I must also buy two of item B. For every item A, I must buy one of item C. For every item E, I must also buy two of item D and one of item B. For every item purchased I earn 1000 points and for every rupee not spent I earn a penalty of 150 points. My objective is to maximise the points I earn. What is the number of items that I must purchase to maximise my points?

  • a. 13
  • b. 14
  • c. 15
  • d. 16
  • e.Not Attempted

5. On her walk through the park, Sheetal collected 50 coloured leaves, all either maple or oak. She sorted them by category when she got home, and found the following:
The number of red oak leaves with spots is even and positive.
The number of red oak leaves without any spot equals the number of red maple leaves without spots. All non-red oak leaves have spots, and there are five times as many of them as there are red spotted oak leaves.
There are no spotted maple leaves that are not red.
There are exactly 6 red spotted maple leaves.
There are exactly 22 maple leaves that are neither spotted nor red.
How many oak leaves did she collect?

  • a. 22
  • b. 17
  • c. 25
  • d. 18
  • e.Not Attempted

6. A King has unflinching loyalty from eight of his ministers M1 to M8, but he has to select only four to make a cabinet committee. He decides to choose these four such that each selected person shares a liking with at least one of the other three selected. The selected persons must also hate at least one of the liking of any of the other three persons selected.
M1 likes fishing and smoking, but hates gambling.
M2 likes smoking and drinking, but hates fishing.
M3 likes gambling, but hates smoking.
M4 likes mountaineering, but hates drinking.
M5 likes drinking, but hates smoking and mountaineering.
M6 likes fishing, but hates smoking and mountaineering.
M7 likes gambling and mountaineering, but hates fishing, and
M8 likes smoking and gambling, but hates mountaineering.
Who are the four people selected by the king?

  • a. M1, M2, M5, M6
  • b. M3, M4, M5, M6
  • c. M4, M5, M6, M8
  • d. M1, M2, M4, M7
  • e.Not Attempted

7. In a "keep-fit" gymnasium class there are fifteen females enrolled in a weight-loss program. They all have been grouped in any one of the five weight-groups W1, W2, W3, W4, or W5. One instructor is assigned to one weight-group only. Sonali, Shalini, Shubhra, and Shahira belong to the same weight-group. Sonali and Rupa are in one weight-group, Rupali and Renuka are also in one weight-group. Rupa, Radha, Renuka, Ruchika, and Ritu belong to different weight-groups. Somya cannot be with Ritu, and Tara cannot be with Radha. Komal cannot be with Radha, Somya, or Ritu. Shahira is in W1 and Somya is in W4 with Ruchika. Sweta and Jyotika cannot be with Rupali, but are in a weight-group with total membership of four. No weight-group can have more than five or less than one member. Amita, Babita, Chandrika, Deepika, and Elina are instructors of weight-groups with membership sizes 5,4,3,2 and 1, respectively. Who is the instructor of Radha?

  • a. Babita
  • b. Elina
  • c. Chandrika
  • d. Deepika
  • e.Not Attempted

DIRECTIONS for questions 8-10:
Answer the following questions based on the passage below.

A group of three or four has to be selected from seven persons. Among the seven are two women, Fiza and Kavita, and five men: Ram, Shyam, David, Peter and Rahim. Ram would not like to be in the group if Shyam is also selected. Shyam and Rahim want to be selected together in the group. Kavita would like to be in the group only if David is also there. David, if selected, would not like Peter in the group. Ram would like to be in the group only if Peter is also there. David insists that Fiza be selected in case he is there in the group

8. Which of the following statements is true?

  • a. Kavita and Ram can be part of a group of four.
  • b. A group of four can have two women.
  • c. A group of four can have all four men.
  • d. None of these
  • e.Not Attempted

9. Which of the following is a feasible group of four?

  • a. Ram, Peter, Fiza, Rahim
  • b. Shyam, Rahim, Kavita, David
  • c. Shyam, Rahim, Fiza, David
  • d. Fiza, David, Ram, Peter
  • e.Not Attempted

10.Which of the following is a feasible group of three?

  • a. David, Ram, Rahim
  • b. Peter, Shyam, Rahim
  • c. Kavita, David, Shyam
  • d. Fiza, David, Ram
  • e.Not Attempted

Directions for questions 11 to 12:
Answer the following questions based on the information given below:

Elle is three times older than Yogesh, Zaheer is half the age of Wahida. Yogesh is older than Zaheer.

11. Which of the following information will be sufficient to estimate Elle's age?

  • a. Zaheer is 10 years old.
  • b. Both Yogesh and Wahida are older than Zaheer by the same number of years.
  • c. Both 1 and 2 above
  • d. None of the above
  • e.Not Attempted

12. Which of the following can be inferred?

  • a. Yogesh is older than Wahida.
  • b. Elle is older than Wahida.
  • c. Elle may be younger than Wahida.
  • d. None of the above.
  • e.Not Attempted

DIRECTIONS for questions 13 and 16:
A and B are two sets (e.g. A = mothers, B = women).
The elements that could belong to both the sets (e.g., women who are mothers) is given by the set C = A.B. The elements which could belong to either A or B, or both, is indicated by the set D = AOB.
A set that does not contain any elements is knows as a null set, represented by @ (for example, if none of the women in the set B is a mother, then C = A.B. is a null set, or C = @).
Let 'V' signify the set of all vertebrates; 'M' the set of all mammals; 'D' dogs; 'F' fish; 'A' Alsatian and 'P' a dog named Pluto.

13. If P.A. = @ and POA = D, then which of the following is true?

  • a. Pluto and Alsation are dogs
  • b. Pluto is an Alsatian
  • c. Pluto is not a dog
  • d. D is a null set.
  • e.Not Attempted

14. If y = FO(D.V) is not a null set, it implies that

  • a. All fish are vertebrates
  • b. All dogs are vertebrates.
  • c. Some fish are dogs.
  • d. None of these
  • e.Not Attempted

15.If Z = (P.D)OM, then

  • a.The elements of Z consist of Pluto the dog or any other mammal
  • b. Z implies any dog or mammal.
  • c. Z implies Pluto or any dog that is a mammal.
  • d. Z is a null set.
  • e. Not Attempted

16.Given that X = M.D is such that X = D, which of the following is true?

  • a. All dogs are mammals.
  • b. Some dogs are mammals.
  • c. X = @
  • d.All mammals are dogs.
  • e.Not Attempted

Directions for question 17 to 20:
Answer the questions independent of each other.

17. At a village mela, the following six nautankis (plays) are schedule as shown in the table below.

No. Nautanki Duration Show times
1. Sati Savitri 1 hour 9.00 am and 2.00 p.m.
2. Joru ka Gulam 1 hour 10.30 am and 11.30 a.m.
3. Sunder Kand 30 minutes 10.00 a.m. and 11.00 a.m.
4. Veer Abhimanyu 1 hour 10.00 a.m. and 11.00 a.m.
5. Reshma aur Shera 1 hour 9.30 am, 12.00 noon and 2.00 p.m.
6. Jhansi ki Rani 30 minutes 11.00 a.m. and 1.30 p.m.
Which of the following ways can you do this?
  • a.Sati-Savitri is viewed first; Sunder Kand is viewed third and Jhansi Ki Rani is viewed last
  • b. Sati-Savitri is viewed Last; Sunder Kand is viewed third and Jhansi Ki Rani is viewed last
  • c. Sati-Savitri is viewed first; Sunder Kand is viewed third and Joru ka Gulam is viewed fourth
  • d. Veer Abhimanyu is viewed third; Reshma aur Shera is viewed fourth and Jhansi Ki Rani is viewed fifth.
  • e.Not Attempted

18. While Balbir had his back turned, a dog ran into his butcher shop, snatched a piece of meat off the counter and ran off. Balbir was mad when he realised what had happened. He asked three other shopkeepers, who had seen the dog, to describe it. The shopkeepers really didn't want to help Balbir. So each of them made a statement which contained one truth and one lie.
Shopkeeper Number 1 said: "The dog had black hair and a long tail."
Shopkeeper Number 2 said: "The dog had a short tail and wore a collar."
Shopkeeper Number 3 said: "The dog had white hair and no collar."
Based on the above statements, which of the following could be a correct description?

  • a. The dog had white hair, short tail and no collar
  • b. The dog had white hair, long tail and a collar.
  • c. The dog had black hair, long tail and a collar.
  • d. The dog had black hair, long tail and no collar
  • e.Not Attempted

19.The Bannerjees, the Sharmas and the Pattabhirmans each have a tradition of eating Sunday lunch as a family. Each family serves a special meal at a certain time of day. Each family has a particular set of chinaware used only for this meal. Use the clues below to answer the following question.
The Sharma family eats at noon.
The family that serves fried brinjal uses blue chinaware
The Bannerjee family eats at 2 o' clock
The family that serves sambar does not use red chinaware.
The family that eats at 1 o' clock serves fried brinjal.
The Pattabhiraman family does not use white chinaware.
The family that eats last likes makki-ki-roti. Which one of the following statements is true?

  • a. The Bannerjees eat makki-ki-roti at 2 o' clock, the Sharmas eat fried brinjal at 12'o clock and the Pattabhiramans eat sambar from red chinaware.
  • b. The Sharmas eat sambar served in white chinaware, the Pattabhiramans eat fried brinjal at 1'o clock and the Bannerjees eat makki-ki-roti in blue chinaware.
  • c. The Sharmas eat sambhar at noon, the Pattabhirmanas eat fried brinjal served in blue chinaware and the Bannerjees eat makki-ki-roti served in red chinaware
  • d. The Bannerjees eat makki-ki-roti served in white chinaware, the Sharmas eat fried brinjal at 12'o clock and the Pattabhiramans eat sambar from red chinaware
  • e.Not Attempted

20. Mrs. Ranga has three children and has difficulty remembering their ages and the months of their birth. The clues below may help her remember.
A. The boy, who was born in June, is 7 years old.
B. One of the children is 4 years old, but is not Anshuman
C. Vaibhav is older than Supriya.
D. One of the children was born in September but it was not Vaibhav.
E. Supriya's birthday is in April.
F. The youngest child is only 2 years old.
Based on the above clues, which one of the following statements is true?

  • a. Vaibhav is the oldest, followed by Anshuman who was born in September, and the youngest is Supriya who was born in April
  • b. Anshuman is the oldest being born in June, followed by Supriya who is 4 year old, and the youngest is Vaibhav who is 2 years old
  • c. Vaibhav is the oldest being 7 years old, followed by Supriya who was born in April, and the youngest is Anshuman who was born in September
  • d. Supriya is the oldest, who was born in April, followed by Vaibhav who was born in June, and Anshuman who was born in September
  • e.Not Attempted

Directions for question 21 to 24:
Answer these questions based on the table given below concerning the busiest twenty international airports in the world.

No.

Name

International Airport type

Code

Location

Passengers

1.

Hartsfield

A

ATL

Atlanta, Georgia, USA

77939536

2.

Chicago-O'Hare

A

ORD

Chicago, Illinois USA

72568076

3.

Los Angeles

A

LAX

Los Angeles, California, USA

63876561

4.

Heathrow Airport

E

LHR

London, United Kingdom

62263710

5.

DFW

A

DFW

Dallas/ft. Worth , Texas USA

60000125

6.

Hander Airport

F

HND

Tokyo, Japan

54338212

7.

Frankfurt Airport

E

FRA

Frankfurt, Germany

45858315

8.

Rouse-Charles de Gaulle

E

CDG

Paris, France

43596943

9.

San Francisco

A

SFO

San Francisco, California USA

40387422

10.

Denver

A

DIA

Denver, Colorado, USA

38034231

11.

Amsterdam Schipol

E

AMS

Amsterdam, Netherlands

36781015

12.

Minneapolis-St.Paul

A

MSP

Minneapolis-St.Paul, USA

34216331

13.

Detroit Metropolitan

A

DTW

Detroit, Michigan, USA

34038381

14.

Miami

A

MIA

Miami, Florida, USA

33899246

15.

Newark

A

EWR

Newark, New Jersey, USA

33814000

16.

McCarran

A

LAS

Las Vegas, Nevada, USA

33669185

17.

Phoenix Sky harbor

A

PHX

Phoenix, Arizona USA

33533353

18.

Kimpo

FE

SEL

Seoul, Korea

33371074

19.

George Bush

A

IAH

Houston, Texas, USA

33089333

20.

John F. Kennedy

A

JFK

New York, New York, USA

32003000

21. What percentage of top ten busiest airports is in the United States of America?

  • a. 60
  • b. 80
  • c. 70
  • d. 90
  • e.Not Attempted

22. How many international airports not located in the USA of type 'A' account for more than 30 million passengers?

  • a. 4
  • b. 5
  • c. 6
  • d. 7
  • e.Not Attempted

23. How many international airports of type "A" account for more than 40 million passengers?

  • a. 5
  • b. 6
  • c. 10
  • d. 14
  • e.Not Attempted

24.Of the five busiest airports, roughly what percentage of passengers is handled by Heathrow airport?

  • a. 30
  • b. 40
  • c. 20
  • d. 50
  • e.Not Attempted

Directions for question 25 to 28:
Answer the questions based on the table given below: The following is a table describing garments manufactured based upon the colour and size for each lot. There are four sizes: M-Medium, L-Large, XL-Extra Large, and XXL- Extra-Extra-Large. There are three colours: Yellow, Red and White.

Number of Garments


Yellow

Red

White

Lot No.

M

L

XL

XXL

M

L

XL

XXL

M

L

XL

XXL

1.

14

14

7

0

0

0


0

0

0

0

0

2.

0

0

0

0

0

0

0

0

42

42

21

0

3.

20

20

10

0

18

18

9

0

0

0

0

0

4.

20

20

10

0

0

0

0

0

30

30

15

0

5.

0

0

0

0

24

24

12

0

30

30

15

0

6.

22

22

11

0

24

24

12

0

32

32

16

0

7.

0

24

24

12

0

0

0

0

0

0

0

0

8.

0

20

20

10

0

2

2

1

0

0

0

0

9.

0

20

20

10

0

0

0

0

0

22

22

11

10.

0

0

0

0

0

26

26

13

0

22

22

11

11.

0

22

22

11

0

26

26

13

0

22

22

11

12.


0

0

2

2

0

0

0

0

0

0

0

0

13.


0

0

0

0

0

0

0

0

0

0

20

20

14.


0

0

0

0

0

0

0

0

0

0

22

22

15.


0

0

10

10

0

0

2

2

0

0

22

22

16.


0

0

0

0

1

0

0

0

1

0

0

0

17.


0

0

0

0

0

5

0

0

0

0

0

0

18.


0

0

0

0

0

32

0

0

0

0

0

0

19.


0

0

0

0

0

32

0

0

0

0

0

0

20.


0

0

0

0

0

5

0

0

0

0

0

0

21.

0

0

0

18

0

0

0

0

0

0

0

0

22.

0

0

0

0

0

0

0

26

0

0

0

0

23.

0

0

0

0

0

0

0

0

0

0

0

22

24.

0

0

0

8

0

0

0

1

0

0

0

0

25.

0

0

0

8

0

0

0

0

0

0

0

12

26.


0

0

0

0

0

0

0

0

0

0

0

14

27.


0

0

0

8

0

0

0

2

0

0

0

12

Production

76

162

136

97

67

194

89

59

135

198

195

156

Order


75

162

135

97

67

195

89

59

135

197

195

155

Surplus

1

0

1

0

0

0

0

0

0

1

0

1

25.How many varieties of fabric, which exceed the order, have been produced?

  • a. 3
  • b. 4
  • c. 3
  • d. 5
  • e.Not Attempted

26. How many lots are used to produce extra-extra-large fabrics?

  • a. 15
  • b. 16
  • c. 17
  • d. 18
  • e.Not Attempted

27. How many lots are used to produce Extra-Extra-Large Yellow or Extra-Extra-Large White fabrics?

  • a. 8
  • b. 9
  • c. 10
  • d. 15
  • e.Not Attempted

28. How many lots are used to produce Yellow coloured fabrics?

  • a. 10
  • b. 11
  • c. 12
  • d. 14
  • e.Not Attempted

Directions for question 29 to 31:
Answer these questions based on the pipeline diagram below. The following sketch shows the pipelines carrying material form one location to another, Each location has a demand for material. The demand at Vaishali is 400 at Mathura is 400, at Jhampur is 700 and at Vidisha is 200. Each arrow indicates the direction of material flow through the pipeline. The flow from Vaishali to Mathura is 300; the quantity of material flow is such that the demands at all these locations are exactly met. The capacity of each pipeline is 1000.

29. What is the free capacity available in the Avanti-Vidihsa Pipeline?

  • a. 300
  • b. 200
  • c. 100
  • d. 0
  • e.Not Attempted

30. What is the free capacity available from Avanti to Vaishali?

  • a. 0
  • b. 100
  • c. 200
  • d. 300
  • e.Not Attempted

31. The quantity moved from Avanti to Vidisha is

  • a. 200
  • b. 800
  • c. 700
  • d. 1000
  • e.Not Attempted

Directions for questions 32 to 34:
The questions based on the pie charts given below: Chart 1 shows the distribution of twelve million tonnes of crude oil transport through different modes over a specific period of time. Chart 2 shows the distribution of the cost of transporting this crude oil. The total cost was Rs. 30 million. Diagram

32. If the cost per tonne of transport by ship, air and road are represented by P, Q and R respectively, which of the following is true?

  • a. R > Q > P
  • b. P > R > Q
  • c. P > Q > R
  • d. R > P > Q
  • e.Not Attempted

33. The cost in rupees per tonne of oil moved by rails and happens to be roughly

  • a. 3
  • b. 1.5
  • c. 4.5
  • d. 8
  • e.Not Attempted

34. From the charts given, it appears that the cheapest mode of transport is:

  • a. Road
  • b. Rail
  • c. Pipeline
  • d. Ship
  • e.Not Attempted

DIRECTIONS for questions 35 to 41:
Each item is followed by two statements, A and B. Answer each question using the following instructions.
Choose A if the question can be answered by one of the statements alone and not by the other.
Choose B if the question can be answered by using either statement alone.
Choose C if the question can be answered by using both the statements together, but cannot be answered by using either statement alone.
Choose D if the question cannot be answered even by using both statements together.

35. Two friends, Ram and Gopal, bought apples from a wholesale dealer. How many apples did they buy?
A. Ram bought one-half the number of apples that Gopal bought.
B. The wholesale dealer had a stock of 500 apples.

  • a. A
  • b. B
  • c. C
  • d. D
  • e. Not Attempted

36. Is country X's GDP higher than country Y's GDP
A. GDPs of the countries X and Y have grown over the past five years at compounded annual rate of 5% and 6% respectively.
B. Five years ago, GDP of country X was higher than that of country Y.

  • a. A
  • b. B
  • c. C
  • d. D
  • e.Not Attempted

37.What is the value of X?
A. X and Y are unequal even integers, less than 10, and X/Y is an odd integer.
B. X and Y are even integers, each less than 10, and product of X and Y is 12

  • a. A
  • b. B
  • c. C
  • d. D
  • e.Not Attempted

38. On a given day a boat ferried 1500 passengers across the river in twelve hours. How many round trips did it make?
A. The boat can carry two hundred passengers at any time.
B. It takes 40 minutes each way and 20 minutes of waiting time at each terminal.

  • a. A
  • b. B
  • c. C
  • d. D
  • e.Not Attempted

39. What will be the time for downloading software?
A. Transfer rate is 6 Kilobytes per second.
B. The size of the software is 4.5 megabytes.

  • a. A
  • b. B
  • c. C
  • d. D
  • e.Not Attempted

40. A square is inscribed in a circle. What is the difference between the area of the circle and that of the square?
A. The diameter of the circle is 25/2 cm
B. The side of the square is 25 cm.

  • a. A
  • b. B
  • c. C
  • d. D
  • e.Not Attempted

41. What are the values of m and n?
A. n is an even integer, m is an odd integer, and m is greater than n.
B. Product of m and n is 30.

  • a. A
  • b. B
  • c. C
  • d. D
  • e.Not Attempted

DIRECTIONS for questions 42 to 44:
Answer these questions based on the data given below There are six companies, I through 6. All of these companies use six operations, A through F. The graph shows the distribution of efforts put in by each company in these six operations.

42.Suppose effort allocation is inter-changed between operations B and C, then C and D, and then D and E. If companies are then ranked in ascending order of effort, which company would be at third rank?

  • a. 2
  • b. 3
  • c. 4
  • d. 5
  • e.Not Attempted

43. Suppose the companies find that they can remove operations B, C and D and re-distribute the effort released equally among the remaining operations. Then, which operation will show the maximum across all companies and all operations?

  • a. Operation E in company 1
  • b.Operation E in Company 4
  • c. Operation F in company 5
  • d. Operation E in company 5
  • e.Not Attempted

44. A new technology is introduced in company 4 such that the total effort for operations B through F get evenly distributed among these. What is the change in the percentage of effort in operation E?

  • a. Reduction of 12.3
  • b. Increase of 12.3
  • c. Reduction of 5.6
  • d. Increase of 5.6
  • e.Not Attempted

DIRECTIONS for questions 45 to 50:
Answer these questions based on the two graphs shown below: Figure 1 shows the amount of work distribution, in man-hours for a software company between offshore and onsite activities. Figure 2 shows the estimated and actual work effort involved in the different offshore activities in the same company during the same period. (Note: Onsite refers to work performed at the customer's premise and offshore refers to work performed at the developer's premise)

45. If 50 percent of the offshore work to be carried out onsite, with the distribution of effort between the tasks remaining the same, which of the following is true of all work carried out onsite?

  • a. The amount of coding done is greater than that of testing.
  • b. The amount of coding done onsite is less than that of design done onsite.
  • c. The amount of design carried out onsite is greater than that of testing.
  • d. The amount of testing carried out offshore is greater than that of total design.
  • e.Not Attempted

46. Roughly what percentage of total work is carried on site?

  • a. 40
  • b. 20
  • c. 30
  • d. 50
  • e.Not Attempted

47. The total effort in hours onsite is nearest to which of the following?

  • a. Sum of estimated and actual effort for offshore design
  • b. The estimated man Hours of offshore coding
  • c. The actual man-hours of off shore testing
  • d. Half the no of estimated man-hours of off shore coding
  • e.Not Attempted

48.If the total working hours were 100 which of the following tasks will account for approx. 50 hours

  • a. Coding
  • b. Design
  • c. Off shore testing
  • d. Off shore testing
  • e.Not Attempted

49. If 50 percent of the offshore work is to be carried out onsite, with the distribution of effort between the tasks remaining the same, the percentage of testing carried out off shore would be

  • a. 40%
  • b. 30%
  • c. 50%
  • d. 70%
  • e.Not Attempted

50. Which of the work requires as many manhours as that spent in coding

  • a. Offshore, design and coding
  • b. Offshore coding
  • c. Testing
  • d. Offshore testing and coding.
  • e.Not Attempted

CAT2002 SECTION - ENGLISH

Instructions
1. The test comprises of 50 questions. You should complete the test within 40 minutes.
2. There is only one correct answer to each question.
3. All questions carry four marks each.
4. Each wrong answer will attract a penalty of one mark.

Directions Questions 1-5:
For each of the words below a context is provided. From the alternatives given pick the word or phrase that is closest in meaning in the given context.

1. Opprobrium: The police officer appears oblivious to the opprobrium generated by his blatantly partisan conduct.

  • a. Harsh criticism
  • b. Acute distrust
  • c. Bitter enmity
  • d. Stark oppressiveness
  • e.Not Attempted

2. Portend: It appears to many that the US "war on terrorism" portends trouble in the Gulf.

  • a. Introduces
  • b. Evokes
  • c. Spells
  • d. Bodes
  • e.Not Attempted

3. Prevaricate: When a videotape of her meeting was played back to her and she was asked to explain her presence there, she started prevaricating.

  • a. Speaking evasively
  • b. Speaking violently
  • c. Lying furiously
  • d. Throwing a tantrum
  • e.Not Attempted

4. Restive: The crowd became restive when the minister failed to appear even by 10 pm.

  • a. Violent
  • b. Angry
  • c. Restless
  • d. Distressed
  • e.Not Attempted

5.Ostensible: Manohar's ostensible job was to guard the building at night.

  • a. Apparent
  • b. Blatant
  • c. Ostentatious
  • d. Insidious
  • e.Not Attempted

Directions: Questions6-9:
In each of the questions below, four different ways of writing a sentence are indicated. Choose the best way of writing the sentence.

6. A. The main problem with the notion of price discrimination is that it is not always a bad thing, but that it is the monopolist who has the power to decide who is charged what price.
B. The main problem with the notion of price discrimination is not that it is always a bad thing, it is the monopolist who has the power to decide who is charged what price.
C. The main problem with the notion of price discrimination is not that it is always a bad thing, but that it is the monopolist who has the power to decide who is charged what price.
D. The main problem with the notion of price discrimination is not it is always a bad thing, but that it is the monopolist who has the power to decide who is charged what price.

  • a. A
  • b. B
  • c. C
  • d. D
  • e.Not Attempted

7. A. A symbiotic relationship develops among the contractors, bureaucracy and the politicians and by a large number of devices costs are artificially escalated and black money is generated by underhand deals.
B. A symbiotic relationship develops among contractors, bureaucracy and politicians, and costs are artificially escalated with a large number of devices and black money is generated through underhand deals.
C. A symbiotic relationship develops among contractors, bureaucracy and the politicians and by a large number of devices costs are artificially escalated and black money is generated on underhand deals.
D. A symbiotic relationship develops among the contractors, bureaucracy and politicians, and by large number of devices costs are artificially escalated and black money is generated by underhand deals

  • a.A
  • b. B
  • c. C
  • d. D
  • e.Not Attempted

8. A. The distinctive feature of tariffs and export subsidies is that they create difference of prices at which goods are traded on the world market and their price within a local market.
B. The distinctive feature of tariffs and export subsidies is that they create a difference of prices at which goods are traded with the world market and their prices in the local market.
C. The distinctive feature of tariffs and export subsidies is that they create a difference between prices at which goods are traded on the world market and their prices within a local market.
D. The distinctive feature of tariffs and export subsidies is that they create a difference across prices at which goods are traded with the world market and their prices within a local market.

  • a. A
  • b. B
  • c. C
  • d. D
  • e.Not Attempted

9. A. Any action of government to reduce the systemic risk inherent in financial markets will also reduce the risks that private operators perceive and thereby encourage excessive hedging.
B. Any action by government to reduce the systemic risk inherent in financial markets will also reduce the risks that private operators perceive and thereby encourage excessive gambling.
C. Any action by government to reduce the systemic risk inherent due to financial markets will also reduce the risks that private operators perceive and thereby encourages excessive hedging.
D. Any action of government to reduce the systemic risk inherent in financial markets will also reduce the risks that private operators perceive and thereby encourages excessive gambling.

  • a. A
  • b.B
  • c. C
  • d. because of the expression of anger in her songs.
  • e.Not Attempted

Directions (Questions 10-15):
Fill the gaps in the passages below with the most appropriate word from the options given for each gap. The right words are the ones used by the author. Be guided by the author's overall style and meaning when you choose the answers.

Von Nuemann and Morgenstern assume a decision framework in which all options are throughly considered, each option being independent of the others, with a numerical value derived for the utility of each possible outcome (these outcomes reflecting, in turn, all possible combinations of choices). The decision is then made to maximize the expected utility.

____Q.110______, such a model reflects major simplifications of the way decisions are made in the real world. Humans are not able to process information as quickly and effectively as the model assumes; they tend not to think ____Q.111_______ as easily as the model calls for; they often deal with a particular option without really assessing its______Q.112________ and when they do assess alternatives, they may be extremely nebulous about their criteria of evaluation.

10.

  • a. Regrettably
  • b. Firstly
  • c. Obviously
  • d. Apparently
  • e.Not Attempted

11.

  • a. Quantitatively
  • b. Systematically
  • c. Scientifically
  • d. Analytically
  • e.Not Attempted

12.

  • a. Implications
  • b. Disadvantages
  • c. Utility
  • d. Alternatives
  • e.Not Attempted

In a large company, _____113__________ people is about as common as using a gun or a switch-blade to ______114_______ an argument. As a result, most managers have little or no experience of firing people, and they find it emotionally traumatic; as a result, they often delay the act interminably, much as an unhappy spouse will prolong a bad marriage. And when the firing is done, it's often done clumsily, with far worse side effects than are necessary. Do the world-class software organizations have a different way of firing people? No, but they do the deed swiftly, humanely, and professionally.
The key point here is to view the fired employee as a 'failed product' and to ask how the process_____115_____ such a phenomenon in the first place.

13.

  • a. Dismissing
  • b. Punishing
  • c. Firing
  • d. Admonishing
  • e.Not Attempted

14.

  • a. Resolve
  • b. Thwart
  • c. Defeat
  • d. Close
  • e.Not Attempted

15.

  • a. Derived
  • b. Engineered
  • c. Produced
  • d. Allowed
  • e. Not Attempted

Directions (Questions 16-20):
The sentences given in each question, when properly sequenced, form a coherent paragraph. Each sentence is labeled with a letter. Choose the most logical order of sentences from among the given choices to construct a coherent paragraph.

16. A. Branded disposabie diapers are available at many supermarkets and drug stores. B. If one supermarket sets a higher price for a diaper, customers may buy that brand elsewhere.
C. By contrast, the demand for private-label products may be less price sensitive since it is available only at a corresponding supermarket chain.
D. So, the demand for branded diapers at any particular store may be quite price sensitive.
E. For instance, only SavOn Drug stores sell SavOn Drugs diapers.
F. Then, stores should set a higher incremental margin percentage for private-label diapers

  • a. ABCDEF
  • b. ABCEDF
  • c. ADBCEF
  • d. AEDBCF
  • e.Not Attempted

17. A. Having a strategy is a matter of discipline. B. It involves the configuration of a tailored value chain that enables a company to offer unique value.
C. It requires a strong focus on profitability and a willingness to make tough trade offs in choosing what not to do.
D. Strategy goes far beyond the pursuit of best practices.
E. A company must stay the course even during times of upheaval while constantly improving and extending its distinctive positioning.
F. When a company's activities fit together as a self-reinforcing system, any competitor wishing to imitate a strategy must replicate the whole system

  • a. ABCDEF
  • b. ACEDBF
  • c. ADBCEF
  • d. AEDBCF
  • e.Not Attempted

18. A. As officials their vision of a country shouldn't run too far beyond that of the local people with whom they have to deal.
B. Ambassadors have to choose their words.
C. To say what they feel they have to say, they appear to be denying or ignoring part of what they know.
D. So, with ambassadors as with other expatriates in black Africa, there appears at a first meeting a kind of ambivalence.
E. They do a specialised job and it is necessary for them to live ceremonial lives.

  • a. BCEDA
  • b. BEDAC
  • c. BEADC
  • d. BCDEA
  • e.Not Attempted

19. A. “This face off will continue for several months given the strong convictions on either side,” says a senior functionary of the high-powered task force on drought.
B. During the past week-and-half, the Central Government has sought to deny some of the earlier apprehensions over the impact of drought.
C. The recent revival of the rains had led to the emergence of a line of divide between the two.
D. The state governments, on the other hand, allege that the Centre is downplaying the crisis only to evade its full responsibility of financial assistance that is required to alleviate the damage.
E. Shrill alarm about the economic impact of an inadequate monsoon had been sounded by the Centre as well as most of the states, in late July and early August.

  • a. EBCDA
  • b. DBACE
  • c. BDCAE
  • d. ECBDA
  • e.Not Attempted

20. A. This fact was established in the 1730s by French survey expeditions to Equador near the Equator and Lapland in the Arctic, which found that around the middle of the earth the arc was about a kilometer shorter.
B. One of the unsettled scientific questions in the late 18th century was the exact nature of the shape of the earth.
C. The length of one-degree arc would be less near the equatorial latitudes than at the poles.
D. One way of doing that is to determine the length of the arc along a chosen longitude or meridian at one-degree latitude separation.
E. While it was generally known that the earth was not a sphere but an ‘oblate spheroid’, more curved at the equator and flatter at the poles, the question of ‘how much more’ way yet to be established.

  • a. BECAD
  • b. BEDCA
  • c. EDACB
  • d. EBDCA
  • e.Not Attempted

Directions (Questions 21-25):
For the word given at the top of each table, match the dictionary definitions on the left (A, B, C, D) with their corresponding usage on the right (E, F, G, H). Out of the four possibilities given in the boxes below the table, select the one that has all the definitions and their usages most closely matched.

21.

Measure
Dictionary Definition Usage
A. Size or quantity found by measuring E. TA measure was instituted to prevent outsiders from entering the campus.
B. Vessel of standard capacity F. Sheila was asked to measure each item that was delivered
C. Suitable action G. The measure of the cricket pitch was 22 yards
D. Ascertain extent or quantity H. Ramesh used a measure to take out one litre of oil.
  • a. A-H, B-F, C-E, D-G
  • b. A-G, B-E, C-F, D-H
  • c. A-G, B-H, C-E, D-F
  • d. A-F, B-H, C-E, D-G
  • e.Not Attempted

22.

Bound
Dictionary Definition Usage
A. Obliged, constrained E. Dinesh felt bound to walk out when the discussion turned to kickbacks.
B. Limiting value F. Buffeted by contradictory forces he was bound to lose his mind
C. Move in a specified direction G. Vidya’s story strains the bounds of credulity.
D. Destined or certain to be H. Bound for a career in law, Jyoti was reluctant to study Milton
  • a. A-F, B-H, C-G, D-E
  • b. A-E, B-G, C-H, D-F
  • c. A-E, B-H, C-F, D-G
  • d. A-F, B-G, C-E, D-H
  • e.Not Attempted

23.

Catch
Dictionary Definition Usage
A. Capture E. All her friends agreed that Prasad was a good catch
B. Grasp with senses or mind F. The proposal sounds very good but where is the catch?
C. Deception G. THussain tries to catch the spirit of India in this painting
D. Thing or person worth trapping H. Sorry, I couldn’t catch you.
  • a. A-H, B-F, C-E, D-G
  • b. A-F, B-G, C-E, D-H
  • c. A-G, B-F, C-E, D-H
  • d. A-G, B-H, C-F, D-E
  • e.Not Attempted

24.

Deal
Dictionary Definition Usage
A. Manage, attend to E. Dinesh insisted on dealing the cards
B. Stock, sell F. This contract deals with handmade cards.
C. Give out to a number of people G. My brother deals in cards.
D. Be concerned with H. I decided not to deal with handmade cards
  • a. A-H, B-E, C-F, D-G
  • b. A-G, B-H, C-E, D-F
  • c. A-G, B-E, C-F, D-H
  • d. A-G, B-F, C-H, D-E
  • e.Not Attempted
25.
Turn
Dictionary Definition Usage
A. Give new direction tO E. It was now his turn to be angry
B. Send F. Leena never turned away a beggar.
C. Change in form G. Ashish asked Laxman to turn his face to the left
D. Opportunity coming successively for each person H. The old school building has been turned into a museum
  • a. A-H, B-E, C-F, D-G
  • b. A-G, B-F, C-E, D-H
  • c. A-G, B-E, C-F, D-H
  • d. A-G, B-F, C-H, D-E
  • e.Not Attempted

Directions (Questions 26-50):
Each of the five passages is given below is followed by questions. Choose the best answer for each question.

Passage 1

Cells are the ultimate multitaskers: they can switch on genes and carry out their orders, talk to each other, divide in two, and much more, all at the same time. But they couldn’t do any of these tricks without a power source to generate movement. The inside of a cell bustles with more traffic than Delhi roads, and, like all vehicles, the cell’s moving parts need engines. Physicists and biologists have looked ‘under the hood’ of the cell and laid out the nuts and bolts of molecular engines.

The ability of such engines to convert chemical energy into motion is the envy of the nanotechnology researchers looking for ways to power molecule-sized devices. Medical researchers also want to understand how these engines work. Because these molecules are essential for cell division, scientists hope to shut down the rampant growth of the cancer cells by deactivating certain motors. Improving motor-driven transport in nerve cells may also be helpful for treating diseases such as Alzheimer’s, Parkinson’s or ALS, also known as Lou Gehrig’s disease.

We wouldn’t make it far in life without motor proteins. Our muscles wouldn’t contract. We couldn’t grow, because the growth process requires cells to duplicate their machinery and pull the copies apart. And our genes would be silent without the services of messenger RNA, which carries genetic instructions over to the cell’s protein-making factories. The movements that make these cellular activities possible occur along a complex network of threadlike fibers, or polymers, along which bundles of molecules travel like trams. The engines that power the cell’s freight are three families of proteins, called myosin, kinesin and dynein. For fuel, these proteins burn molecules, of ATP, which cells make when they break down the carbohydrates and fats from the foods we eat. The energy from burning ATP causes changes in the proteins’ shape that allow them to heave themselves along the polymer track. The results are impressive: In one second, these molecules can travel between 50 and 100 times their own diameter. If a car with 5-foot-wide engine were as efficient, it would travel 170 to 340 kmph.

Ronald Vale, a researcher at the Howard Hughes Medical Institute and the University of California at San Francisco, and Ronald Milligan of the Scripps Research Institute have realised a long-awaited goal by reconstructing the process by which myosin and kinesin move, almost down to the atom. The dynein motor, on the other hand, is still poorly understood. Myosin molecules, best known for their role in muscle contraction, form chains that lie between filaments of another protein called actin. Each myosin molecule has a tiny head that pokes out from the chain like oars from a canoe. Just as rowers propel their boat by stroking their oars through the water, the myosin molecules stick their heads into the actin and hoist themselves forward along the filament. While myosin moves along in short strokes, its cousin Kinesin walks steadily along a different type of filament called a microtubule. Instead of using a projecting head as a lever, kinesin walks on two ‘legs’. Based on these differences, researchers used to think that myosin and kinesin were virtually unrelated. But newly discovered similarities in the motors’ ATP-processing machinery now suggest that they share a common ancestor-molecule. At this point, scientists can only speculate as to what type of primitive cell-like structure this ancestor occupied as it learned to burn ATP and use the energy to change shape. “We’ll never really know, because we can’t dig up the remains of ancient proteins, but that was probably a big evolutionary leap,” says Vale.

On a slightly larger scale, loner cells like sperm or infectious bacteria are prime movers that resolutely push their way through to other cells. As L. Mahadevan and Paul Matsudaira of the Massachusetts Institute of Technology explain, the engines in this case are springs or ratchets that are clusters of molecules, rather than single protein like myosin and kinesin. Researchers don’t yet fully understand these engines’ fueling process or the details of how they move, but the result is a force to be reckoned with. For example, one such engine is a springlike stall connecting a single-celled organism called a vorticellid to the leaf fragment it calls home. When exposed to calcium, the spring contracts, yanking the vorticellid down at speeds approaching 3 inches (8 centimeters) per second.

Springs like this are coiled bundles of filaments that expand or contract in response to chemical cues. A wave of positively charged calcium ions, for example, neutralizes the negative charges that keep the filaments extended. Some sperm use springlike engines made of actin filaments to shoot out a barb that penetrates the layers that surround an egg. And certain viruses use a similar apparatus to shoot their DNA into the host’s cell. Ratchets are also useful for moving whole cells, including some other sperms and pathogens. These engines are filaments that simply grow at one end, attracting chemical building blocks from nearby. Because the other end is anchored in place, the growing end pushes against any barrier that gets in its way.

Both springs and ratchets are made up of small units that each move just slightly, but collectively produce a powerful movement. Ultimately, Mahadevan and Matsudaira hope to better understand just how these particles create an effect that seems to be so much more than the sum of its parts. Might such an understanding provide inspiration for ways to power artificial nano-sized devices in the future? “The short answer is absolutely,” says Mahadevan. “Biology has had a lot more time to evolve enormous richness in design for different organisms. Hopefully, studying these structures will not only improve our understanding of the biological world, it will also enable us to copy them, take apart their components and re-create them for other purposes.”

26. According to the author, research on the power source of movement in cells can contribute to

  • a. Control over the movement of genes within human systems.
  • b. The understanding of nanotechnology.
  • c. Arresting the growth of cancer in a human being.
  • d. The development of cures for a variety of diseases.
  • e.Not Attempted

27. The author has used several analogies to illustrate his arguments in the article. Which of the following pairs of words are examples of the analogies used? (a) Cell activity and vehicular traffic. (b) Polymers and tram tracks. (c) Genes and canoes (d) Vorticellids and ratchets.

  • a. a and b
  • b. b and c
  • c. a and d
  • d. a and c
  • e.Not Attempted

28. Read the five statements below: a, b, c, d and e. From the options given, select the one which includes a statement that is NOT representative of an argument presented in the passage. (a) Sperms use spring like engines made of actin filament.
(b) Myosin and kinesin are unrelated.
(c) Nanotechnology researchers look for ways to power molecule-sized devices.
(d) Motor proteins help muscle contraction.
(e) The dynein motor is still poorly understood.

  • a. a, b and c
  • b. c, d and e
  • c. a, d and e
  • d. a, c and d
  • e.Not Attempted

29. Read the four statements below: a, b, c and d. From the options given, select the one which includes only statements(s) that are representative of arguments presented in the passage.
(a) Protein motors help growth processes.
(b) Improved transport in nerve cells will help arrest tuberculosis and cancer.
(c) Cells, together, generate more than the sum of power generated by them separately.
(d) Vorticellid and the leaf fragment are connected by a calcium engine.

  • a. a and b but not c
  • b. a and c but not d
  • c. a and d but not b
  • d. c and d but not b.
  • e.Not Attempted

30.Read the four statements below: a, b, c, and d. From the options given, select the one which include statements(s) that are representative of arguments presented in the passage.
(a) Myosin, kinesin and actin are three types of proteins
(b) Growth processes involve a routine in a cell that duplicates their machinery and pulls the copies apart.
(c) Myosin molecules can generate vibrations in muscles
(d) Ronald and Mahadevan are researchers at Massachusetts Institute of Technology.

  • a. a and b but not c and d
  • b. b and c but not a
  • c. b and d but not a and c
  • d. a, b and c but not d
  • e.Not Attempted

Passage 2

The conceptions of life and the world which we call ‘philosophical’ are a product of two factors: one, inherited, religious and ethical conceptions; the other, the sort of investigation which may be called ‘scientific’, using this word in its broadest sense. Individual philosophers have differed widely in regard to the proportions in which these two factors entered into their systems, but it is the presence of both, in some degree, that characterises philosophy.

‘Philosophy’ is a word which has been used in many ways, some wider, some narrower. I propose to use it in a very wide sense, which I will now try to explain.

Philosophy, as I shall understand the word, is something intermediate between theology and science. Like theology, it consists of speculations on matters as to which definite knowledge has, so far, been unascertainable; but like science, it appeals to human reason rather than to authority, whether that of tradition or that of revelation. All definite knowledge -- so I should contend -- belongs to science; all dogma as to what surpasses definite knowledge belongs to theology. But between theology and science there is a ‘No man’s Land’ exposed to attack from both sides; this ‘No Man’s Land’ is philosophy. Almost all the questions of most interest to speculative minds are such as science cannot answer, and the confident answers of theologians no longer seem so convincing as they did in former centuries. Is the world divided into mind and matter, and if so, what is mind and what is matter? Is mind subject to matter, or is it possessed of independent powers? Has the universe any unity or purpose? Is it evolving towards some goal? Are there really laws of nature, or do we believe in them only because of our innate love of order? Is man what he seems to the astronomer, a tiny lump of carbon and water impotently crawling on a small and unimportant planet? Or is he what he appears to Hamlet? Is he perhaps both at once? Is there a way of living that is noble and another that is base, or are all ways of living merely futile? If there is a way of living that is noble, in what does it consist, and how shall we achieve it? Must the good be eternal in order to deserve to be valued, or is it worth seeking even if the universe is inexorably moving towards death? Is there such a thing as wisdom or is what seems such merely the ultimate refinement of folly? To such questions no answer can be found in the laboratory. Theologies have professed to give answers, all to definite; but their definiteness causes modern minds to view them with suspicion. The studying of these questions, if not the answering of them, is the business of philosophy.

Why, then, you may ask, waste time on such insoluble problems? To this one may answer as a historian, or as an individual facing the terror of cosmic loneliness.

The answer of the historian, in so far as I am capable of giving it, will appear in the course of this work. Ever since men became capable of free speculation, their actions in innumerable important respects, have depended upon their theories as to the world and human life, as to what is good and what is evil. This is as true in the present day as at any former time. To understand an age or a nation, we must understand its philosophy, and to understand its philosophy we must ourselves be in some degree philosophers. There is here a reciprocal causation: the circumstances of men’s lives do much to determine their philosophy, but, conversely, their philosophy does much to determine their circumstances.

There is also, however, a more personal answer. Science tells us what we can know, but what we can know is little, and if we forget how much we cannot know we may become insensitive to many things of very great importance. Theology, on the other hand, induces a dogmatic belief that we have knowledge, where in fact we have ignorance, and by doing so generates a kind of impertinent insolence towards the universe. Uncertainty, in the presence of vivid hopes and fears, is painful, but must be endured if we wish to live without the support of comforting fairy tales. It is not good either to forget the questions that philosophy asks, or to persuade ourselves that we have found indubitable answers to them. To teach how to live without certainty, and yet without being paralyzed by hesitation, is perhaps the chief thing that philosophy, in our age, can still do for those who study it.

31. The purpose of philosophy is to

  • a. Reduce uncertainty and chaos
  • b. Help us to cope with uncertainty and ambiguity
  • c. Help us to find explanations for uncertainty
  • d. Reduce the terror of cosmic loneliness
  • e.Not Attempted

32. Based on this passage what can be concluded about the relation between philosophy and science?

  • a. The two are antagonistic.
  • b. The two are complimentary.
  • c. There is no relation between the two
  • d. Philosophy derives from science
  • e.Not Attempted

33.From reading the passage, what can be concluded about the profession of the author? He is most likely NOT to be a

  • a. Historian
  • b. Philosopher
  • c. Scientist
  • d. Theologian
  • e.Not Attempted

34. According to the author, which of the following statements about the nature of the universe must be definitely true?

  • a. The universe has unity
  • b. The universe has a purpose.
  • c. The universe is evolving towards a goal
  • d. None of the above
  • e.Not Attempted

Passage 3

If translated into English, most of the ways economists talk among themselves would sound plausible enough to poets, journalists, business people, and other thoughtful though noneconomical folk. Like serious talk anywhere among boat designers and baseball fans, say -- the talk is hard to follow when one has not made a habit of listening to it for a while. The culture of the conversation makes the words arcane. But the people in the unfamiliar conversation are not Martians. Underneath it (the economist’s favorite phrase) conversational habits are similar. Economics uses mathematical models and statistical tests and market arguments, all of which look alien to the literary eye. But looked at closely they are not so alien. They may be seen as figures of speech -- metaphors, analogies, and appeals to authority.

Figures of speech are not mere frills. They think for us. Someone who thinks of a market as an ‘invisible hand’ and the organization of work as a ‘production function’ and its coefficients as being ‘significant’ as an economist does, is giving the language a lot of responsibility. It seems a good idea to look hard at his language. If the economic conversation were found to depend a lot on its verbal forms, this would not mean that economics would be not a science, or just a matter of opinion, or some sort of confidence game. Good poets, though not scientists, are serious thinkers about symbols; good historians, though not scientists, are serious thinkers about data. Good scientists also use language. What is more, (though it remains to be shown) they use the cunning of language, without particularly meaning to. The language used is a social object and using language is a social act. It requires cunning (or, if you prefer, consideration) attention to the other minds present when one speaks.

The paying of attention to one’s audience is called ‘rhetoric’, a word that I later exercise hard. One uses rhetoric, of course, to warn of a fire in a theatre or to arouse the xenophobia of the electorate. This sort of yelling is the vulgar meaning of the word, like the president’s ‘heated rhetoric’ in a press conference or the ‘mere rhetoric’ to which our enemies stoop. Since the Greek flame was lit, though, the word has been used also in a broader and more amiable sense, to mean the study of all the ways of accomplishing things with language: inciting a mob to lynch the accused, to be sure, but also persuading readers of a novel that its characters breathe, or bringing scholars to accept the better argument and reject the worse.

The question is whether the scholar -- who usually fancies himself an announcer of ‘results’ or a stater of ‘conclusions’, free of rhetoric -- speaks rhetorically. Does he try to persuade? It would seems so. Language, I just said, is not a solitary accomplishment. The scholar doesn’t speak into the void, or to himself. He speaks to a community of voices. He desires to be heeded, praised, published, imitated, honored, en-Nobeled. These are the desires. The devices of language are the means.

Rhetoric is the proportioning of means to desires in speech. Rhetoric is an economics of language, the study of how scarce means are allocated to the insatiable desires of people to be heard. It seems on the face of it a reasonable hypothesis that economists are like other people in being talkers, who desire listeners. Why they go to the library or the laboratory as much as when they go to the office on the polls. The purpose here is to see if this is true, and to see if it is useful: to study the rhetoric of economic scholarship.

The subject is scholarship. It is not the economy, or the adequacy of economic theory as a description of the economy, or even mainly the economist’s role in the economy. The subject is the conversation economists have among themselves, for purposes of persuading each other that the interest elasticity of demand for investment is zero or that the money supply is controlled by the Federal Reserve.

Unfortunately, though, the conclusions are of more than academic interest. The conversations of classicists or of astronomers rarely affect the lives of other people. Those of economists do so on a large scale. A well known joke describes a May Day parade through Red Square with the usual mass of soldiers, guided missiles, rocket launchers. At last come rank upon rank of people in gray business suits. A bystander asks, “Who are those?” “Aha!” comes the reply, “those are economists -- you have no idea what damage they can do!” Their conversations do it.

35. According to the passage, which of the following is the best set of reasons for which one needs to “look hard” at an economist’s language?
(a) Economists accomplish a great deal through their language.
(b) Economics is an opinion-based subject.
(c) Economics has a great impact on other’s lives.
(d) Economics is damaging

  • a. a and b
  • b. c and d
  • c. a and c
  • d. b and d
  • e. Not Attempted

36. In the light of the definition of rhetoric given in the passage, which of the following will have the least element of rhetoric?

  • a. An election speech
  • b. An advertisement jingle
  • c. Dialogues in a play
  • d. Commands given by army officers.
  • e.Not Attempted

37. As used in the passage, which of the following is the closest meaning to the statement, "The culture of the conversation makes the words arcane?"

  • a. Economists belong to a different culture.
  • b. Only mathematicians can understand economists.
  • c. Economists tend to use terms unfamiliar to the lay person, but depend on familiar linguistic forms.
  • d. Economists use similes and adjectives in their analysis.
  • e.Not Attempted

38. As used in the passage, which of the following is the closest alternative to the word 'arcane?'

  • a. Mysterious
  • b. Secret
  • c. Covert
  • d. Perfidious
  • e.Not Attempted

39. Based on your understanding of the passage, which of the following conclusions would you agree with?

  • a. The geocentric and the heliocentric views of the solar system are equally tenable.
  • b. The heliocentric view is superior because of better rhetoric.
  • c. Both views use rhetoric to persuade
  • d. Scientists should not use rhetoric.
  • e.Not Attempted

Passage 4

There are a seemingly endless variety of laws, restriction, customs and traditions that affect the practice of abortion around the world. Globally, abortion is probably the single most controversial issue in the whole area of women's rights and family matters. It is an issue that inflames women's right groups, religious institutions and the self-proclaimed 'guardians' of public morality. The growing worldwide belief is that the right to control one's fertility is a basic human right. This has resulted in a worldwide trend towards liberalization of abortion laws. Forty percent of the world's population lives in countries where induced abortion is permitted on request. An additional 25 percent live in countries where it is allowed if the women's life would be endangered if she went to full term with her pregnancy. The estimate is that between 26 and 31 million legal abortions were performed in 1987. However, there were also between 10 and 22 million illegal abortions performed in that year.

Feminists have viewed the patriarchal control of women's bodies as one of the prime issues facing the contemporary women's movement. They observe that the definition and control of women's reproductive freedom have always been the province of men. Patriarchal religion, as manifest in Islamic fundamentalism, traditionalist Hindu practice, orthodox Judaism, and Roman Catholicism, has been an important historical contributory factor for this and continues to be an important presence in contemporary societies. In recent times, governments, usually controlled by men, have 'given' women the right to contraceptive use and abortion access when their countries were perceived to have an overpopulation problem. When these countries are perceived to be underpopulated, that right has been absent. Until the nineteenth century, a woman's rights to an abortion followed English common law; it could only be legally challenged if there was a 'quickening', when the first movements of the foetus could be felt. In 1800, drugs to induce abortions were widely advertised in local newspapers. By 1900, abortion was banned in every state except to save the life of the mother. The change was strongly influenced by the medical profession, which focussed its campaign ostensibly on health and safety issues for pregnant women and the sanctity of life. Its position was also a means of control of nonlicensed medical practitioners such as midwives and women healers who practiced abortion.

The anti-abortion campaign was also influenced by political considerations. The large influx of eastern and southern European immigrants with their large families was seen as threat to the population balance of the future United States. Middle and Upper class Protestants were advocates of abortion as a form of birth control. By supporting abortion prohibitions the hope was that these Americans would have more children and thus prevent the tide of immigrant babies from overwhelming the demographic characteristics of Protestant America.

The anti-abortion legislative position remained in effect in the United States through the first sixty-five years of the twentieth century. In the early 1960s, even when it was widely known that the drug thalidomide taken during pregnancy to alleviate anxiety was shown to contribute to the formation of deformed 'flipper-like' hands or legs of children, abortion was illegal in the United States. A second health tragedy was the severe outbreak of rubella during the same time period, which also resulted in major birth defects. These tragedies combined with a change of attitude towards a woman's right to privacy led a number of states to pass abortion-permitting legislation.

On one side of the controversy are those who call themselves 'pro-life'. They view the foetus as a human life rather than as an unformed complex of cells; therefore they hold to the belief that abortion is essentially murder of an unborn child. These groups cite both legal and religious reasons for their opposition to abortion. Pro-lifers point to the rise in legalized abortion figures and see this as morally intolerable. On the other side of the issue are those who call themselves 'pro-choice'. They believe that women, not legislators or judges, should have the right to decide whether and under what circumstances they will bear children. Pro-choicers are of the opinion that laws will not prevent women from having abortions and cite the horror stories of the past when many women died at the hands of 'backroom' abortionists and in desperate attempts to selfabort. They also observe that legalized abortion is especially important for rape victims and incest victims who became pregnant. They stress physical and mental health reasons why women should not have unwanted children.

To get a better understanding of the current abortion controversy, let us examine a very important work by Kristin Luker titled Abortion and the Politics of Motherhood. Luker argues that female pro-choice and pro-life activists hold different world views regarding gender, sex, and the meaning of parenthood. Moral positions on abortions are seen to be tied intimately to views on sexual behaviour, the care of children, family life, technology, and the importance of the individual. Luker identifies 'pro-choice' women as educated, affluent, and liberal. Their contrasting counterparts, 'pro-life' women, support traditional concepts of women as wives and mothers. It would be instructive to sketch out the differences in the world views of these two sets of women. Luker examines California, with its liberalized abortion law, as a case history. Public documents and newspaper accounts over a twenty-year period were analysed and over 200 interviews were held with both pro-life and pro-choice activists.

Luker found that pro-life and pro-choice activists have intrinsically different views with respect to gender. Pro-life women have a notion of public and private life. The proper place for men is in the public sphere of work; for women, it is the private sphere of the home. Men benefit through the nurturance of women; women benefit through the protection of men. Children are seen to be the ultimate beneficiaries of this arrangement by having the mother as a full-time loving parent and by having clear role models. Pro-choice advocates reject the view of separate spheres. They object to the notion of the home being the 'women's sphere'. Women's reproductive and family roles are seen as potential barriers to full equality. Motherhood is seen as a voluntary, not a mandatory or 'natural' role.

In summarizing her findings, Luker believes that women become activists in either of the two movements as the end result of lives that center around different conceptualizations of motherhood. Their beliefs and values are rooted to the concrete circumstances of their lives, their educations, incomes, occupations, and the different marital and family choices that they have made. They represent two different world views of women's roles in contemporary sociey and as such the abortion issues represents the battleground for the justification of their respective views.

40. According to your understanding of the author’s arguments which countries are more likely to allow abortion?

  • a. India and China
  • b. Australia and Mongolia
  • c. Cannot be inferred from the passage
  • d. Both (1) and (2)
  • e.Not Attempted

41. Which amongst these was NOT a reason for banning of abortions by 1900?

  • a. Medical professionals stressing the health and safety of women
  • b. Influx of eastern and southern European immigrants.
  • c. Control of unlicensed medical practitioners
  • d. A tradition of matriarchal control.
  • e.Not Attempted

42. A pro-life woman would advocate abortion if

  • a. The mother of an unborn child is suicidal
  • b. Bearing a child conflicts with a woman's career prospects
  • c. The mother becomes pregnant accidentally.
  • d. None of the above
  • e.Not Attempted

43. Pro-choice women object to the notion of the home being the 'women's sphere' because they believe

  • a. That the home is a 'joint sphere' shared between men and women
  • b. That reproduction is a matter of choice for women
  • c. That men and women are equal
  • d. Both b and c
  • e.Not Attempted

44. Two health tragedies affecting US society in the 1960s led to

  • a. A change in attitude to women's right to privacy
  • b. Retaining the anti-abortion laws with some exceptions.
  • c. Scrapping to anti-abortion laws
  • d. Strengthening of the pro-life lobby.
  • e.Not Attempted

45. Historically, the pro-choice movement has got support from, among others

  • a. Major patriarchal religions
  • b. Countries with low population density
  • c. Medical profession
  • d. None of fhe above
  • e.Not Attempted

Passage 5

The production of histories of India has become very frequent in recent years and may well call for some explanation. Why so many and why this one is particular? The reason is a two fold one: changes in the Indian scene requiring a reinterpretation of the facts and changes in attitudes of historians about the essential elements of Indian history. These two considerations are in addition to the normal fact of fresh information, whether in the form of archeological discoveries throwing fresh light on an obscure period or culture, or the revelations caused by the opening of archives or the release of private papers. The changes in the Indian scene are too obvious to need emphasis. Only two generations ago British rule seemed to most Indian as well as British observers likely to extend into an indefinite future; now there is a teenage generation which knows nothing of it. Changes in the attitudes of historians have occurred everywhere, changes in attitudes to the content of the subject as well as to particular countries, but in India there have been some special features. Prior to the British, Indian historiographers were mostly Muslims, who relied, as in the case of Sayyid Ghulam Hussain, on their own recollection of events and on information from friends and men of affairs. Only a few like Abu'I Fazl had access to official papers. These were personal narratives of events varying in value with the nature of the writer. The early British writers were officials. In the eighteenth century they were concerned with some aspect of Company policy, or, like Robert Orme in his Military Transactions, gave a straight narrative in what was essentially a continuation of the Muslim tradition. In the early nineteenth century the writers were still, with two notable exceptions, officials, but they were now engaged in chronicling, in varying moods of zest, pride, and awe, the rise of the British power in India to supremacy. The two exceptions were James Mill, with his critical attitude to the Company and John Marchman, the Baptist missionary. But they, like the officials, were anglo-centric in their attitude, so that the history of modern India in their hands came to be the history of the rise of the British in India.

The official school dominated the writing of Indian history until we get the first professional historian's approach, Ramsay Muir and P. E. Roberts in England and H.H. Dodwell in India. Then Indian historians trained in the English school joined in, of whom the most distinguished was Sir Jadunath Sarkar and the other notable writers: Surendranath Sen, Dr Radhakumud Mukerji, and Professor Nilakanta Sastri. They, it may be said, restored India to Indian history, but their bias was mainly political. Finally have come the nationalists who range from those who can find nothing good or true in the British to sophisticated historical philosophers like K.M. Panikker.

Along with types of historians with their varying bias have gone changes in the attitude to the content of Indian history. Here Indian historians have been influenced both by their local situation and by changes of thought elsewhere. It is in this field that this work can claim some attention since it seeks to break new ground, or perhaps to deepen a freshly turned furrow in the field of Indian history. The early official historians were content with the glamour and drama of political history from Plassey to the Mutiny, from Dupleix to the Sikhs. But when the raj was settled down, glamour departed from politics, and they turned to the less glorious but more solid ground of administration. Not how India was conquered but how it was governed was the theme of this school of historians. It found its archpriest in H.H. Dodwell, its priestess in Dame Lilian Penson, and its chief shrine in the Volume VI of the Cambridge History of India. Meanwhile in Britain other currents were moving which led historical study into the economic and social fields. R.C. Dutt entered the first of these currents with his Economic History of India to be followed more recently by the whole group of Indian economic historians. W.E. Moreland extended these studies to the Mughal Period. Social history is now being increasingly studied and there is also of course a school of nationalist historians who see modern Indian history in terms of the rise and the fulfillment of the national movement.

All these approaches have value, but all share in the quality of being compartmental. It is not enough to remove political history from its pedestal of being the only kind of history worth having if it is merely to put other types of history in its place. Too exclusive an attention to economic, social or administrative history can be as sterile and misleading as too much concentration on politics. A whole subject needs a whole treatment for understanding. A historian must dissect his subject into its elements and then fuse them together again into an integrated whole. The true history of a country must contain all the features just cited but must present them as parts of a single consistent theme.

46.Which of the following may be the closest in meaning to the statement "restored India to Indian history?"

  • a. Indian historians began writing Indian history
  • b. Trained 'historians began writing Indian history
  • c. Writing India-centric Indian history began
  • d. Indian history began to be written in India
  • e.Not Attempted

47. Which of the following is the closest implication of the statement "to break new ground, or perhaps to deepen a freshly turned furrow?"

  • a. Dig afresh or dig deeper
  • b. Start a new stream of thought or help establish a recently emerged perspective
  • c. Begin or conduct further work on existing archeological sites to unearth new evidence.
  • d. Begin writing a history free of any biases.
  • e.Not Attempted

48. Historian moved from writing political history to writing administrative history because

  • a. Attitude of the historian change
  • b. The 'Raj' was settled down
  • c. Politics did not retain its past glamour
  • d. Administrative history was based on solid ground
  • e.Not Attempted

49. According to the outline which of the following is not among the attitude of Indian historian of Indian origin?

  • a. Writing story as personal narrative
  • b. Writing history with political bias
  • c. Writing non-political history due to take of glamour
  • d. Writing history dissecting elements and integrating them again.
  • e.Not Attempted

50. In the table given below match the historian to the approaches taken bv them.
A. Administrative E. Robert Orme
B. Political F. H.H. Dodwell
C. Narrative G. Radhakumud Mukharji
D. Economic H. R C Dutt

  • a. A-F, B-G, C-E, D-H
  • b. A-G, B-F, C-E, D-H
  • c. A-E, B-F, C-G, D-H
  • d. A-F, B-H, C-E, D-G
  • e.Not Attempted

CAT2002 SECTION - MATHS

Instructions
1. The test comprises of 50 questions. You should complete the test within 40 minutes.
2. There is only one correct answer to each question.
3. All questions carry four marks each.
4. Each wrong answer will attract a penalty of one mark.

Directions Questions 1-10:
answer the questions independent of each other

1. How many numbers greater than 0 and less than a million can be formed with the digits 0, 7 and 8?

  • a. 486
  • b. 1086
  • c. 728
  • d. none of these
  • e.Not Attempted

2. If pqr = 1, the value of the expression is equal to [1/(1 + p + q -1)] + [1/(1 + p + r -1)] + [1/(1 + r + p -1)]

  • a. p + q + r
  • b. 1/ p + q + r
  • c. 1
  • d. p-1 +q-1 +r-1
  • e.Not Attempted

3. 76n - 66n, where n is an integer > 0, is divisible by

  • a. 13
  • b. 127
  • c. 559
  • d. none of these
  • e.Not Attempted

4. In how many ways is it possible to choose a white square and a black square on a chess board so that the squares must not lie in the same row or column?

  • a. 56
  • b. 896
  • c. 60
  • d. 768
  • e.Not Attempted

5.If u, v, w and m are natural numbers such that um + vm = wm, then one of the following is true.

  • a. m ≥ min (u, v, w)
  • b. m ≥ max (u, v, w)
  • c. m < min (u, v, w)
  • d. none of these
  • e.Not Attempted

6. Only a single rail track exists between station A and B on a railway line. One hour after the north bound super fast train N leaves station A for Station B, a south bound passenger train S reaches station A from station B. The speed of the super fast train is twice that of a normal express train E, while the speed of a passenger train S is half that of E. On a particular day N leaves for station B from

  • a. 1 : 3
  • b. 1 : 4
  • c. 1 : 5
  • d. 1 : 6
  • e.Not Attempted

7. Instead of walking along two adjacent sides of a rectangular field, a boy took a short cut along the diagonal and saved a distance equal to half the longer side. Then the ratio of the shorter side to the longer side is:

  • a. 1/2
  • b. 2/3
  • c. 1/4
  • d. 3/4
  • e.Not Attempted

8. The area of the triangle whose vertices are (a, a), (a + 1, a + 1), (a + 2, a) is:

  • a. a3
  • b. 1
  • c. 2a
  • d. 2 1/2
  • e.Not Attempted

9.A train approaches a tunnel AB. Inside the tunnel is a cat located at a point that is 3/8 of the distance AB measured from the entrance A. When the train whistles the cat runs. If the cat moves to the entrance of the tunnel, A, the train catches the cat exactly at the entrance. If the cat moves to the exit, B, the train catches the cat at exactly the exit. The speed of the train is greater than the speed of the cat by what order?

  • a. 3 : 1
  • b. 4 : 1
  • c. 5 :1
  • d. none of these
  • e.Not Attempted

10. Six persons are playing a card game. Suresh is facing Raghubir who is to the left of Ajay and to the right of Pramod. Ajay is to the left of Dhiraj. Yogendra is to the left of Pramod. If Dhiraj exchanges his seat with Yogendra and Pramod exchanges with Raghubir, who will be sitting to the left of Dhiraj?

  • a. Yogendra
  • b. Raghubir
  • c. Suresh
  • d. Ajay
  • e.Not Attempted

DIRECTIONS for questions 11 and 12:
Answer these questions based on the information given below.
Each of the 11 letters A, H, I, M, O, T, U, V, W, X and Z appears same when looked at in a mirror. They are called symmetric letters. Other letters in the alphabet are symmetric letters.

How many four-letter computer passwords can be formed using only the symmetric letters (no repetition allowed)? 11.

  • a. 7920
  • b. 330
  • c. 14640
  • d. 419430
  • e.Not Attempted

12. How many three-letter computer passwords can be formed (no repetition allowed) with at least one symmetric letter?

  • a. 990
  • b. 2730
  • c. 12870
  • d. 15600
  • e.Not Attempted

DIRECTIONS for questions 13 to 15:
Answer the questions independent of each other

13. After the division of a number successively by 3, 4 and 7, the remainders obtained are 2, 1 and 4 respectively. What will be the remainder if 84 divides the same number?

  • a. 80
  • b. 76
  • c. 41
  • d. 53
  • e.Not Attempted

14. Three pieces of cakes of weight 4 1/2 lbs, 6 3/4 lbs and 7 1/5 lbs respectively are to be divided into parts of equal weights. Further, each part must be as heavy as possible. If one such part is served to each guest, then what is the maximum number of guests that could be entertained?

  • a. 54
  • b. 72
  • c. 20
  • d. none of these
  • e.Not Attempted

15. At a bookstore. "MODERN BOOK STORE" is flashed using neon lights. The words are individually flashed at intervals of 2 1/2, 4 1/4, 5 1/8 seconds respectively and each word is put off after a second. The least time after which the full name of the bookstore can be read again is:

  • a. 49.5 seconds
  • b. 73.5 seconds
  • c. 1744.5 seconds
  • d. 855 seconds
  • e. Not Attempted

16.The number of real roots of the equation [A2/x + B2/x-1 = 1] where A and B are real numbers not equal to zero simultaneously is

  • a. None
  • b. 1
  • c. 2
  • d. 1 or 2
  • e.Not Attempted

17. When 2256 is divided by 17 the remainder would be

  • a. 1
  • b. 16
  • c. 14
  • d. none of these
  • e.Not Attempted

18. 10 straight lines, no two of which are parallel and no three of which pass through any common point, are drawn on a plane. The total number of regions (including finite and infinite regions) into which the plane would be divided by the lines is

  • a. 56
  • b. 255
  • c. 1024
  • d. not unique
  • e.Not Attempted

19. Suppose, for any real number x, [x] denotes the greatest integer less than or equal to x. Let L(x, y) = [x] + [y] + [x + y] and R(x, y) = [2x] + [2y]. Then it's impossible to find any two positive real numbers x and y for which

  • a. L(x, y) = R(x, y)
  • b. L(x, y) not equal R(x, y)
  • c. L(x, y) < R(x, y)
  • d. L(x, y) > R(x, y)
  • e.Not Attempted

20. A child was asked to add first few natural numbers (that is, 1 + 2 + 3 ) so long his patience permitted. As he stopped, he gave the sum as 575. When the teacher declared the result wrong the child discovered he had missed one number in the sequence during addition. The number he missed was:

  • a. less than 10
  • b. 10
  • c. 15
  • d. more than 15
  • e.Not Attempted

21. A car rental agency has the following terms. If a car is rented for 5 hours or less the charge is 60 per hour or Rs.12 per kilometer whichever is more. On the other hand, if the car is rented for more than 5 hours, the charge is Rs.50 per hour or Rs.7.50 per Kilometer whichever is more. Akil rented a car from this agency, drove it for 30 kilometers and ended up paying Rs.300. For how many hours did herent the Car?

  • a. 4
  • b. 5
  • c.6
  • d. none of these
  • e.Not Attempted

22.Amol was asked to calculate the arithmetic mean of ten positive integers each of which had two digits. By mistake, he interchanged the two digits, say a and b, in one of these ten integers. As a result, his answer for the arithmetic mean was 1.8 more than what it should have been. Then b - a equals

  • a. 1
  • b. 2
  • c. 3
  • d. none of these
  • e.Not Attempted

23. If x2 + 5y2 + z2 = 2y (2x + z) then which of the following statements are necessarily true?
A. x=2y. B. x=2z C.2x=z.

  • a. Only A
  • b.Only B and C
  • c. Only A and B
  • d. none of these
  • e.Not Attempted

24. Let S denote the infinite sum 2 + 5x + 9x2 + 14x3 + 20x4 + ..., where |x| < 1 and the coefficient of xn-1 is n (n + 3), (n = 1, 2, ...). Then S equals

  • a. (2-x)/(1-x)3
  • b. (2-x)/(1+x)3
  • c. (2+x)/(1-x)3
  • d. (2+x)/(1+x)3
  • e.Not Attempted
25. Shyam visited Ram on vacation. In the mornings, they both would go for yoga. In the evenings they Would play tennis. To have more fun, they indulge only in one actively per day, i.e., either they went for yoga or played tennis each day. There were days when they were lazy and stayed home all day long. There were 24 mornings when they did nothing, 14 evenings when they stayed at home, and a total of 22 days when they did yoga or played tennis. For how many days Shyam stayed with Ram?
  • a. 32
  • b. 24
  • c. 30
  • d. none of these
  • e.Not Attempted

DIRECTIONS for questions 26 and 27:
Answer these questions based on the information given below.

A boy is asked to put in a basket one mango when ordered 'One', one orange when ordered 'Two', one apple when ordered 'Three'and is asked to take out from the basket one mango and an orange when ordered 'Four'. A sequence of orders is given as:

1 2 3 3 2 1 4 2 3 1 4 2 2 3 3 1 4 1 1 3 2 3 4

26. How many total oranges were in the basket at the end of the above sequence?

  • a. 1
  • b. 4
  • c. 3
  • d. 2
  • e.Not Attempted

27. How many total fruits will be in the basket at the end of the above order sequence?

  • a. 9
  • b. 8
  • c. 11
  • d. 10
  • e.Not Attempted

DIRECTIONS for questions 28 to 40:
Answer the questions independent of each other

28. A rich merchant had collected many gold coins. He did not want anybody to know about them. One day, his wife asked, "How many gold coins do we have?" After pausing a moment, he replied, "Well! If I divide the coins into two unequal numbers, then 48 times the difference between the two numbers equals the difference between the squares of the two numbers." The wife looked puzzled. Can you help the merchant's wife by finding out how many coins the merchant has?

  • a. 96
  • b. 53
  • c. 43
  • d. none of these
  • e.Not Attempted

29.On a straight road XY, 100 metres long, five heavy stones are placed two metres apart beginning at the end X. A worker, starting at X, has to transport all the stones to Y, by carrying only one stone at a time. The minimum distance he has to travel (in metres) is:

  • a. 472
  • b. 422
  • c. 744
  • d. 844
  • e.Not Attempted

30.Four horses are tethered at four corners of a square plot of side 14 metres (m) so that the adjacent horses can reach one another. There is a small circular pond of area 20 m2 at the centre. The area left ungrazed is:

  • a. 22m2
  • b. 42m2
  • c. 84m2
  • d. 168m2
  • e.Not Attempted

31. If f (x) = log { (1 + x) / (1 - x) }, then f (x) + f (y) is:

  • a. f (x + y)
  • b. f { (x + y) / (1 + xy) }
  • c. (x + y) f { 1/ (1 + xy) }
  • d. f (x) + f (y) / (1 + xy)
  • e.Not Attempted

32. The length of the common chord of two circles of radii 15 cm and 20 cm, whose centres are 25 cm apart, is (cm):

  • a. 24
  • b. 25
  • c. 15
  • d. 20
  • e.Not Attempted

33.In a triangle ABC, the internal bisector of the angle A meets BC at D. If AB = 4, AC = 3 and A = 600, then length of AD is:

  • a. 2 √3
  • b. 12 √(3/7)
  • c. 15 √(3/8)
  • d. 6 √(3/7)
  • e.Not Attempted

34. If there are 10 positive real numbers n1 < n2 < n3 ...< n10. How many triplets of these numbers (n1, n2, n3), (n2, n3, n4), ... can be generated such that in each that in each triplet the first number is always less than the second number, and the second number is always less than the third number?

  • a. 45
  • b. 90
  • c. 120
  • d. 180
  • e.Not Attempted

35. 3 small pumps and a large pump are filling a tank. Each of the three small pumps works at 2/3rd the rate of the large pump. If all 4 pumps work at the same time, they should fill the tank in what fraction of the time that it would have taken the large pump alone?

  • a. 4/7
  • b. 1/3
  • c. 2/3
  • d. 3/4
  • e. Not Attempted

36. Davji Shop sells samosas in boxes of different sizes. The samosas are priced at Rs.2 per samosa up to 200 samosas. For every additional 20 samosas, the price of the whole lot goes down by 10 paise per samosa. What should be the maximum size of the box that would maximize the revenue?

  • a. 240
  • b. 300
  • c. 400
  • d. none of these
  • e.Not Attempted

37. It takes 6 technicians a total of 10 hours to build a new server from Direct Computer, with each working at the same rate. If six technicians start to build the server at 11.00 AM, and one technician per hour is added beginning at 5.00 PM, at what time will the server be complete?

  • a. 6:40pm
  • b. 7:00pm
  • c. 7:20pm
  • d. 8pm
  • e.Not Attempted

38. In the above figure, ACB is a right angled triangle. CD is the altitude. Circles are inscribed within the triangle ACD, BCD. P and Q are the centres of the circles. The distance PQ is

  • a. 5
  • b. √50
  • c. 7
  • d. 8
  • e.Not Attempted

39. Three travelers are sitting around a fire, and are about to eat a meal. One of them has five small loaves of bread, the second has three small loaves of bread. The third has no food, but has eight coins. He offers to pay for some bread. They agree to share the eight loaves equally among the three travelers, and the third traveler will pay eight coins for his share of the eight loaves. All loaves were the same size. The second traveler (who had three loaves) suggests that he be paid three coins and that the first traveler be paid five coins. The first traveler says that he should get more than five coins. How much the first traveler should get?

  • a. 5
  • b. 7
  • c. 1
  • d. none of these
  • e.Not Attempted

40. A piece of string is 40 centimeters long. It is cut into three pieces. The longest piece is 3 times as long as the middle-sized piece and the shortest piece is 23 centimeters shorter than the longest piece. Find the length of the shortest piece.

  • a. 27
  • b. 5
  • c. 4
  • d. 9
  • e.Not Attempted

DIRECTIONS for questions 41 to 42:
Answer these questions based on the following diagram.
In the diagram below: LABC = 900 = LDCH = LDOE =LEHK = LFKL = LGLM =LLMN.
AB = BC = 2CH = 2CD = EH = FK = 2HK = 4KL = 2 LM = MN

41.The magnitude of angle(LFGO)

  • a. 30°
  • b. 45°
  • c. 60°
  • d. none of these
  • e.Not Attempted

42. The ratio of the areas of the two quadrangles ABCD and DEFG is

  • a. 1:2
  • b. 2:1
  • c. 12:7
  • d. none of these
  • e.Not Attempted

DIRECTIONS for questions 43 to 50:
Answer the questions independent of each other

43. Mayank, Mirza, Little and Jaspal bought a motorbike for $60.00. Mayank paid one half of the sum of the amounts paid by the other boys. Mirza paid one third of the sum of the amounts paid by the other boys; and Little paid one fourth of the sum of the amounts paid by the other boys. How much did Jaspal have to pay?

  • a. 15
  • b. 13
  • c. 17
  • d. none of these
  • e.Not Attempted

44. The owner of a local jewellery store hired 3 watchmen to guard his diamonds, but a thief still got in and stole some diamonds. On the way out, the thief met each watchman, one at a time. To each he gave 1/2 of the diamonds he had then, and 2 more besides. He escaped with one diamond. How many did he steal originally?

  • a. 40
  • b. 36
  • c. 25
  • d. none of these
  • e.Not Attempted

45. Neeraj has agreed to mow the front lawn, which is a 20m by 40m rectangle. The mower mows a 1 m wide strip. If Neeraj starts at one corner and mows around the lawn toward the center, about how many times would he go round before he has mowed half the lawn?

  • a. 2.5
  • b. 3.5
  • c. 3.8
  • d. 4.0
  • e.Not Attempted

46. If x, y and z are real numbers such that: x + y + z =5 and xy + yz + zx = 3, what is the largest value that x can have?

  • a. 5/3
  • b. √19
  • c. 13/3
  • d. none of these
  • e.Not Attempted

47. The nth element of a series is represented as Xn = (-1)n Xn-1 If X0 = x and x > 0 then the following is always true

  • a. Xn is positive if n is even
  • b. Xn is positive if n is odd
  • c. Xn is negative if n is even
  • d. None of these
  • e.Not Attempted

48. Number S is obtained by squaring the sum of digits of a two digit number D. If difference between S and D is 27, then the two digit number D is:

  • a. 24
  • b. 54
  • c. 34
  • d. 45
  • e.Not Attempted

49. On a 20 km tunnel connecting two cities A and B there are three gutters. The distance between gutter 1 and 2 is half the distance between gutter 2 and 3. The distance from city A to its nearest gutter, gutter 1 is equal to the distance of city B from gutter 3. On a particular day the hospital in city A receives information that an accident has happened at the third gutter. The victim can be saved only if an operation is started within 40 minutes. An ambulance started from city A at 30 km/hr and crossed the first gutter after 5 minutes. If the driver had doubled the speed after that, what is the maximum amount of time the doctor would get to attend the patient at the hospital?
Assume 1 minute is elapsed for taking the patient into and out of the ambulance.

  • a. 4 minutes
  • b. 2.5 minutes
  • c. 1.5 minutes
  • d. Patient died before reaching the hospital
  • e.Not Attempted

50. In the figure given below, ABCD is a rectangle.
The area of the isosceles right triangle ABE = 7 cm2. (FC) = 3(BE).
The area of ABCD (in cm2) is:

  • a. 21
  • b. 28
  • c. 42
  • d. 56
  • e.Not Attempted

CAT2003 SECTION - ENGLISH

Instructions
1. The test comprises of 50 questions. You should complete the test within 40 minutes.
2. There is only one correct answer to each question.
3. All questions carry four marks each.
4. Each wrong answer will attract a penalty of one mark.

Directions Questions 1-30:

Each of the five passages given below is followed by five questions. Choose the best answer to each question.

PASSAGE I

At the heart of the enormous boom in wine consumption that has taken place in the English-speaking world over the last two decades or so is a fascinating, happy paradox. In the days when wine was exclusively the preserve of a narrow cultural elite, bought either at auctions or from gentleman wine merchants in wing collars and bow ties, to be stored in rambling cellars and decanted to order by one's butler, the ordinary drinker didn't get a look-in. Wine was considered a highly technical subject, in which anybody without the necessary ability could only fall flat on his or her face in embarrassment. It wasn't just that you needed a refined aesthetic sensibility for the stuff if it wasn't to be hopelessly wasted on you. It required an intimate knowledge of what came from where, and what it was supposed to taste like.

Those were times, however, when wine appreciation essentially meant a familiarity with the great French classics, with perhaps a smattering of other wines -- like sherry and port. That was what the wine trade dealt in. These days, wine is bought daily in supermarkets and high-street chains to be consumed that evening, hardly anybody has a cellar to store it in and most don't even possess a decanter. Above all, the wines of literally dozens of countries are available on our market. When a supermarket offers its customers a couple of fruity little numbers from Brazil, we scarcely raise an eyebrow.

It seems, in other words, that the commercial jungle that wine has now become has not in the slightest deterred people from plunging adventurously into the thickets in order to taste and see. Consumers are no longer intimidated by the thought of needing to know their Pouilly-Fume from their Pouilly-Fuisse, just at the very moment when there is more to know than ever before.

The reason for this new mood of confidence is not hard to find. It is on every wine label from Australia, New Zealand, South Africa and the United States: the name of the grape from which the wine is made. At one time that might have sounded like a fairly technical approach in itself. Why should native English-speakers know what Cabernet Sauvignon or Chardonnay were? The answer lies in the popularity that wines made from those grape varieties now enjoy. Consumers effectively recognize them as brand names, and have acquired a basic lexicon of wine that can serve them even when confronted with those Brazilian upstarts.

In the wine heartlands of France, they are scared to death of that trend -- not because they think their isn't as good as the best from California or South Australia (what French winemaker will ever admit that?) but because they don't traditionally call their wines Cabernet Sauvignon or Chardonnay. They call them Chateau Ducru-Beaucaillou or Corton-Charlemagne, and they aren't about to change. Some areas, in the middle of southern France, have now produced a generation of growers using the varietal names on their labels and are tempting consumers back to French wine. It will be an uphill struggle, but there is probably no other way if France is to avoid simply becoming a specialty source of old-fashioned wines for old-fashioned connoisseurs.

Wine consumption was also given a significant boost in the early 1990s by the work of Dr. Serge Renaud, who has spent many years investigating the reasons for the uncannily low incidence of coronary heart disease in the south of France. One of his major findings is that the fat-derived cholesterol that builds up in the arteries and can eventually lead to heart trouble, can be dispersed by the tannins in wine. Tannin is derived from the skins of grapes, and is therefore present in higher levels in red wines, because they have to be infused with their skins to attain the red colour. That news caused a huge upsurge in red wine consumption in the United States. It has not been accorded the prominence it deserves in the UK, largely because the medical profession still sees all alcohol as a menace to health, and is constantly calling for it to be made prohibitively expensive. Certainly the manufacturers of anticoagulant drugs might have something to lose if we all got the message that we would do just as well by our hearts by taking half a bottle of red wine every day!

1. The tone that the author uses while asking "What French winemaker will ever admit that?" is best described as

  • a. caustic.
  • b. satirical.
  • c. critical.
  • d. hypocritical.
  • e.Not Attempted

2. What according to the author should the French do to avoid becoming a producer of merely old-fashioned wines?

  • a. Follow the labelling strategy of the English-speaking countries.
  • b. Give their wines English names.
  • c. Introduce fruity wines as Brazil has done.
  • d. Produce the wines that have become popular in the English-speaking world.
  • e.Not Attempted

3. The development which has created fear among winemakers in the wine heartlands of France is the

  • a. tendency not to name wines after the grape varieties that are used in the wines.
  • b. 'education' that consumers have derived from wine labels from English-speaking countries.
  • c. new generation of local winegrowers who use labels that show names of grape varieties.
  • d. ability of consumers to understand a wine's qualities when confronted with "Brazilian upstarts".
  • e.Not Attempted

4. Which one of the following, if true, would provide most support for Dr. Renaud's findings about the effect of tannins?

  • a. A survey showed that film celebrities based in France have a low incidence of coronary heart disease.
  • b. Measurements carried out in southern France showed red wine drinkers had significantly higher levels of coronary heart incidence than white wine drinkers did.
  • c. Data showed a positive association between sales of red wine and incidence of coronary heart disease.
  • d. Long-term surveys in southern France showed that the incidence of coronary heart disease was significantly lower in red wine drinkers than in those who did not drink red wine.
  • e.Not Attempted

5. Which one of the following CANNOT be reasonably attributed to the labelling strategy followed by wine producers in Englishspeaking countries?

  • a. Consumers buy wines on the basis of their familiarity with a grape variety's name.
  • b. Even ordinary customers now have more access to technical knowledge about wine.
  • c. Consumers are able to appreciate better quality wines.
  • d. Some non-English speaking countries like Brazil indicate grape variety names on their labels.
  • e.Not Attempted

PASSAGE II

Right through history, imperial powers have clung to their possessions to death. Why, then, did Britain in 1947 give up the jewel in its crown, India? For many reasons. The independence struggle exposed the hollowness of the white man's burden. Provincial self rule since 1935 paved the way for full self rule. Churchill resisted independence, but the Labour government of Atlee was anti-imperialist by ideology. Finally, the Royal Indian ­Navy mutiny in 1946 raised fears of a second Sepoy mutiny, and convinced British waverers that it was safer to withdraw gracefully. But politico-military explanations are not enough. The basis of empire was always money. The end of empire had much to do with the fact that British imperialism had ceased to be profitable. World War II left Britain victorious but deeply indebted, needing Marshall Aid and loans from the World Bank. This constitutes a strong financial case for ending the no-longer-profitable empire.

Empire building is expensive. The US is spending one billion dollars a day in operations in Iraq that fall well short of full-scale imperialism. Through the centuries, empire building was costly, yet constantly undertaken because it promised high returns. The investment was in armies and conquest. The returns came through plunder and taxes from the conquered.

No immorality was attached to imperial loot and plunder. The biggest conquerors were typically revered (hence titles like Alexander the Great, Akbar the Great, and Peter the Great). The bigger and richer the empire, the plunderer was admired. This mindset gradually changed with the rise of new ideas about equality and governing for the public good, ideas that culminated in the French and American revolutions. Robert Clive was impeached for making a little money on the side, and so was Warren Hastings. The white man's burden came up as a new moral rationale for conquest. It was supposedly for the good of the conquered. This led to much muddled hypocrisy. On the one hand, the empire needed to be profitable. On the other hand, the white man's burden made brazen toot impossible.

An additional factor deterring loot was the 1857 Sepoy Mutiny. Though crushed, it reminded the British vividly that that they were a tiny ethnic group who could not rule a gigantic subcontinent without the support of important locals. After 1857, the British stopped annexing one princely state after another, and instead treated the princes as allies. Land revenue was fixed in absolute terms, partly to prevent local unrest and partly, to promote the notion of the white man's burden. The empire proclaimed itself to be a protector of the Indian peasant against exploitation by Indian elites. This was denounced as hypocrisy by nationalists like Dadabhoy Naoroji in the 19th century, who complained that land taxes led to an enormous drain from India to Britain. Objective calculations by historians like Angus Maddison suggest a drain of perhaps 1.6 percent of Indian Gross National Product in the 19th century. But land revenue was more or less fixed by the Raj in absolute terms, and so its real value diminished rapidly with inflation in the 20th century. By World War II, India had ceased to be a profit centre for the British Empire.

Historically, conquered nations paid taxes to finance fresh wars of the conqueror. India itself was asked to pay a large sum at the end of World War I to help repair Britain's finances. But, as shown by historian Indivar Kamtekar, the independence movement led by Gandhiji changed the political landscape, and made mass taxation of India increasingly difficult. By World War II, this had become politically impossible. Far from taxing India to pay for World War II, Britain actually began paying India for its contribution of men and goods. Troops from white dominions like Australia, Canada and New Zealand were paid for entirely by these countries, but Indian costs were shared by the British government. Britain paid in the form of non-convertible sterling balances, which mounted swiftly. The conqueror was paying the conquered, undercutting the profitability on which all empire is founded. Churchill opposed this, and wanted to tax India rather than owe it money. But he was overruled by India hands who said India would resist payment, and paralyze the war effort. Leo Amery, Secretary of State for India, said that when you are driving in a taxi to the station to catch a life-or-death train, you do not loudly announce that you have doubts whether to pay the fare. Thus, World War II converted India from a debtor to a creditor with over one billion pounds in sterling balances. Britain, meanwhile, became the biggest debtor in the world. It's not worth ruling over people you are afraid to tax.

6. Why didn't Britain tax India to finance its World War II efforts?

  • a.Australia, Canada and New Zealand had offered to pay for Indian troops.
  • b. India had already paid a sufficiently large sum during World War I.
  • c.It was afraid that if India refused to pay, Britain's war efforts would be jeopardised.
  • d. The British empire was built on the premise that the conqueror pays the conquered.
  • e.Not Attempted

7. What was the main lesson the British learned from the Sepoy Mutiny of 1857?

  • a. That the local princes were allies, not foes.
  • b. That the land revenue from India would decline dramatically.
  • c. That the British were a small ethnic group.
  • d. That India would be increasingly difficult to rule.
  • e.Not Attempted

8. Which of the following was NOT a reason for the emergence of the 'white man's burden' as a new rationale for empire-building in India?

  • a. The emergence of the idea of the public good as an element of governance.
  • b. The decreasing returns from imperial loot and increasing costs of conquest.
  • c. The weakening of the immorality attached to an emperor's looting behaviour.
  • d. A growing awareness of the idea of equality among peoples.
  • e.Not Attempted

9. Which of the following best captures the meaning of the 'white man's burden', as it is used by the author?

  • a. The British claim to a civilizing mission directed at ensuring the good of the natives.
  • b. The inspiration for the French and American revolutions.
  • c. The resource drain that had to be borne by the home country's white population.
  • d. An imperative that made open looting of resources impossible.
  • e.Not Attempted

10. Which one of the following best expresses the main purpose of the author?

  • a. To present the various reasons that can lead to the collapse of an empire and the granting of independence to the subjects of an empire.
  • b. To point out the critical role played by the 'white man's burden' in making a colonizing power give up its claims to native possessions.
  • c. To highlight the contradictory impulse underpinning empire building which is a costly business but very attractive at the same time.
  • d.To illustrate how erosion of the financial basis of an empire supports the granting of independence to an empire's constituents.
  • e.Not Attempted

PASSAGE III

The controversy over genetically modified food continues unabated in the West. Genetic modification (GM) is the science by which the genetic material of a plant is altered, perhaps to make it more resistant to pests or killer weeds or to enhance its nutritional value. Many food biotechnologists claim that GM will be a major contribution of science to mankind in the 21st century. On the other hand, large numbers of opponents, mainly in Europe, claim that the benefits of GM are a myth propagated by multinational corporations to increase their profits, that they pose a health hazard, and have therefore called for governments to ban the sale of genetically-modified food.

­The anti-GM campaign has been quite effective in Europe, with several European Union member countries imposing a virtual ban for five years over genetically-modified food imports. Since the genetically-modified food industry is particularly strong in the United States of America, the controversy also constitutes another chapter in the US-Europe skirmishes which have become particularly acerbic after the US invasion of Iraq.

To a large extent, the GM controversy has been ignored in the Indian media, although Indian biotechnologists have been quite active in GM research. Several groups of Indian biotechnologists have been working on various issues connected with crops grown in India. One concrete achievement which has recently figured in the news is that of a team led by the former vice-chancellor of Jawaharlal Nehru University, Asis Datta -- it has successfully added an extra gene to potatoes to enhance the protein content of the tuber by at least 30 percent. Not surprisingly, the new potato has been called the protato. The protato is now in its third year of field trials. It is quite likely that the GM controversy will soon hit the headlines in India since a spokesperson of the Indian Central government has recently announced that the government may use the protato in its midday meal programme for schools as early as next year.

Why should "scientific progress", with huge potential benefits to the poor and malnourished, be so controversial? The anti-GM lobby contends that pernicious propaganda has vastly exaggerated the benefits of GM and completely evaded the costs which will have to be incurred if the genetically-modified food industry is allowed to grow unchecked. In particular, they allude to different types of costs.

This group contends that the most important potential cost is that the widespread distribution and growth of genetically-modified food will enable the corporate world (alias the multinational corporations - MNCs) to completely capture the food chain. A "small" group of biotech companies will patent the transferred genes as well as the technology associated with them. They will then buy up the competing seed merchants and seed-breeding centres, thereby controlling the production of food at every possible level. Independent farmers, big and small, will be completely wiped out of the food industry. At best, they will be reduced to the status of being sub-contractors.

This line of argument goes on to claim that the control of the food chain will be disastrous for the poor since the MNCs, guided by the profit motive, will only focus on the high-value food items demanded by the affluent. Thus, in the long run, the production of basic staples which constitute the food basket of the poor will taper off. However, this vastly overestimates the power of the MNCs. Even if the research promoted by them does focus on the high value food items, much of biotechnology research is also funded by governments in both developing and developed countries. Indeed, the protato is a by-product of this type of research. If the protato passes the field trials, there is no reason to believe that it cannot be marketed in the global potato market. And this type of success story can be repeated with other basic food items.

The second type of cost associated with the genetically-modified food industry is environmental damage. The most common type of "genetic engineering" involves gene modification in plants designed to make them resistant to applications of weed-killers. This then enables farmers to use massive dosages of weed killers so as to destroy or wipe out all competing varieties of plants in their fields. However, some weeds through genetically modified pollen contamination may acquire resistance to a variety of weed-killers. The only way to destroy these weeds is through the use of ever-stronger herbicides which are poisonous and linger on in the environment. 11. The author doubts the anti-GM lobby's contention that MNC control of the food chain will be disastrous for the poor because

  • a. MNCs will focus on high-value food items.
  • b. MNCs are driven by the motive of profit maximization.
  • c. MNCs are not the only group of actors in genetically modified food research.
  • d. economic development will help the poor buy MNC-produced food.
  • e.Not Attempted

12. Using the clues in the passage, which of the following countries would you expect to be in the forefront of the anti-GM campaign?

  • a. USA and Spain.
  • b. India and Iraq.
  • c. Germany and France.
  • d. Australia and New Zealand.
  • e.Not Attempted

13. Genetic modification makes plants more resistant to killer weeds. However, this can lead to environmental damage by

  • a. wiping out competing varieties of plants which now fall prey to killer weeds.
  • b. forcing application of stronger herbicides to kill weeds which have become resistant to weak herbicides.
  • c. forcing application of stronger herbicides to keep the competing plants weed-free.
  • d. not allowing growth of any weeds, thus reducing soil fertility.
  • e.Not Attempted

14. According to the passage, biotechnology research

  • a. is of utility only for high value food items.
  • b. is funded only by multinational corporations.
  • c. allows multinational corporations to control the food basket of the poor.
  • d. addresses the concerns of rich and poor countries.
  • e.Not Attempted

15. Which of the following about the Indian media's coverage of scientific research does the passage seem to suggest?

  • a. Indian media generally covers a subject of scientific importance when its mass application is likely.
  • b. Indian media's coverage of scientific research is generally dependent on MNCs interests.
  • c. Indian media, in partnership with the government, is actively involved in publicizing the results of scientific research.
  • d. Indian media only highlights scientific research which is funded by the government.
  • e. Not Attempted

PASSAGE IV

Social life is an outflow and meeting of personality, which means that its end is the meeting of character; temperament, and sensibility, in which our thoughts and feelings, and sense perceptions are brought into play at their lightest and yet keenest.

This aspect, to my thinking, is realized as much in large parties composed of casual acquaintances or even strangers, as in intimate meetings of old friends. I am not one of those superior persons who hold cocktail parties in contempt, looking upon them as barren or at best as very tryingly kaleidoscopic places for gathering, because of the strangers one has to meet in them; which is no argument, for even our most intimate friends must at one time have been strangers to us. These large gatherings will be only what we make of them - if not anything better, they can be as good places to collect new friends from as the slave-markets of Istanbul were for beautiful slaves or New Market for race horses.

But they do offer more immediate enjoyment. For one thing, in them one can see the external expression of social life in appearance and behaviour at its widest and most varied -- where one can admire beauty of body or air, hear voices remarkable either for sweetness or refinement, look on elegance of clothes or deportment. What is more, these parties are schools for training in sociability, for in them we have to treat strangers as friends. So, we see social sympathy in widest commonalty spread, or at least should. We show an atrophy of the natural human instinct of getting pleasure and happiness out of other human beings if we cannot treat strangers as friends for the moment. And I would go further and paraphrase Pater to say that not to be able to discriminate every moment some passionate attitude in those about us, even when we meet them casually, is on this short day of frost and sun which our life is, to sleep before evening.

So, it will be seen that my conception of social life is modest, for it makes no demands on what we have, though it does make some on what we are. Interest, wonder, sympathy, and love, the first two leading to the last two, are the psychological prerequisites for social life; and the need for the first two must not be underrated. We cannot make the most even of our intimate social life unless we are able to make strangers of our oldest friends everyday by discovering unknown areas in their personality, and transform them into new friends. In sum, social life is a function of vitality.

It is tragic, however, to observe that it is these very natural springs of social life which are drying up among us. It is becoming more and more difficult to come across fellow-feeling for human beings as such in our society and in all its strata. In the poor middle class, in the course of all my life, I have hardly seen any social life properly so-called. Not only has the grinding routine of making a living killed all desire for it in them, it has also generated a standing mood of peevish hostility to other human beings. Increasing economic distress in recent years has infinitely worsened this state of affairs, and has also brought a sinister addition -- class hatred. This has become the greatest collective emotional enjoyment of the poor middle class, and indeed they feel most social when they form a pack, and snarl or howl at people who are better off than they.

Their most innocent exhibition of sociability is seen when they spill out from their intolerable homes into the streets and bazaars. I was astonished to see the milling crowds in the poor suburbs of Calcutta. But even there a group of flippant young loafers would put on a conspiratorial look if they saw a man in good clothes passing by them either on foot or in a car. I had borrowed a car from a relative to visit a friend in one of these suburbs, and he became very anxious when I had not returned before dusk. Acid and bombs, he said, were thrown at cars almost every evening in that area. I was amazed. But I also know as a fact that my brother was blackmailed to pay five rupees on a trumped up charge when passing in a car through one such locality.

The situation is differently inhuman, but not a whit more human, among the well-to-do. Kindliness for fellow human beings has been smothered in them, taken as a class, by the arrogance of worldly position, which among the Bengalis who show this snobbery is often only a third-class position.

16. The word 'they' in the first sentence of the third paragraph refers to

  • a. Large parties consisting of casual acquaintances and strangers.
  • b. Intimate meetings of old friends.
  • c. New friends.
  • d. Both 1 & 2.
  • e.Not Attempted

17. In this passage the author is essentially

  • a. showing how shallow our social life is.
  • b. poking fun at the lower middle class people who howl at better off people.
  • c. lamenting the drying up of our real social life.
  • d. criticizing the upper class for lavish showy parties.
  • e.Not Attempted

18. The author's conception of 'social life' requires that

  • a. people attend large gatherings.
  • b. people possess qualities like wonder and interest.
  • c. people do not spend too much time in the company of intimate friends.
  • d. large parties consist of casual acquaintances and intimate friends.
  • e.Not Attempted

19. The word 'discriminate' in the last sentence of the third paragraph means

  • a. recognise.
  • b. count.
  • c. distinguish
  • d. analyse.
  • e.Not Attempted

20. What is the author trying to show through the two incidents in the paragraph beginning, "Their most innocent exhibition of sociability . . ."?

  • a. The crowds in poor Calcutta suburbs can turn violent without any provocation.
  • b. Although poor, the people of poor Calcutta suburbs have a rich social life.
  • c. It is risky for rich people to move around in poor suburbs
  • d. Achieving a high degree of sociability does not stop the poor from hating the rich.
  • e.Not Attempted

PASSAGE V

Modern science, exclusive of geometry, is a comparatively recent creation and can be said to have originated with Galileo and Newton. Galileo was the first scientist to recognize clearly that the only way to further our understanding of the physical world was to resort to experiment. However obvious Galileo's contention may appear in the light of our present knowledge, it remains a fact that the Greeks, in spite of their proficiency in geometry, never seem to have realized the importance of experiment. To a certain extent this may be attributed to the crudeness of their instruments of measurement. Still, an excuse of this sort can scarcely be put forward when the elementary nature of Galileo's experiments and observations is recalled. Watching a lamp oscillate in the cathedral of Pisa, dropping bodies from the leaning tower of Pisa, rolling balls down inclined planes, noticing the magnifying effect of water in a spherical glass vase, such was the nature of Galileo's experiments and observations. As can be seen, they might just as well have been performed by the Greeks. At any rate, it was thanks to such experiments that Galileo discovered the fundamental law of dynamics, according to which the acceleration imparted to a body is proportional to the force acting upon it.

The next advance was due to Newton, the greatest scientist of all time, if account be taken of his joint contributions to mathematics and physics. As a physicist, he was of course an ardent adherent of the empirical method, but his greatest title to fame lies in another direction. Prior to Newton, mathematics, chiefly in the form of geometry, had been studied as a fine art without any view to its physical applications other than very trivial cases. But with Newton all the resources of mathematics were turned to advantage in the solution of physical problems. Thenceforth mathematics appeared as an instrument of discovery, the most powerful one known to man, multiplying the power of thought just as in the mechanical domain the lever multiplied our physical action. It is this application of mathematics to the solution of physical problems, this combination of two separate fields of investigation, which constitutes the essential characteristic of the Newtonian method. Thus problems of physics were metamorphosed into problems of mathematics.

But in Newton's day the mathematical instrument was still in a very backward state of development. In this field again Newton showed the mark of genius by inventing the integral calculus. As a result of this remarkable discovery, problems, which would have baffled Archimedes, were solved with ease. We know that in Newton's hands this new departure in scientific method led to the discovery of the law of gravitation. But here again the real significance of Newton's achievement lay not so much in the exact quantitative formulation of the law of attraction, as in his having established the presence of law and order at least in one important realm of nature, namely, in the motions of heavenly bodies. Nature thus exhibited rationality and was not mere blind chaos and uncertainty. To be sure, Newton's investigations had been concerned with but a small group of natural phenomena, but it appeared unlikely that this mathematical law and order should turn out to be restricted to certain special phenomena; and the feeling was general that all the physical processes of nature would prove to be unfolding themselves according to rigorous mathematical laws.

When Einstein, in 1905, published his celebrated paper on the electrodynamics of moving bodies, he remarked that the difficulties, which surrounded the equations of electrodynamics, together with the negative experiments of Michelson and others, would be obviated if we extended the validity of the Newtonian principle of the relativity of Galilean motion, which applied solely to mechanical phenomena, so as to include all manner of phenomena: electrodynamics, optical, etc. When extended in this way the Newtonian principle of relativity became Einstein's special principle of relativity. Its significance lay in its assertion that absolute Galilean motion or absolute velocity must ever escape all experimental detection. Henceforth absolute velocity should be conceived of as physically meaningless, not only in the particular realm of mechanics, as in Newton's day, but in the entire realm of physical phenomena. Einstein's special principle, by adding increased emphasis to this relativity of velocity, making absolute velocity metaphysically meaningless, created a still more profound distinction between velocity and accelerated or rotational motion. This latter type of motion remained absolute and real as before. It is most important to understand this point and to realize that Einstein's special principle is merely an extension of the validity of the classical Newtonian principle to all classes of phenomena.

21. According to the author, why did the Greeks NOT conduct experiments to understand the physical world?

  • a. Apparently they did not think it necessary to experiment.
  • b. They focused exclusively on geometry.
  • c. Their instruments of measurement were very crude.
  • d. The Greeks considered the application of geometry to the physical world more important.
  • e.Not Attempted

22.The statement "Nature thus exhibited rationality and was not mere blind chaos and uncertainty" suggests that

  • a. problems that had baffied scientists like Archimedes were not really problems.
  • b. only a small group of natural phenomena was chaotic.
  • c. physical phenomena conformed to mathematical laws.
  • d. natural phenomena were evolving towards a less chaotic future.
  • e.Not Attempted

23. Newton may be considered one of the greatest scientists of all time because he

  • a. discovered the law of gravitation.
  • b. married physics with mathematics.
  • c. invented integral calculus.
  • d. started the use of the empirical method in science.
  • e.Not Attempted

24. Which of the following statements about modern science best captures the theme of the passage?

  • a. Modern science rests firmly on the platform built by the Greeks.
  • b. We need to go back to the method of enquiry used by the Greeks to better understand the laws of dynamics.
  • c. Disciplines like Mathematics and Physics function best when integrated into one.
  • d. New knowledge about natural phenomena builds on existing knowledge.
  • e.Not Attempted
25. The significant implication of Einstein's special principle of relativity is that
  • a. absolute velocity was meaningless in the realm of mechanics.
  • b. Newton's principle of relativity needs to be modified.
  • c. there are limits to which experimentation can be used to understand some physical phenomena.
  • d. it is meaningless to try to understand the distinction between velocity and accelerated or rotational motion.
  • e.Not Attempted

POEM I

Ithaka14
As you set out for Ithaka
hope the voyage is a long one,
full of adventure, full of discovery. Laistrygonians, Cyclops,
angry Poseidon–don't be afraid of them:
you'll never find things like that on your way as long as you keep your thoughts raised high,
as long as a rare excitement
stirs your spirit and your body.
Laistrygonians and Cyclops,
wild Poseidon - you won't encounter them
unless you bring them along inside your soul,
unless your soul sets them up in front of you.

Hope your voyage is a long one.
May there be many a summer morning when,
with what pleasure, what joy,
you come into harbours seen for the first time;
may you stop at Phoenician trading stations
to buy fine things,
mother of pearl and coral, amber and ebony,
sensual perfume of every kind-
as many sensual perfumes as you can;
and may you visit many Egyptian cities
to gather stores of knowledge from their scholars.

Keep Ithaka always in your mind.
Arriving there is what you're destined for.
But do not hurry the journey at all.
Better if it lasts for years,
so you are old by the time you reach the island,
wealthy with all you've gained on the way,
not expecting Ithaka to make you rich.

Ithaka gave you the marvellous journey.
Without her you would not have set out.
She has nothing left to give you now.

And if you find her poor, Ithaka won't have fooled you.
Wise as you will have become, so full of experience,
you'll have understood by then what these Ithakas mean.

26. Which of the following best reflects the central theme of this poem?

  • a. If you don't have high expectations, you will not be disappointed.
  • b. Don't rush to your goal; the journey is what enriches you.
  • c. The longer the journey the greater the experiences you gathe
  • d. You cannot reach Ithaka without visiting Egyptian ports.
  • e.Not Attempted

27. The poet recommends a long journey. Which of the following is the most comprehensive reason for it?

  • a. You can gain knowledge as well as sensual experience.
  • b. You can visit new cities and harbours.
  • c. You can experience the full range of sensuality.
  • d. You can buy a variety of fine things.
  • e.Not Attempted

28. In the poem, Ithaka is a symbol of

  • a. the divine mother.
  • b. your inner self.
  • c. the path to wisdom.
  • d. life's distant goal.
  • e.Not Attempted

29. What does the poet mean by 'Laistrygonians' and 'Cyclops'?

  • a. Creatures which, along with Poseidon, one finds during a journey.
  • b. Mythological characters that one should not be afraid of.
  • c. Intra-personal obstacles that hinder one's journey.
  • d. Problems that one has to face to derive the most from one's journey.
  • e.Not Attempted

30. Which of the following best reflects the tone of the poem?

  • a. Prescribing.
  • b. Exhorting.
  • c. Pleading.
  • d. Consoling.
  • e.Not Attempted

DIRECTIONS for question 31 to 35 :
In each of the questions, four different ways of presenting an idea are given. Choose the one that conforms most closely to Standard English usage.

31. A. The running of large businesses consist of getting somebody to make something that somebody else sold to somebody else for more than its cost.
B. The running of a large business consists of getting somebody to make something that somebody else will sell to somebody else for more than it costs.
C. The running of a large business consists of getting somebody to sell something that somebody else made for more than it cost.
D. The running of large businesses consist of getting somebody to make something else that somebody else will sell to somebody else for more than it costs

  • a. A
  • b. B
  • c. C
  • d. D
  • e.Not Attempted

32. A. From the sixteenth century onwards, people started feeling disdainful and self-conscious about their body and its products that led to a heightened focus on emotional and bodily regulations.
B. The heightened focus on controlling the body and emotions comes from disdain and self-consciousness about the body and its products, found in the sixteenth century.
C. From the sixteenth century onwards, a growing disdain for and self-consciousness about the body and its products took hold, leading to a heightened focus on emotional and bodily regulation.
D. The heightened focus on emotional and bodily regulations started from the sixteenth century onwards, when people felt disdain and self-consciousness about the body and its products.

  • a. A
  • b. B
  • c. C
  • d. D
  • e.Not Attempted

33. A. We are forced to fall back on fatalism as an explanation of irrational events.
B. We are forced to falling back on the fatalism as an explanation of irrational events.
C. We are forced to fall back on fatalism as explanations of irrational events.
D. We are forced to fall back to fatalism as an explanation of irrational events.

  • a. A
  • b. B
  • c. C
  • d. D
  • e.Not Attempted

34. A. Creativity in any field is regarded not only as valuable for itself but also as a service to the nation.
B. Creativity in any field is not regarded only as valuable on its own, but also as a service to the nation.
C. Creativity, in any field, is not only regarded as valuable, but also as a service to the nation.
D. Creativity in any field is regarded not only as valuable in itself but also as a service to the nation.

  • a. A
  • b. B
  • c. C
  • d. D
  • e.Not Attempted

35. A. If precision of thought had facilitated precision of behaviour, and if reflection had preceded action, it would be ideal for humans.
B. It would be ideal for humans if reflection preceded action and precision of thought facilitated precision of behaviour.
C. It would be ideal for humans if precedence of reflection was followed by action and precision of thought, by precise behaviour.
D. It would have been ideal for humans, if precise action and behaviour preceded precise reflection.

  • a. A
  • b. B
  • c. C
  • d. D
  • e. Not Attempted

DIRECTIONS for question number 36-40 :
The sentences given in each question, when properly sequenced, form a coherent paragraph. Each sentence is labelled with a letter. Choose the most logical order of sentences from among the given choices to construct a coherent paragraph.

36. A. A few months ago I went to Princeton University to see what the young people who are going to be running our country in a few decades are like.
B. I would go to sleep in my hotel room around midnight each night, and when I awoke, my mailbox would be full of repliessent at 1:15 a.m., 2:59 a.m., 3:23 a.m.
C. One senior told me that she went to bed around two and woke up each morning at seven; she could afford that much rest because she had learned to supplement her full day of work by studying in her sleep.
D. Faculty members gave me the names of a few dozen articulate students, and I sent them e-mails, inviting them out to lunch or dinner in small groups.
E. As she was falling asleep she would recite a math problem or a paper topic to herself; she would then sometimes dream about it, and when she woke up, the problem might be solved.

  • a. DABCE
  • b. DACEB
  • c. ADBCE
  • d. AECBD
  • e.Not Attempted

37. A. Four days later, Oracle announced its own bid for PeopleSoft, and invited the firm's board to a discussion.
B. Furious that his own plans had been endangered, PeopleSoft's boss, Craig Conway, called Oracle's offer "diabolical", and its boss, Larry Ellison, a "sociopath".
C. In early June, PeopleSoft said that it would buy J.D. Edwards, a smaller rival.
D. Moreover, said Mr. Conway, he "could imagine no price nor combination of price and other conditions to recommend accepting the offer."
E. On June 12th, PeopleSoft turned Oracle down.

  • a. CABDE
  • b. CADBE
  • c. CEDAB
  • d. CAEBD
  • e.Not Attempted

38.A. Surrendered, or captured, combatants cannot be incarcerated in razor wire cages; this 'war' has a dubious legality.
B. How can then one characterize a conflict to be waged against a phenomenon as war?
C. The phrase 'war against terror', which has passed into the common lexicon, is a huge misnomer.
D. Besides, war has a juridical meaning in international law, which has codified the laws of war, imbuing them with a humanitarian content.
E. Terror is a phenomenon, not an entity-either State or non-State.

  • a. ECDBA
  • b. BECDA
  • c. EBCAD
  • d. CEBDA
  • e.Not Attempted

39. A. I am much more intolerant of a human being's shortcomings than I am of an animal's, but in this respect I have been lucky, for most of the people I have come across have been charming.
B. Then you come across the unpleasant human animal-the District Officer who drawled, 'We chaps are here to help you chaps,' and then proceeded to be as obstructive as possible.
C. In these cases of course, the fact that you are an animal collector helps; people always seem delighted to meet someone with such an unusual occupation and go out of their way to assist you.
D. Fortunately, these types are rare, and the pleasant ones I have met more than compensated for them-but even so, I think I will stick to animals.
E. When you travel round the world collecting animals you also, of necessity, collect human beings.

  • a. EACBD
  • b. ABDCE
  • c. ECBDA
  • d. ACBDE
  • e.Not Attempted

40. A. To avoid this, the QWERTY layout put the keys most likely to be hit in rapid succession on opposite sides. This made the keyboard slow, the story goes, but that was the idea.
B. A different layout, which had been patented by August Dvorak in 1936, was shown to be much faster.
C. The QWERTY design (patented by Christopher Sholes in 1868 and sold to Remington in 1873) aimed to solve a mechanical problem of early typewriters.
D. Yet the Dvorak layout has never been widely adopted, even though (with electric typewriters and then PCs) the antijamming rationale for QWERTY has been defunct for years.
E. When certain combinations of keys were struck quickly, the type bars often jammed.

  • a. BDACE
  • b. CEABD
  • c. BCDEA
  • d. CAEBD
  • e.Not Attempted
DIRECTIONS for question number 41 to 45 :
In each question, the word at the top of the table is used in four different ways, numbered 1 to 4. Choose the option in which the usage of the word is INCORRECT or INAPPROPRIATE.

41.

Bundle
(1) The newborn baby was a bundle of joy for the family.
(2) Mobile operators are offering a bundle of additional benefits.
(3) He made a bundle in the share market.
(4) It was sheer luck that brought a bundle of boy-scouts to where I was lying wounded.
  • a. 1
  • b. 2
  • c. 3
  • d. 4
  • e.Not Attempted

42.

Distinct
(1) He is distinct about what is right and what is wrong.
(2) Mars became distinct on the horizon in the months of August.
(3) The distinct strains of Ravi's violin could be heard above the general din.
(4) Ghoshbabu's is a distinct case of water rising above its own level.
  • a. 1
  • b. 2
  • c. 3
  • d. 4
  • e.Not Attempted

43.

Implication
(1) Everyone appreciated the headmaster's implication in raising flood relief in the village.
(2) This letter will lead to the implication of several industrialists in the share market scam.
(3) Several members of the audience missed the implication of the minister's promise.
(4) Death, by implication, is the only solution the poem offers the reader.
  • a. 1
  • b. 2
  • c. 3
  • d. 4
  • e.Not Attempted

44.

Host
(1) If you host the party, who will foot the bill ?.
(2) Kerala's forests are host to a range of snakes.
(3) Ranchi will play the host to the next national film festival.
(4) A virus has infected the host computer.
  • a. 1
  • b. 2
  • c. 3
  • d. 4
  • e.Not Attempted

45.

Sort
(1) What sort of cheese do you use in pizza ?
(2) Farmers of all sort attended the rally..
(3) They serve tea of a sort on these trains.
(4) Let's sort these boys into four groups.
  • a. 1
  • b. 2
  • c. 3
  • d. 4
  • e.Not Attempted

DIRECTIONS for 1uestion number 46 to 50 :
There are two gaps in each of the following sentences. From the pairs of words given, choose the one that fills the gaps most appropriately. The first word in the pair should fill the first gap.

46.The British retailer, M&S, today formally ..... defeat in its attempt to ..... King's, its US subsidiary, since no potential purchasers were ready to cough up the necessary cash.

  • a. admitted, acquire
  • b. conceded, offload
  • c. announced, dispose
  • d. ratified, auction
  • e.Not Attempted

47. Early ..... of maladjustment to college culture is ..... by the tendency to develop friendship networks outside college which mask signals of maladjustment.

  • a. treatment, compounded
  • b. detection, facilitated
  • c. identification, complicated
  • d. prevention, helped
  • e.Not Attempted

48. The ..... regions of Spain all have unique cultures, but the ..... views within each region make the issue of an acceptable common language of instruction an even more contentious one

  • a. different, discrete
  • b. distinct, disparate
  • c. divergent, distinct
  • d. different, competing
  • e.Not Attempted

49. A growing number of these expert professionals ..... having to train foreigners as the students end up ..... the teachers who have to then unhappily contend with no jobs at all or new jobs with drastically reduced pay packets

  • a. resent, replacing
  • b. resist, challenging
  • c. welcome, assisting
  • d. are, supplanting
  • e.Not Attempted

50. Companies that try to improve employees' performance by ..... rewards encourage negative kinds of behaviour instead of ..... a genuine interest in doing the work well.

  • a. giving, seeking
  • b. bestowing, discouraging
  • c. conferring, discrediting
  • d. withholding, fostering
  • e.Not Attempted

CAT2003 SECTION - DI

Instructions
1. The test comprises of 50 questions. You should complete the test within 40 minutes.
2. There is only one correct answer to each question.
3. All questions carry four marks each.
4. Each wrong answer will attract a penalty of one mark.

Directions Questions 1-3:
Answer the questions on the basis of the information given below

Each point in the graph below shows the profit and turnover data for a company. Each company belongs to one of the three industries: textile, cement and steel.

1. For how many companies does the profit exceed 10% of turnover?

  • a. 8
  • b. 7
  • c. 6
  • d. 5
  • e.Not Attempted

2. An investor wants to buy stock of only steel or cement companies with a turnover more than 1000 and profit exceeding 10% of turnover. How many choices are available to the investor?

  • a. 4
  • b. 5
  • c. 6
  • d. 7
  • e.Not Attempted

3. For how many steel companies with a turnover of more than 2000 is the profit less than 300?

  • a. 0
  • b. 1
  • c. 2
  • d. 7
  • e.Not Attempted

DIRECTIONS for Q4 to Q6:
Answer the questions on the basis of the information given below:
One of the functions of the Reserve Bank of India is to mobilise funds for the Government of India by issuing securities. The following table shows details of funds mobilised during July 2002-July 2003. Notice that on each date there were two rounds of issues, each with a different maturity

4. How many times was the issue of securities under-subscribed, i.e. how often did the total amount mobilised fall short of the amount notified?

  • a. 0
  • b. 1
  • c. 2
  • d. 3
  • e.Not Attempted

5. Which of the following is true?

  • a. The second round issues have a higher maturity than the first round for all dates.
  • b.The second round issue of any date has a lower maturity only when the first round notified amount exceeds that of the second round.
  • c. On at least one occasion, the second round issue having lower maturity received a higher number of competitive bids
  • d. None of the above three statements is true.
  • e.Not Attempted

6. Which of the following statements is NOT true?

  • a. Competitive bids received always exceed non-competitive bids received.
  • b. The number of competitive bids accepted does not always exceed the number of non-competitive bids accepted.
  • c. The value of competitive bids accepted on any particular date is never higher for higher maturity.
  • d. The value of non-competitive bids accepted in the first round is always greater than that in the second round.
  • e.Not Attempted

DIRECTIONS for Q7 to Q10:
Answer the questions on the basis of the information given below:
The length of an infant is one of the measures of his/her development in the early stages of his/her life. The figure below shows the growth chart of four infants in the first five months of life. (Refer to diagram on next page)

7. After which month did Seeta's rate of growth start to decline?

  • a. Second month
  • b. Third month
  • c. Fourth month
  • d. Never
  • e.Not Attempted

8. Who grew at the fastest rate in the first two months of life?

  • a. Geeta
  • b. Seeta
  • c. Ram
  • d. Shvam
  • e.Not Attempted

9. The rate of growth during the third month was the lowest for

  • a. Geeta
  • b. Seeta
  • c. Ram
  • d. Shyam
  • e.Not Attempted

10. Among the four infants, who grew the least in the first five months of life?

  • a. Geeta
  • b. Seeta
  • c. Ram
  • d. Shyam
  • e.Not Attempted
DIRECTIONS for Questions 11 to 13:
Answer the questions on the basis of the information given below.
The table below provides certain demographic details of 30 respondents who were part of a survey. The demographic characteristics are: gender, number of children, and age of respondents. The first number in each cell is the number of respondents in that group. The minimum and maximum age of respondents in each group is given in brackets. For example, there are five female respondents with no children and among these five, the youngest is 34 years old, while the oldest is 49.

11. The percentage of respondents aged less than 40 years is at least:

  • a. 10
  • b. 16.67
  • c. 20
  • d. 30
  • e.Not Attempted

12. Given the information above, the percentage of respondents older than 35 can be at most

  • a. 30%
  • b. 73.3%
  • c. 76.67%
  • d. 90%
  • e.Not Attempted

13. The percentage of respondents that fall into the 35 to 40 years age group (both inclusive) is at least:

  • a. 6.67%
  • b. 10%
  • c. 13.33%
  • d. 26.67%
  • e.Not Attempted

DIRECTIONS for Questions 14 and 15:
Answer the questions on the basis of the information given below:

Figures in Rs A B C D Total
Sales 24568 25468 23752 15782 89570
Operating costs 17198 19101 16151 10258 62708
Interest costs 2457 2292 2850 1578 9177
Profit 4914 4075 4750 3946 17684

14. Which firm has the highest profitability?

  • a. A
  • b. B
  • c. C
  • d. D
  • e.Not Attempted

15. If Firm A acquires Firm B, approximately what percentage of the total market (total sales) will they corner together?

  • a. 55%
  • b. 45%
  • c. 35%
  • d. 50%
  • e. Not Attempted

DIRECTIONS for Questions 16 to 18:
Answer the questions on the basis of the information given below
Details of the top 20 MBA schools in the US as ranked by US News and World Report, 1997 are given below.

16. Madhu has received admission in all schools listed above. She wishes to select the highest overall ranked school whose a) annual tuition fee does not exceed $23,000 and b) median starting salary is at least $70,000. Which school will she select?

  • a. University of Virginia.
  • b. Northwestern University.
  • c. University of Pennsylvania.
  • d. University of California-Berkeley.
  • e.Not Attempted

17. In terms of starting salary and tuition fee, how many schools are uniformly better (higher median starting salary AND lower tuition fee) than Dartmouth College?

  • a. 1
  • b. 2
  • c. 3
  • d. 4
  • e.Not Attempted

18. How many schools in the list above have single digit rankings on at least 3 of the 4 parameters (overall ranking, ranking by academics, ranking by recruiters and ranking by placement)?

  • a. 10
  • b. 5
  • c. 7
  • d. 8
  • e.Not Attempted

DIRECTIONS for Questions 19 to 21:
In each question, there are two statements: A and B, either of which can be true or false on the basis of the information given below.
A research agency collected the following data regarding the admission process of a reputed management school in India.
Choose A if only A is true
Choose B if only B is true
Choose C if both A and B are true
Choose D if neither A nor B is true

19. Statement A: The success rate of moving from written test to interview stage for males was worse than for females in 2003.
Statement B: The success rate of moving from written test to interview stage for females was better in 2002 than in 2003.

  • a. A
  • b. B
  • c. C
  • d. D
  • e.Not Attempted

20. Statement A: In 2002, the number of females selected for the course as a proportion of the number of females who bought application forms, was higher than the corresponding proportion for males.
Statement B: In 2002, among those called for interview, males had a greater success rate than females

  • a. A
  • b. B
  • c. C
  • d. D
  • e.Not Attempted

21. Statement A: The percentage of absentees in the written test among females decreased from 2002 to 2003.
Statement B: The percentage of absentees in the written test among males was larger than among females in 2003.

  • a. A
  • b. B
  • c. C
  • d. D
  • e.Not Attempted

DIRECTIONS for Questions 22 to 24:
Answer the questions on the basis of the information given below
Spam that enters our electronic mailboxes can be classified under several spam heads. The following table shows the distribution of such spam worldwide over time. The total number of spam emails received during December 2002 was larger than the number received in June 2003. The total number of spam emails received during September 2002 was larger than the number received in March 2003. The figures in the table represent the percentage of all spam emails received during that period, falling into those respective catedgories.

22. In which category was the percentage of spam emails increasing but at a decreasing rate?

  • a. Financial
  • b. Scams
  • c. Products
  • d. None of the above
  • e.Not Attempted

23. In the health category, the number of spam emails received in December 2002 as compared to June 2003

  • a. was large
  • b. was smaller
  • c. was equal
  • d. Cannot be determined
  • e.Not Attempted

24. In the financial category, the number of spam emails received in September 2002 as compared to March 2003

  • a. was larger
  • b. was smaller
  • c. was equal
  • d. cannot be determined
  • e.Not Attempted

DIRECTIONS for Questions 25 to 27:
Answer the questions on the basis of the information given below.
Table A below provides data about ages of children in a school. For the age given in the first column, the second column gives the number of children not exceeding that age. For example, first entry indicates that there are 9 children aged 4 years or less. Tables B and C provide data on the heights and weights respectively of the same group of children in a similar format. Assuming that an older child is always taller and weighs more than a younger child, answer the following questions. (Refer to table on next page)

25. What is the number of children of age 9 years or less whose height does not exceed 135 cm?

  • a. 48
  • b. 45
  • c. 3
  • d. Cannot be determined
  • e.Not Attempted

26. How many children of age more than 10 years are taller than 150 cm. and do not weigh more than 48 kg.?

  • a. 16
  • b. 40
  • c. 3
  • d. Cannot be determined
  • e.Not Attempted

27. Among the children older than 6 years but not exceeding 12 years, how many weigh more than 38 kg.?

  • a. 34
  • b. 52
  • c. 44
  • d. Cannot be determined
  • e.Not Attempted

DIRECTIONS for Questions 28 and 29:
Answer the questions on the basis of the information given below.
The Head of a newly formed government desires to appoint five of the six elected members A, B, C, D, E and F to portfolios of Home, Power, Defence, Telecom and Finance. F does not want any portfolio if D gets one of the five. C wants either Home or Finance or no portfolio. B says that if D gets either Power or Telecom then she must get the other one. E insists on a portfolio if A gets one.

28. Which is a valid assignment?

  • a. A-Home, B-Power, C-Defence, D-Telecom, E-Finance
  • b. C-Home, D-Power, A-Defence, B-Telecom, E-Finance.
  • c. A-Home, B-Power, E-Defence, D-Telecom, F-Finance.
  • d. B-Home, F-Power, E-Defence, C-Telecom, A-Finance
  • e.Not Attempted

29. If A gets Home and C gets Finance, then which is NOT a valid assignment for Defence and Telecom?

  • a. D-Defence, B-Telecom.
  • b. F-Defence, B-Telecom.
  • c. B-Defence, E-Telecom.
  • d. B-Defence, D-Telecom.
  • e.Not Attempted

DIRECTIONS for Questions 30 to 32:
Answer the questions on the basis of the information given below.
Rang Barsey Paint Company (RBPC) is in the business of manufacturing paints. RBPC buys RED, YELLOW, WHITE, ORANGE, and PINK paints. ORANGE paint can be also produced by mixing RED and YELLOW paints in equal proportions. Similarly, PINK paint can also be produced by mixing equal amounts of RED and WHITE paints. Among other paints, RBPC sells CREAM paint, (formed by mixing WHITE and YELLOW in the ratio 70:30) AVOCADO paint (formed by mixing equal amounts of ORANGE and PINK paint) WASHEDORANGE paint (formed by mixing equal amounts of ORANGE and WHITE paint). The following table provides the price at which RBPC buys paints.

Color Rs./Litre
RED 20
YELLOW 25
WHITE 15
ORANGE 22
PINK 18

30. The cheapest way to manufacture AVOCADO paint would cost

  • a. Rs. 19.50 per litre
  • b. Rs. 20.00 per litre
  • c. Rs. 19.75 per litre
  • d. Rs. 20.25 per litre
  • e.Not Attempted

31. WASHEDORANGE can be manufactured by mixing

  • a. CREAM and RED in the ratio 14:10
  • b. CREAM and RED in the ratio 3:1.
  • c. YELLOW and PINK in the ratio 1:1
  • d. RED, YELLOW, and WHITE in the ratio 1:1:2
  • e.Not Attempted

32. Assume that AVOCADO, CREAM, and WASHEDORANGE each sells for the same price. Which of three is the most profitable to manufacture?

  • a. AVOCADO
  • b. CREAM
  • c. WASHEDORANGE
  • d. Sufficient data is not available
  • e.Not Attempted

Directions for question number 33-36:
In each question there are two statements A and B.
Choose (A) if the question can be answered by one of the statements alone but not by the other.
Choose (B) if the question can be answered by using either statement alone.
Choose (C) if the question can be answered by using both the statements together but cannot be an­swered using either statement alone.
Choose (D) if the question cannot be answered even by using both the statements A and B.

33.F and M are father and mother of S, respectively. S has four uncles and three aunts. F has two siblings. The siblings of F and M are unmarried. How many brothers does M have?
A. F has two brothers.
B. M has five siblings.

  • a. A
  • b. B
  • c. C
  • d. D
  • e.Not Attempted

34.A game consists of tossing a coin successively. There is an entry fee of Rs. 10 and an additional fee of Rs. 1 for each toss of the coin. The game is considered to have ended normally when the coin turns heads on two consecutive throws. In this case the player is paid Rs. 100. Alternatively, the player can choose to ter­minate the game prematurely after any of the tosses. Ram has incurred as loss of Rs 50 by playing this game. How many times did he toss the coin?
A. The game ended normally.
B. The total number of tails obtained in the game was 138.

  • a. A
  • b. B
  • c. C
  • d. D
  • e.Not Attempted

35. Each packet of SOAP costs Rs. 10. Inside each packet is a gift coupon labelled with one of the letters S, O, A and P. If a customer submits four such coupons that make up the word Soap, the customer gets a free Soap packet. Ms. X kept buying packet after packet of Soap till she could get one set of coupons that formed the world Soap. How many coupons with label P did she get in the above process?
A. The last label obtained by her was S and the total amount spent was Rs. 210.
B. The total number of vowels obtained was 18.

  • a. A
  • b. B
  • c. C
  • d. D
  • e. Not Attempted

36.If A and B run a race, then A wins by. 60 seconds, If B and C run the same race, then B wins by 30 seconds. Assuming that C maintains a uniform speed what is the time taken by C to finish the race?
A. A and C run the same race and A wins by 375 meters.
B The length of the race is 1 km.

  • a. A
  • b. B
  • c. C
  • d. D
  • e.Not Attempted

DIRECTIONS for Questions 37 to 39:
Answer the questions on the basis of the information given below.
A, B, C, D, E, and F are a group of friends. There are two housewives, one professor, one engineer, one accountant and one lawyer in the group. There are only two married couples in the group. The lawyer is married to D, who is a housewife. No woman in the group is either an engineer or an accountant. C, the accountant, is married to F, who is a professor. A is married to a housewife. E is not a housewife

37. Which of the following is one of the married couples?

  • a. A &B
  • b. E &B
  • c. D&E
  • d. D&A
  • e.Not Attempted

38. What is E's profession?

  • a. Engineer
  • b. Lawyer
  • c. Professor
  • d. Accountant
  • e.Not Attempted

39. How many members of the group are males?

  • a. 2
  • b. 3
  • c. 4
  • d. Cannot be determined
  • e.Not Attempted

Directions for question number 40 & 41:
Answer the questions on the basis of the information given below.
Some children were taking free throws at the basket­ball court in school during lunch break. Below are some facts about how many baskets these children shot.
i. Ganesh shot 8 baskets less than Ashish
ii. Dhanraj and Ramesh together shot 37 baskets
iii. Jugraj shot 8 baskets more than Dhanraj
iv. Ashish and Ganesh together shot 40 baskets

40. Which of the following statements is true?

  • a. Ramesh shot 18 baskets and Dhanraj shot 19 baskets
  • b. Ganesh shot 24 baskets and Ashish shot 16 baskets
  • c. Jugraj shot 19 baskets and Dhanraj shot 27 baskets
  • d. Dhanraj shot 11 baskets and Ashish shot 16 baskets
  • e.Not Attempted

41. Which of the following statements is true?

  • a. Dhanraj and Jugraj shot 46 baskets
  • b. Ganesh shot 18 baskets and Ramesh shot 21 baskets
  • c. Dhanraj shot 3 more baskets than Ramesh
  • d. Ramesh and Jugraj together shot 29 baskets
  • e.Not Attempted

DIRECTIONS for Questions 42 to 44:
Answer the questions on the basis of the information given below.
Seven varsity basketball players (A, B, C, D, E, F, and G) are to be honoured at a special luncheon. The players will be seated on the dais in a row. A and G have to leave the luncheon early and so must be seated at the extreme right. B will receive the most valuable player's trophy and so must be in the centre to facilitate presention. C and D are bitter rivals and therefore must be seated as far apart as possible.

42. Which of the following cannot be seated at either end?

  • a. C
  • b. D
  • c. F
  • d. G
  • e.Not Attempted

43. Which of the following pairs cannot be seated together?

  • a. B&D
  • b. C&F
  • c. D&G
  • d. E&A
  • e.Not Attempted

44. Which of the following pairs cannot occupy the seats on either side of B?

  • a. F&D
  • b. D&E
  • c. E&G
  • d. C&F
  • e.Not Attempted
DIRECTIONS for Questions 45 to 47:
Answer the questions on the basis of the information given below.
Five women decided to go shopping to M.G. Road, Bangalore. They arrived at the designated meeting place the following order: 1. Archana, 2. Chellamma, 3. Dhenuka, 4. Helen, and 5. Shahnaz. Each woman spent at least Rs. 1000. Below are some additional facts about how much they spent during their shopping spree.
The woman who spent Rs. 2234 arrived before the lady who spent Rs. 1193.
One woman spent Rs. 1340 and she was not Dhenuka.
One woman spent Rs. 1378 more than Chellamma.
One woman spent Rs. 2517 and she was not Archana.
Helen spent more than Dhenuka.
Shahnaz spent the largest amount and Chellamma the smallest

45. The woman who spent Rs. 1193 is

  • a. Archana
  • b. Chellamma
  • c. Dhenuka
  • d. Helen
  • e.Not Attempted

46. What was the amount spent by Helen?

  • a. Rs. 1193.
  • b. Rs. 1340.
  • c. Rs. 2234.
  • d. Rs.2517
  • e.Not Attempted

47. Which of the following amounts was spent by one of them?

  • a. Rs. 1139
  • b. Rs. 1378
  • c. Rs. 2571
  • d. Rs. 2718
  • e.Not Attempted

DIRECTIONS for Questions 48 to 50:
Answer the questions on the basis of the information given below.
Five friends meet every morning at Sree Sagar restaurant for an idli-vada breakfast. Each consumes a different number of idlis and vadas. The number of idlis consumed are 1, 4, 5, 6, and 8, while the number of vadas consumed are 0, 1, 2, 4, and 6. Below are some more facts about who eats what and how much.
i. The number of vadas eaten by Ignesh is three times the number of vadas consumed by the person who eats four idlis.
ii. Three persons, including the one who eats four vadas, eat without chutney.
iii. Sandeep does not take any chutney.
iv. The one who eats one idli a day does not eat any vadas or chutney. Further, he is not Mukesh.
v. Daljit eats idli with chutney and also eats vada.
vi. Mukesh, who does not take chutney, eats half as many vadas as the person who eats twice as many idlis as he does.
vii. Bimal eats two more idlis than Ignesh, but Ignesh eats two more vadas than Bimal.

48. Which of the following statements is true?

  • a. Sandeep eats 2 vadas
  • b. Mukesh eats 4 vadas
  • c. Ignesh eats 6 vadas
  • d. Bimal eats 4 vadas
  • e.Not Attempted

49. Which one of the following statements is true?

  • a. Daljit eats 5 idlis.
  • b. Ignesh eats 8 idlis
  • c. Bimal eats 1 idli
  • d. Bimal eats 6 idlis.
  • e.Not Attempted

50. Which of the following statements is true?

  • a. Mukesh eats 8 idlis and 4 vadas but no chutney.
  • b. The person who eats 5 idlis and 1 vada does not take chutney.
  • c. The person who eats equal number of vadas and idlis also takes chutney.
  • d. The person who eats 4 idlis and 2 vadas also takes chutney.
  • e.Not Attempted

CAT2004 SECTION - MATHS

Instructions
1.This test has three sections which examine various abilities. In all there are 123 questions. You will be given two hours to complete the test. In distributing the time over the three sections, please bear in mind that you need to demonstrate your competence in all three sections.
2. Directions for answering the questions are given before each group of questions. Read these directions carefully and answer the questions by darkening the appropriate circles on the Answer Sheet. There is only one correct answer to each question.
3. Each section carries 50 marks. Each section is divided into two sub-sections, A and B. For example, Section I is divided into two sub-sections. Sub-section I-A and Sub-section I-B. All questions in Sub-sections I-A and II-A carry one mark each. All questions in Sub-sections I-B, II-B and III-B carry two marks each. In Sub-section III-A, a group of 10 questions carries half a mark for each question; the remaining questions in Sub-section III-A carry one mark each. Wrong answers carry negative marks.
4. Do your rough work only on the Test Booklet and NOT on the Answer Sheet.
5. Follow the instructions of the invigilator. Candidates found violating the instructions will be disqualified.

SECTION II

Sub-Section II-A


Number of Questions : 20

DIRECTIONS for Questions 1 to 17:
Answer the questions independently of each other.

1. A father and his son are waiting at a bus stop in the evening. There is a lamp post behind them. The lamp post, the father and his son stand on the same straight line. The father observes that the shadows of his head and his son's head are incident at the same point on the ground. If the heights of the lamp post, the father and his son are 6 metres, 1.8 metres and 0.9 metres respectively, and the father is standing 2.1 metres away from the post, then how far (in metres) is the son standing from his father?

  • a. 0.9
  • b. 0.75
  • c. 0.6
  • d. 0.45
  • e.Not Attempted

2. A milkman mixes 20 litres of water with 80 litres of milk. After selling one-fourth of this mixture, he adds water to replenish the quantity that he has sold. What is the current proportion of water to milk?

  • a. 2 : 3
  • b. 1 : 2
  • c. 1 : 3
  • d. 3 : 4
  • e.Not Attempted

3. Karan and Arjun run a 100-metre race, where Karan beats Arjun by 10 metres. To do a favour to Arjun, Karan starts 10 metres behind the starting line in a second 100-metre race. They both run at their earlier speeds. Which of the following is true in connection with the second race?

  • a. Karan and Arjun reach the finishing line simultaneously.
  • b. Arjun beats Karan by 1 metre.
  • c. Arjun beats Karan by 11 metres.
  • d. Karan beats Arjun by 1 metre.
  • e.Not Attempted

4. N persons stand on the circumference of a circle at distinct points. Each possible pair of persons, not standing next to each other, sings a two-minute song one pair after the other. If the total time taken for singing is 28 minutes, what is N?

  • a. 5
  • b. 7
  • c. 9
  • d. None of these
  • e.Not Attempted

5. If the sum of the first 11 terms of an arithmetic progression equals that of the first 19 terms, then what is the sum of the first 30 terms?

  • a. 0
  • b. -1
  • c. 1
  • d. Not unique
  • e.Not Attempted

6. If a man cycles at 10 km/hr, then he arrives at a certain place at 1 p.m. If he cycles at 15 km/hr, he will arrive at the same place at 11 a.m. At what speed must he cycle to get there at noon?

  • a. 11 km/hr
  • b. 12 km/hr
  • c. 13 km/hr
  • d. 14 km/hr
  • e.Not Attempted

7. On January 1, 2004 two new societies, S1, and S2, are formed, each with n members. On the first day of each subsequent month, S1 adds b members while S2 multiplies its current number of members by a constant factor r. Both the societies have the same number of members on July 2, 2004. If b = 10.5n, what is the value of r?

  • a. 2.0
  • b. 1.9
  • c. 1.8
  • d. 1.7
  • e.Not Attempted

8. The total number of integer pairs (x, y) satisfying the equation x + y = xy is

  • a. 0
  • b. 1
  • c. 2
  • d. None of the above
  • e.Not Attempted

9. If f(x) = x3 - 4x + p, and f(0) and f(1) are of opposite signs, then which of the following is necessarily true?

  • a. -1 < p < 2
  • b. 0 < p < 3
  • c. -2 < p < 1
  • d. -3 < p < 0
  • e.Not Attempted

10. Suppose n is an integer such that the sum of the digits of n is 2, and 1010 < n < 1011. The number of different values for n is

  • a. 11
  • b. 10
  • c. 9
  • d. 8
  • e.Not Attempted

11. If a/(b+c) = b/(c+a) = c/(a+b) = r , then r cannot take any value except

  • a. 1/2
  • b. -1
  • c. 1/2 or -1
  • d. -1/2 or -1
  • e.Not Attempted

12. Let
What is the value of y?

  • a. (√13+3)/2
  • b. (√13-3)/2
  • c. (√15+3)/2
  • d. (√15-3)/2
  • e.Not Attempted

13. Let f(x) = ax2 - b |x|, where a and b are constants. Then at x = 0, f(x) is

  • a. maximized whenever a> 0, b >0
  • b. maximized whenever a > 0, b < 0
  • c. minimized whenever a > 0, b > 0
  • d. minimized whenever a > 0, b < 0
  • e.Not Attempted

14. Two boats, traveling at 5 and 10 kms per hour, head directly towards each other. They begin at a distance of 20 kms from each other. How far apart are they (in kms) one minute before they collide?

  • a. 1/12
  • b. 1/6
  • c. 1/4
  • d. 1/3
  • e.Not Attempted

15. Each family in a locality has at most two adults, and no family has fewer than 3 children. Considering all the families together, there are more adults than boys, more boys than girls, and more girls than families. Then the minimum possible number of families in the locality is

  • a. 4
  • b. 5
  • c. 2
  • d. 3
  • e. Not Attempted

16. In Nuts And Bolts factory, one machine produces only nuts at the rate of 100 nuts per minute and needs to be cleaned for 5 minutes after production of every 1000 nuts. Another machine produces only bolts at the rate of 75 bolts per minute and needs to be cleaned for 10 minutes after production of every 1500 bolts. If both the machines start production at the same time, what is the minimum duration required for producing 9000 pairs of nuts and bolts?

  • a. 130 minutes
  • b. 135 minutes
  • c. 170 minutes
  • d. 180 minutes
  • e.Not Attempted

17. A rectangular sheet of paper, when halved by folding it at the mid point of its longer side, results in a rectangle, whose longer and shorter sides are in the same proportion as the longer and shorter sides of the original rectangle. If the shorter side of the original rectangle is 2, what is the area of the smaller rectangle?

  • a. 4√2
  • b. 2 √2
  • c. √2
  • d. None of the above
  • e.Not Attempted

DIRECTIONS for Questions 18 to 20:
Answer the questions on the basis of the information given below.

In the adjoining figure, I and II are circles with centres P and Q respectively. The two circles touch each other and have a common tangent that touches them at points R and S respectively. This common tangent meets the line joining P and Q at O. The diameters of I and II are in the ratio 4:3 It is also known that the length of PO is 28 cm.

18. What is the ratio of the length of PQ to that of QO?

  • a. 1:4
  • b. 1:3
  • c. 3:8
  • d. 3:4
  • e.Not Attempted

19. What is the radius of the circle II?

  • a. 2cm
  • b. 3cm
  • c. 4cm
  • d. 5cm
  • e.Not Attempted

20. The length of SO is

  • a. 8 √3 cm
  • b. 10√ 3 cm
  • c. 12 √3 cm
  • d. 14 √3 cm
  • e.Not Attempted

SECTION II

Sub-Section II-B


Number of Questions : 15

DIRECTIONS for Questions 21 to 22:
Answer the questions independently of each other.

21.In the adjoining figure, chord ED is parallel to the diameter AC of the circle. If Ð CBE = 65 degree, then what is the value of Ð DEC?

  • a. 35 degree
  • b. 55 degree
  • c. 45 degree
  • d. 25 degree
  • e.Not Attempted

22. On a semicircle with diameter AD, chord BC is parallel to the diameter. Further, each of the chords AB and CD has length 2, while AD has length 8. What is the length of BC?

  • a. 7
  • b. 7.5
  • c. 7.75
  • d. None of the above
  • e.Not Attempted

DIRECTIONS for Questions 23 and 24:
Answer the questions on the basis of the information given below.

f1(x)= x for 0 = x = 1 
     = 1 for x = 1 
     = 0 otherwise 
f2(x)= f1(-x) for all x
f3(x)= -f2(x) for all x
f4(x)= f3(-x) for all x

23. How many of the following products are necessarily zero for every x
f1(x)f2(x), f2(x)f3(x), f2(x)f4(x)?

  • a. 0
  • b. 1
  • c. 2
  • d. 3
  • e.Not Attempted

24. Which of the following is necessarily true?

  • a. f4(x) = f1(x) for all x
  • b. f1(x) = -f3(-x) for all x
  • c. f2(-x) = f4(x) for all x
  • d. f1 (x) + f3(x) = 0 for all x
  • e.Not Attempted

DIRECTIONS for Questions 25 and 26:
Answer the questions on the basis of the information given below.

In an examination, there are 100 questions divided into three groups A, B and C such that each group contains at least one question. Each question in group A carries 1 mark, each question in group B carries 2 marks and each question in group C carries 3 marks. It is known that the questions in group A together carry at least 60% of the total marks. 25. If group B contains 23 questions, then how many questions are there in group C?

  • a. 1
  • b. 2
  • c. 3
  • d. Cannot be determined
  • e.Not Attempted

26. If group C contains 8 questions and group B carries at least 20% of the total marks, which of the following best describes the number of questions in group B?

  • a. 11 or 12
  • b. 12 or 13
  • c. 13 or 14
  • d. 14 or 15
  • e.Not Attempted

DIRECTIONS for Questions 27 to 35:
Answer the questions independently of each other.

27. A sprinter starts running on a circular path of radius r metres. Her average speed (in metres/minute) is pr during the first 30 seconds, πr/2 during next one minute, πr/4 during next 2 minutes, πr/8 during next 4 minutes, and so on. What is the ratio of the time taken for the nth round to that for the previous round?

  • a. 4
  • b. 8
  • c. 16
  • d. 32
  • e.Not Attempted

28. Consider the sequence of numbers a1, a3, a3, ..... to infinity where a1 = 81.33 and a2 = -19 and aj = aj-1 - aj-2 for j ≥ 3. What is the sum of the first 6002 terms of this sequence?

  • a. -100.33
  • b. -30
  • c. 62.33
  • d. 119.33
  • e.Not Attempted

29.The remainder, when (1523 + 2323) is divided by 19, is

  • a. 4
  • b. 15
  • c. 0
  • d. 18
  • e.Not Attempted

30. In the adjoining figure, the lines represent one-way roads allowing travel only northwards or only westwards. Along how many distinct routes can a car reach point B from point A?

  • a. 15
  • b. 56
  • c. 120
  • d. 336
  • e.Not Attempted

31. Let C be a circle with centre P0 and AB be a diameter of C. Suppose P1 is the mid point of the line segment P0B, P2 is the mid point of the line segment P1B and so on. Let C1, C2, C3, ...... be circles with diameters P0P1, P1P2, P2P3 ... respectively. Suppose the circles C1 C2, C3, .... are all shaded. The ratio of the area of the unshaded portion of C to that of the original circle C is

  • a. 8 : 9
  • b. 9 : 10
  • c. 10 : 11
  • d. 11 : 12
  • e.Not Attempted

32. Let u = (log2x)2 -6 log2x + 12 where x is a real number. Then the equation xu = 256, has

  • a. no solution for x
  • b. exactly one solution for x
  • c. exactly two distinct solutions for x
  • d. exactly three distinct solutions for x
  • e.Not Attempted

33. A new flag is to be designed with six vertical stripes using some or all of the colours yellow, green, blue and red. Then, the number of ways this can be done such that no two adjacent stripes have the same colour is

  • a. 12 x 81
  • b. 16 x 192
  • c. 20 x 125
  • d. 24 x 216
  • e.Not Attempted

34. If the lengths of diagonals DF, AG and CE of the cube shown in the adjoining figure are equal to the three sides of a triangle, then the radius of the circle circumscribing that triangle will be

  • a. equal to the side of the cube
  • b. √3 times the side of the cube
  • c. 1/√ 3 times the side of the cube
  • d. impossible to find from the given information
  • e.Not Attempted

35. A circle with radius 2 is placed against a right angle. Another smaller circle is also placed as shown in the adjoining figure. What is the radius of the smaller circle?

  • a. 3-2 √2
  • b. 4-2 √2
  • c. 7-4 √2
  • d. 6-4√ 2
  • e. Not Attempted

CAT2004 SECTION - EU

Instructions
1.This test has three sections which examine various abilities. In all there are 123 questions. You will be given two hours to complete the test. In distributing the time over the three sections, please bear in mind that you need to demonstrate your competence in all three sections.
2. Directions for answering the questions are given before each group of questions. Read these directions carefully and answer the questions by darkening the appropriate circles on the Answer Sheet. There is only one correct answer to each question.
3. Each section carries 50 marks. Each section is divided into two sub-sections, A and B. For example, Section I is divided into two sub-sections. Sub-section I-A and Sub-section I-B. All questions in Sub-sections I-A and II-A carry one mark each. All questions in Sub-sections I-B, II-B and III-B carry two marks each. In Sub-section III-A, a group of 10 questions carries half a mark for each question; the remaining questions in Sub-section III-A carry one mark each. Wrong answers carry negative marks.
4. Do your rough work only on the Test Booklet and NOT on the Answer Sheet.
5. Follow the instructions of the invigilator. Candidates found violating the instructions will be disqualified.

SECTION III

Sub-Section III-A

Number of Questions : 45

DIRECTIONS for Questions 1 to 10:
Fill up the blanks, numbered [1], [2] ...up to [10], in the two passages below with the most appropriate word from the options given for each blank.

At that time the White House was as serene as a resort hotel out of season. The corridors were [1]. In the various offices, [2] gray men in waistcoats talked to one another in low-pitched voices. The only color, or choler, curiously enough, was provided by President Eisenhower himself. Apparently, his [3] was easily set off; he scowled when he [4] the corridors.

1.

  • a. striking
  • b. hollow
  • c. empty
  • d. white
  • e.Not Attempted

2.

  • a. quiet
  • b. faded
  • c. loud
  • d. stentorian
  • e.Not Attempted

3.

  • a. laughter
  • b. curiosity
  • c. humour
  • d. temper
  • e.Not Attempted

4.

  • a. paced
  • b. strolled
  • c. stormed
  • d. prowled
  • e.Not Attempted

"Between the year 1946 and the year 1955, ! did not file any income tax returns." With that [5] statement, Ramesh embarked on an account of his encounter with the Income Tax Department. "I originally owed Rs. 20,000 in unpaid taxes. With [6] and [7], the 20,000 became 60,000. The Income Tax Department then went into action, and I learned first hand just how much power the Tax Department wields. Royalties and trust funds can be [8]; automobiles may be [9], and auctioned off. Nothing belongs to the [10] until the case is settled."

5.

  • a. devious
  • b. blunt
  • c. tactful
  • d. pretentious
  • e.Not Attempted

6.

  • a. interest
  • b. taxes
  • c. principal
  • d. returns
  • e.Not Attempted

7.

  • a. sanctions
  • b. refunds
  • c. fees
  • d. fines
  • e.Not Attempted

8.

  • a.closed
  • b.detached
  • c.attached
  • d.impounded
  • e.Not Attempted

9.

  • a. smashed
  • b. seized
  • c. dismantled
  • d. frozen
  • e.Not Attempted

10.

  • a. purchaser
  • b. victim
  • c. investor
  • d. offender
  • e.Not Attempted
DIRECTIONS for Questions 11 to 13:
Identify the incorrect sentence or sentences.

11. A. Harish told Raj to plead guilty.
B. Raj pleaded guilty of stealing money from the
C. The court found Raj guilty of all the crimes
D. He was sentenced for three years in jail.

  • a. A and C
  • b. B and D
  • c. A, C, and D
  • d. B, C, and D
  • e.Not Attempted

12. A. Last Sunday, Archana had nothing to do.
B. After waking up, she lay on the bed thinking of what to do.
C. At 11 o'clock she took shower and got ready.
D. She spent most of the day shopping.

  • a. B and C
  • b. C
  • c. A and B
  • d. B, C, and D
  • e.Not Attempted

13. A. It was a tough situation and Manasi was taking pains to make it better.
B. Slowly her efforts gave fruit and things started improving.
C. Everyone complemented her for her good work.
D. She was very happy and thanked everyone for their help.

  • a. A
  • b. D
  • c. B and C
  • d. A and C
  • e.Not Attempted

DIRECTIONS for Questions 14 to 16:
Each statement has a part missing. Choose the best option from the four options given below the statement to make up the missing part.

14. The ancient Egyptians believed ..... so that when these objects were magically reanimated through the correct rituals, they would be able to function effectively.

  • a. that it was essential that things they portrayed must have every relevant feature shown as clearly as possible
  • b. it was essential for things they portray to have had every relevant feature shown as clearly as possible,
  • c. it was essential that the things they portrayed had every relevant feature shown as clearly as possible,
  • d. that when they portrayed things, it should have every relevant feature shown as clearly as possible
  • e.Not Attempted

15. Archaeologists believe that the pieces of red-ware pottery excavated recently near Bhavnagar and ..... shed light on a hitherto dark 600-year period in the Harappan history of Gujarat.

  • a. estimated with a reasonable certainty as being about 3400 years old,
  • b. are estimated reasonably certain to be about 3400 years old
  • c. estimated at about 3400 years old with reasonable certainty,
  • d. estimated with reasonable certainty to be about 3400 years old,
  • e. Not Attempted

16.Many people suggest ..... and still others would like to convince people not to buy pirated cassettes.

  • a. to bring down audiocassette prices to reduce the incidence of music piracy, others advocate strong legal action against the offenders,
  • b. Nbringing down audiocassette prices to reduce the incidents of music piracy, others are advocating strong legal action against offenders,
  • c. bringing down audiocassette prices to reduce the incidence of music piracy, others advocate strong legal action against offenders,.
  • d. audiocassette prices to be brought down to reduce incidence of music piracy, others advocate that strong legal action must be taken against offenders,
  • e.Not Attempted

DIRECTIONS for Questions 17 to 19:
In each question, the word at the top of the table is used in four different ways, numbered 1 to 4. Choose the option in which the usage of the word is INCORRECT or INAPPROPRIATE.

17. BOLT

  • a. The shopkeeper showed us a bolt of fine silk.
  • b. As he could not move, he made a bolt for the gate.
  • c. Could you please bolt the door?
  • d. The thief was arrested before he could bolt from the scene of the crime.
  • e.Not Attempted

18. PASSING

  • a. She did not have passing marks in mathematics.
  • b. The mad woman was cursing everybody passing her on the road.
  • c. At the birthday party all the children enjoyed a game of passing the parcel.
  • d. A passing taxi was stopped to rush the accident victim to the hospital.
  • e.Not Attempted

19. Fallout

  • a. Nagasaki suffered from the fallout of nuclear radiation.
  • b. People believed that the political fallout of the scandal would be insignificant.
  • c. Who can predict the environmental fallout of the WTO agreements?
  • d. The headmaster could not understand the fallout of several of his good students at the public examination.
  • e.Not Attempted

DIRECTIONS for Questions 20 to 22:
The sentences given in each question, when properly sequenced, form a coherent paragraph.Each sentence is labeled with a letter. Choose the most logical order of sentences from among the given choices to construct a coherent paragraph.

20. A. The two neighbours never fought each other. B. Fights involving three male fiddler crabs have been recorded, but the status of the participants was unknown. C. They pushed or grappled only with the intruder. D. We recorded 17 cases in which a resident that was fighting an intruder was joined by an immediate neighbour, an ally. E. We therefore tracked 268 intruder males until we saw them fighting a resident male.

  • a. BEDAC
  • b. DEBAC
  • c. BDCAE
  • d. BCEDA
  • e.Not Attempted

21. A. He felt justified in bypassing Congress altogether on a variety of moves. B. At times he was fighting the entire Congress. C. Bush felt he had a mission to restore power to the presidency. D. Bush was not fighting just the democrats. E. Representative democracy is a messy business, and a CEO of the White House does not like a legislature of second guessers and time wasters.

  • a. CAEDB
  • b. DBAEC
  • c. CEADB
  • d. ECDBA
  • e.Not Attempted

22. A. In the west, Allied Forces had fought their way through southern Italy as far as Rome. B. In June 1944 Germany's military position in World War Two appeared hopeless. C. In Britain, the task of amassing the men and materials for the liberation of northern Europe had been completed. D. The Red Army was poised to drive the Nazis back through Poland. E. The situation on the eastern front was catastrophic.

  • a. EDACB
  • b. BEDAC
  • c. BDECA
  • d. CEDAB
  • e.Not Attempted

DIRECTIONS for Questions 23 and 24:
Four alternative summaries are given below each text. Choose the option that best captures the essence of the text.

23. You seemed at first to take no notice of your school-fellows, or rather to set yourself against them because they were strangers to you. They knew as little of you as you did of them; this would have been the reason for their keeping aloof from you as well, which you would have felt as a hardship. Learn never to conceive a prejudice against others because you know nothing of them. It is bad reasoning, and makes enemies of half the world. Do not think ill of them till they behave ill to you; and then strive to avoid the faults which you see in them. This will disarm their hostility sooner than pique or resentment or complaint

  • a. The discomfort you felt with your school fellows was because both sides knew little of each other. You should not complain unless you find others prejudiced against you and have attempted to carefully analyze the faults you have observed in them.
  • b. The discomfort you felt with your school fellows was because both sides knew little of each other. Avoid prejudice and negative thoughts till you encounter bad behaviour from others, and then win them over by shunning the faults you have observed.
  • c. You encountered hardship amongst your school fellows because you did not know them well. You should learn to not make enemies because of your prejudices irrespective of their behaviour towards you.
  • d. You encountered hardship amongst your school fellows because you did not know them well. You should learn to not make enemies because of your prejudices unless they behave badly with you.
  • e.Not Attempted

24. The human race is spread all over the world, from the polar regions to the tropics. The people of whom it is made up eat different kinds of food, partly according to the climate in which they live, and partly according to the kind of food which their country produces. In hot climates, meat and fat are not much needed; but in the Arctic regions they seem to be very necessary for keeping up the heat of the body. Thus, in India, people live chiefly on different kinds of grains, eggs, milk, or sometimes fish and meat. In Europe, people eat more meat and less grain. In the Arctic regions, where no grains and fruits are produced, the Eskimo and other races live almost entirely on meat and fish.

  • a. Food eaten by people in different regions of the world depends on the climate and produce of the region, and varies from meat and fish in the Arctic to predominantly grains in the tropics.
  • b. Hot climates require people to eat grains while cold regions require people to eat meat and fish.
  • c. In hot countries people eat mainly grains while in the Arctic, they eat meat and fish because they cannot grow grains.
  • d. While people in Arctic regions like meat and fish and those in hot regions like India prefer mainly grains, they have to change what they eat depending on the local climate and the local produce.
  • e.Not Attempted

DIRECTIONS for Questions 25 to 45:
Each of the five passages given below is followed by a set of questions. Choose the best answer to each question.

PASSAGE I

Recently I spent several hours sitting under a tree in my garden with the social anthropologist William Ury, a Harvard University professor who specializes in the art of negotiation and wrote the bestselling book, Getting to Yes. He captivated me with his theory that tribalism protects people from their fear of rapid change. He explained that the pillars of tribalism that humans rely on for security would always counter any significant cultural or social change. In this way, he said, change is never allowed to happen too fast. Technology, for example, is a pillar of society. Ury believes that every time technology moves in a new or radical direction, another pillar such as religion or nationalism will grow stronger - in effect, the traditional and familiar will assume greater importance to compensate for the new and untested. In this manner, human tribes avoid rapid change that leaves people insecure and frightened.

But we have all heard that nothing is as permanent as change. Nothing is guaranteed. Pithy expressions, to be sure, but no more than clichs. As Ury says, people don't live that way from day-to-day. On the contrary, they actively seek certainty and stability. They want to know they will be safe.

Evert so, we scare ourselves constantly with the idea of change. An IBM CEO once said: 'We only re-structure for a good reason, and if we haven't re-structured in a while, that's a good reason.' We are scared that competitors, technology and the consumer will put us out of business - so we have to change all the time just to stay alive. But if we asked our fathers and grandfathers, would they have said that they lived in a period of little change? Structure may not have changed much. It may just be the speed with which we do things.

Change is over-rated, anyway. Consider the automobile. It's an especially valuable example, because the auto industry has spent tens of billions of dollars on research and product development in the last 100 years. Henry Ford's first car had a metal chassis with an internal combustion, gasoline-powered engine, four wheels with rubber tyres, a foot operated clutch assembly and brake system, a steering wheel, and four seats, and it could safely do 18 miles per hour. A hundred years and tens of thousands of research hours later, we drive cars with a metal chassis with an internal combustion, gasoline-powered engine, four wheels with rubber tyres, a foot operated clutch assembly and brake system, a steering wheel, four seats - and the average speed in London in 2001 was 17.5 miles per hour!

That's not a hell of a lot of return for the money. Ford evidently doesn't have much to teach us about change. The fact that they're still manufacturing cars is not proof that Ford Motor Co. is a sound organization, just proof that it takes very large companies to make cars in great quantities - making for an almost impregnable entry barrier. Fifty years after the development of the jet engine, planes are also little changed. They've grown bigger, wider and can carry more people. But those are incremental, largely cosmetic changes.

Taken together, this lack of real change has come to mean that in travel – whether driving or flying - time and technology have not combined to make things much better. The safety and design have of course accompanied the times and the new volume of cars and flights, but nothing of any significance has changed in the basic assumptions of the final product. At the same time, moving around in cars or aeroplanes becomes less and less efficient all the time. Not only has there been no great change, but also both forms of transport have deteriorated as more people clamour to use them. The same is true for telephones, which took over hundred years to become mobile, or photographic film, which also required an entire century to change.

The only explanation for this is anthropological. Once established in calcified organizations, humans do two things: sabotage changes that might render people dispensable, and ensure industry-wide emulation. In the 1960s, German auto companies developed plans to scrap the entire combustion engine for an electrical design. (The same existed in the 1970s in Japan, and in the 1980s in France.) So for 40 years we might have been free of the wasteful and ludicrous dependence on fossil fuels. Why didn't it go anywhere? Because auto executives understood pistons and carburettors, and would be loath to cannibalize their expertise, along with most of their factories. 25. According to the passage, which of the following statements is true?

  • a. Executives of automobile companies are inefficient and ludicrous.
  • b. The speed at which an automobile is driven in a city has not changed much in a century.
  • c. Anthropological factors have fostered innovation in automobiles by promoting use of new technologies.
  • d. Further innovation in jet engines has been more than incremental.
  • e.Not Attempted

26. Which of the following views does the author fully support in the passage?

  • a. Nothing is as permanent as change.
  • b. Change is always rapid.
  • c. More money spent on innovation leads to more rapid change.
  • d. Over decades, structural change has been incremental.
  • e.Not Attempted

27. Which of the following best describes one of the main ideas discussed in the passage?

  • a. Rapid change is usually welcomed in society.
  • b. Industry is not as innovative as it is made out to be.
  • c. We should have less change than what we have now.
  • d. Competition spurs companies into radical innovation.
  • e.Not Attempted

28. According to the passage, the reason why we continued to be dependent on fossil fuels is that:

  • a. Auto executives did not wish to change.
  • b. No alternative fuels were discovered.
  • c. Change in technology was not easily possible.
  • d. German, Japanese and French companies could not come up with new technologies.
  • e.Not Attempted

PASSAGE II

The painter is now free to paint anything he chooses. There are scarcely any forbidden subjects, and today everybody is prepared to admit that a painting of some fruit can be as important as a painting of a hero dying. The Impressionists did as much as anybody to win this previously unheard-of freedom for the artist. Yet, by the next generation, painters began to abandon the subject altogether, and began to paint abstract pictures. Today the majority of pictures painted are abstract.

Is there a connection between these two developments? Has art gone abstract because the artist is embarrassed by his freedom? Is it that, because he is free to paint anything, he doesn't know what to paint? Apologists for abstract art often talk of it as the art of maximum freedom. But could this be the freedom of the desert island? It would take too long to answer these questions properly. I believe there is a connection. Many things have encouraged the development of abstract art. Among them has been the artists' wish to avoid the difficulties of finding subjects when all subjects are equally possible.

I raise the matter now because I want to draw attention to the fact that the painter's choice of a subject is a far more complicated question than it would at first seem. A subject does not start with what is put in front of the easel or with something which the painterhappens to remember. A subject starts with the painter deciding he would like to paint such-and-such because for some reason orother he finds it meaningful. A subject begins when the artist selects something for special mention. (What makes it special or meaningful may seem to the artist to be purely visual - its colours or its form.) When the subject has been selected, the function of the painting itself is to communicate and justify the significance of that selection.

It is often said today that subject matter is unimportant. But this is only a reaction against the excessively literary and moralistic interpretation of subject matter in the nineteenth century. In truth the subject is literally the beginning and end of a painting. The painting begins with a selection (I will paint this and not everything else in the world); it is finished when that selection is justified (now you can see all that I saw and felt in this and how it is more than merely itself).

Thus, for a painting to succeed it is essential that the painter and his public agree about what is significant. The subject may have a personal meaning for the painter or individual spectator; but there must also be the possibility of their agreement on its general meaning. It is at this point that the culture of the society and period in question precedes the artist and his art. Renaissance art would have meant nothing to the Aztecs—and vice versa. If, to some extent, a few intellectuals can appreciate them both today it is because their culture is an historical one: its inspiration is history and therefore it can include within itself, in principle if not in everyparticular, all known developments to date.

When a culture is secure and certain of its values, it presents its artists with subjects. The general agreement about what is significant is so well established that the significance of a particular subject accrues and becomes traditional. This is true, for instance, of reeds and water in China, of the nude body in Renaissance, of the animal in Africa. Furthermore, in such cultures the artist is unlikely to be a free agent: he will be employed for the sake of particular subjects, and the problem, as we have just described it, will not occur to him.

When a culture is in a state of disintegration or transition the freedom of the artist increases' but the question of subject matter becomes problematic for him: he, himself, has to choose for society. This was at the basis of all the increasing crises in European art during the nineteenth century. It is too often forgotten how many of the art scandals of that time were provoked by the choice of subject (Gericault, Courbet, Daumier, Degas, Lautrec, Van Gogh, etc.).

By the end of the nineteenth century there were, roughly speaking, two ways in which the painter could meet this challenge of deciding what to paint and so choosing for society. Either he identified himself with the people and so allowed their lives to dictate his subjects to him; or he had to find his subjects within himself as painter. By people I mean everybody except the bourgeoisie. Many painters did of course work for the bourgeoisie according to their copy-book of approved subjects, but all of them, filling the Salon and the Royal Academy year after year, are now forgotten, buried under the hypocrisy of those they served so sincerely.

29. When a culture is insecure, the painter chooses his subject on the basis of:

  • a. The prevalent style in the society of his time.
  • b. Its meaningfulness to the painter.
  • c. What is put in front of the easel.
  • d. Past experience and memory of the painter.
  • e.Not Attempted

30. In the sentence, "I believe there is a connection" (second paragraph), what two developments is the author referring to?

  • a. Painters using a dying hero and using a fruit as a subject of painting.
  • b. Growing success of painters and an increase in abstract forms.
  • c. Artists gaining freedom to choose subjects and abandoning subjects altogether.
  • d. Rise of Impressionists and an increase in abstract forms.
  • e.Not Attempted

31. Which of the following is NOT necessarily among the attributes needed for a painter to succeed:

  • a. The painter and his public agree on what is significant.
  • b. The painting is able to communicate and justify the significance of its subject selection.
  • c. The subject has a personal meaning for the painter.
  • d. The painting of subjects is inspired by historical developments.
  • e.Not Attempted

32. In the context of the passage, which of the following statements would NOT be true?

  • a. Painters decided subjects based on what they remembered from their own lives.
  • b. Painters of reeds and water in China faced no serious problem of choosing a subject.
  • c. The choice of subject was a source of scandals in nineteenth century European art.
  • d. Agreement on the general meaning of a painting is influenced by culture and historical context.
  • e.Not Attempted

33. Which of the following views is taken by the author?

  • a. The more insecure a culture, the greater the freedom of the artist.
  • b. The more secure a culture, the greater the freedom of the artist.
  • c. The more secure a culture, more difficult the choice of subject.
  • d. The more insecure a culture, the less significant the choice of the subject.
  • e.Not Attempted

PASSAGE III

The viability of the multinational corporate system depends upon the degree to which people will tolerate the unevenness it creates. It is well to remember that the 'New Imperialism' which began after 1870 in a spirit of Capitalism Triumphant, soon became seriously troubled and after 1914 was characterized by war, depression, breakdown of the international economic system and war again, rather than Free Trade, Pax Britannica and Material Improvement. A major reason was Britain's inability to cope with the by-products of its own rapid accumulation of capital; i.e., a class-conscious labour force at home; a middle class in the hinterland; and rival centres of capital on the Continent and in America. Britain's policy tended to be atavistic and defensive rather than progressive more concerned with warding off new threats than creating new areas of expansion. Ironically, Edwardian England revived the paraphernalia of the landed aristocracy it had just destroyed. Instead of embarking on a 'big push' to develop the vast hinterland of the Empire, colonial administrators often adopted policies to arrest the development of either a native capitalist class or a native proletariat which could overthrow them.

As time went on, the centre had to devote an increasing share of government activity to military and other unproductive expenditures; they had to rely on alliances with an inefficient class of landlords, officials and soldiers in the hinterland to maintain stability at the cost of development. A great part of the surplus extracted from the population was thus wasted locally.

The New Mercantilism (as the Multinational Corporate System of special alliances and privileges, aid and tariff concessions is sometimes called) faces similar problems of internal and external division. The centre is troubled: excluded groups revolt and even some of the affluent are dissatisfied with the roles. Nationalistic rivalry between major capitalist countries remains an important divisive factor. Finally, there is the threat presented by the middle classes and the excluded groups of the underdeveloped countries. The national middle classes in the underdeveloped countries came to power when the centre weakened but could not, through their policy of import substitution manufacturing, establish a viable basis for sustained growth. They now face a foreign exchange crisis and an unemployment (or population) crisis the first indicating their inability to function in the international economy and the second indicating their alienation from the people they are supposed to lead. In the immediate future, these national middle classes will gain a new lease of life as they take advantage of the spaces created by the rivalry between American and non-American oligopolists striving to establish global market positions.

The native capitalists will again become the champions of national independence as they bargain with multinational corporations. But the conflict at this level is more apparent than real, for in the end the fervent nationalism of the middle class asks only for promotion within the corporate structure and not for a break with that structure. In the last analysis their power derives from the metropolis and they cannot easily afford to challenge the international system. They do not command the loyalty of their own population and cannot really compete with the large, powerful, aggregate capitals from the centre. They are prisoners of the taste patterns and consumption standards set at the centre.

The main threat comes from the excluded groups. It is not unusual in underdeveloped countries for the top 5 per cent to obtain between 30 and 40 per cent of the total national income, and for the top one-third to obtain anywhere from 60 to 70 per cent. At most, one-third of the population can be said to benefit in some sense from the dualistic growth that characterizes development in the hinterland. The remaining two-thirds, who together get only one-third of the income, are outsiders, not because they do not contribute to the economy, but because they do not share in the benefits. They provide a source of cheap labour which helps keep exports-to the developed world at a low price and which has financed the urban-biased growth of recent years. In fact, it is difficult to see how the system in most underdeveloped countries could survive without cheap labour since removing it (e.g. diverting it to public works projects as is done in socialist countries) would raise consumption costs to capitalists and professional elites.

34. The author is in a position to draw parallels between New Imperialism and New Mercantilism because

  • a. both originated in the developed Western capitalist countries.
  • b. New Mercantilism was a logical sequel to New Imperialism.
  • c. they create the same set of outputs - a labour force, middle classes and rival centres of capital.
  • d. both have comparable uneven and divisive effects.
  • e.Not Attempted

35. According to the author, the British policy during the 'New Imperialism' period tended to be defensive because

  • a. it was unable to deal with the fallouts of a sharp increase in capital.
  • b. its cumulative capital had undesirable side-effects.
  • c. its policies favoured developing the vast hinterland.
  • d. it prevented the growth of a set-up which could have been capitalistic in nature.
  • e. Not Attempted

36.In the sentence, "They are prisoners of the taste patterns and consumption standards set at the centre." (fourth paragraph), what is the meaning of 'centre'?

  • a. National government
  • b. Native capitalists
  • c. New capitalists
  • d. None of the above
  • e.Not Attempted

37. Under New Mercantilism, the fervent nationalism of the native middle classes does not create conflict with the multinational corporations because they (the middle classes)

  • a. negotiate with the multinational corporations.
  • b. are dependent on the international system for their continued prosperity.
  • c. are not in a position to challenge the status quo.
  • d. do not enjoy popular support.
  • e.Not Attempted

PASSAGE IV

Throughout human history the leading causes of death have been infection and trauma. Modern medicine has scored significant victories against both, and the major causes of ill health and death are now the chronic degenerative diseases, such as coronary artery disease, arthritis, osteoporosis, Alzheimer's, macular degeneration, cataract and cancer. These have a long latency period before symptoms appear and a diagnosis is made. It follows that the majority of apparently healthy people are pre-ill. But are these conditions inevitably degenerative? A truly preventive medicine that focused on the pre-ill, analysing the metabolic errors which lead to clinical illness, might be able to correct them before the first symptom. Genetic risk factors are known for all the chronic degenerative diseases, and are important to the individuals who possess them. At the population level, however, migration studies confirm that these illnesses are linked for the most part to lifestyle factors' exercise, smoking and nutrition. Nutrition is the easiest of these to change, and the most versatile tool for affecting the metabolic changes needed to tilt the balance away from disease. Many national surveys reveal that malnutrition is common in developed countries. This is not the calorie and/or micronutrient deficiency associated with developing nations (Type A malnutrition); but multiple micronutrient depletion, usually combined with calorific balance or excess (Type B malnutrition). The incidence and severity of Type B malnutrition will be shown to be worse if newer micronutrient groups such as the essential fatty acids, xanthophylls and flavonoids are included in the surveys. Commonly ingested levels of these micronutrients seem to be far too low in many developed countries. There is now considerable evidence that Type B malnutrition is a major cause of chronic degenerative diseases. If this is the case, then it is logical to treat such diseases not with drugs but with multiple micronutrient repletion, or "pharmaco-nutrition'. This can take the form of pills and capsules- 'nutraceuticals', or food formats known as 'functional foods'. This approach has been neglected hitherto because it is relatively unprofitable for drug companies 'the products are hard to patent ' and it is a strategy which does not sit easily with modern medical interventionism. Over the last 100 years, the drug industry has invested huge sums in developing a range of subtle and powerful drugs to treat the many diseases we are subject to. Medical training is couched in pharmaceutical terms and this approach has provided us with an exceptional range of therapeutic tools in the treatment of disease and in acute medical emergencies. However, the pharmaceutical model has also created an unhealthy dependency culture, in which relatively few of us accept responsibility for maintaining our own health. Instead, we have handed over this responsibility to health professionals who know very little about health maintenance, or disease prevention. One problem for supporters of this argument is lack of the right kind of hard evidence. We have a wealth of epidemiological data linking dietary factors to health profiles / disease risks, and a great deal of information on mechanism: how food factors interact with our biochemistry. But almost all intervention studies with micronutrients, with the notable exception of the omega 3 fatty acids, have so far produced conflicting or negative results. In other words, our science appears to have no predictive value. Does this invalidate the science? Or are we simply asking the wrong questions? Based on pharmaceutical thinking, most intervention studies have attempted to measure the impact of a single micronutrient on the incidence of disease. The classical approach says that if you give a compound formula to test subjects and obtain positive results, you cannot know which ingredient is exerting the benefit, so you must test each ingredient individually. But in the field of nutrition, this does not work. Each intervention on its own will hardly make enough difference to be measured. The best therapeutic response must therefore combine micronutrients to normalise our internal physiology. So do we need to analyse each individual's nutritional status and then tailor a formula specifically for him or her? While we do not have the resources to analyse millions of individual cases, there is no need to do so. The vast majority of people are consuming suboptimal amounts of most micronutrients, and most of the micronutrients concerned are very safe. Accordingly, a comprehensive and universal program of micronutrient support is probably the most cost-effective and safest way of improving the general health of the nation.

38. Type-B malnutrition is a serious concern in developed countries because

  • a. developing countries mainly suffer from Type-A malnutrition.
  • b. it is a major contributor to illness and death.
  • c. pharmaceutical companies are not producing drugs to treat this condition.
  • d. national surveys on malnutrition do not include newer micronutrient groups.
  • e.Not Attempted

39. Why are a large number of apparently healthy people deemed pre-ill?

  • a. They may have chronic degenerative diseases.
  • b. They do not know their own genetic risk factors which predispose them to diseases.
  • c. They suffer from Type-B malnutrition.
  • d. There is a lengthy latency period associated with chronically degenerative diseases.
  • e.Not Attempted

40. The author recommends micronutrient-repletion for large-scale treatment of chronic degenerative diseases because

  • a. it is relatively easy to manage.
  • b. micronutrient deficiency is the cause of these diseases.
  • c. it can overcome genetic risk factors.
  • d. it can compensate for other lifestyle factors.
  • e.Not Attempted

41. Tailoring micronutrient-based treatment plans to suit individual deficiency profiles is not necessary because

  • a. it very likely to give inconsistent or negative results.
  • b. it is a classic pharmaceutical approach not suited to micronutrients.
  • c. most people are consuming suboptimal amounts of safe-to-consume micronutrients.
  • d. it is not cost effective to do so.
  • e.Not Attempted

PASSAGE V

Fifty feet away three male lions lay by the road. They didn't appear to have a hair on their heads. Noting the color of their noses (leonine noses darken as they age, from pink to black), Craig estimated that they were six years old - young adults. "This is wonderful!" he said, after staring at them for several moments. "This is what we came to see. They really are maneless." Craig, a professor at the University of Minnesota, is arguably the leading expert on the majestic Serengeti lion, whose head is mantled in long, thick hair. He and Peyton West, a doctoral student who has been working with him in Tanzania, had never seen the Tsavo lions that live some 200 miles east of the Serengeti. The scientists had partly suspected that the maneless males were adolescents mistaken for adults by amateur observers. Now they knew better. The Tsavo research expedition was mostly Peyton's show. She had spent several years in Tanzania, compiling the data she needed to answer a question that ought to have been answered long ago: Why do lions have manes? It's the only cat, wild or domestic, that displays such ornamentation. In Tsavo she was attacking the riddle from the opposite angle. Why do its lions not have manes? (Some "maneless" lions in Tsavo East do have partial manes, but they rarely attain the regal glory of the Serengeti lions'.) Does environmental adaptation account for the trait? Are the lions of Tsavo, as some people believe, a distinct subspecies of their Serengeti cousins? The Serengeti lions have been under continuous observation for more than 35 years, beginning with George Schaller's pioneering work in the 1960s. But the lions in Tsavo, Kenya's oldest and largest protected ecosystem, have hardly been studied. Consequently, legends have grown up around them. Not only do they look different, according to the myths, they behave differently, displaying greater cunning and aggressiveness. "Remember too," Kenya: The Rough Guide warns, "Tsavo's lions have a reputation of ferocity." Their fearsome image became well-known in 1898, when two males stalled construction of what is now Kenya Railways by allegedly killing and eating 135 Indian and African laborers. A British Army officer in charge of building a railroad bridge over the Tsavo River, Lt. Col. J. H. Patterson, spent nine months pursuing the pair before he brought them to bay and killed them. Stuffed and mounted, they now glare at visitors to the Field Museum in Chicago. Patterson's account of the leonine reign of terror, The Man-Eaters of Tsavo, was an international best-seller when published in 1907. Still in print, the book has made Tsavo's lions notorious. That annoys some scientists. "People don't want to give up on mythology," Dennis King told me one day. The zoologist has been working in Tsavo off and on for four years. "I am so sick of this man-eater business. Patterson made a helluva lot of money off that story, but Tsavo'slions are no more likely to turn man-eater than lions from elsewhere."But tales of their savagery and wiliness don't all come from sensationalist authors looking to make a buck. Tsavo lions are generally larger than lions elsewhere, enabling them to take down the predominant prey animal in Tsavo, the Cape buffalo, one of the strongest, most aggressive animals of Earth. The buffalo don't give up easily: They often kill or severely injure an attacking lion, and a wounded lion might be more likely to turn to cattle and humans for food. And other prey is less abundant in Tsavo than in other traditional lion haunts. A hungry lion is more likely to attack humans. Safari guides and Kenya Wildlife Service rangers tell of lions attacking Land Rovers, raiding camps, stalking tourists. Tsavo is a tough neighborhood, they say, and it breeds tougher lions. But are they really tougher? And if so, is there any connection between their manelessness and their ferocity? An intriguing hypothesis was advanced two years ago by Gnoske and Peterhans: Tsavo lions may be similar to the unmanned cave lions of the Pleistocene. The Serengeti variety is among the most evolved of the species -the latest model, so to speak- while certain morphological differences in Tsavo lions (bigger bodies, smaller skulls, and maybe even lack of a mane) suggest that they are closer to the primitive ancestor of all lions. Craig and Peyton had serious doubts about this idea, but admitted that Tsavo lions pose a mystery to science.

42. The book Man-Eaters of Tsavo annoys some scientists because

  • a. it revealed that Tsavo lions are ferocious.
  • b. Patterson made a helluva lot of money from the book by sensationalism.
  • c. it perpetuated the bad name Tsavo lions had.
  • d. it narrated how two male Tsavo lions were killed.
  • e.Not Attempted

43. According to the passage, which of the following has NOT contributed to the popular image of Tsavo lions as savage creatures?

  • a. Tsavo lions have been observed to bring down one of the strongest and most aggressive animals—the Cape buffalo.
  • b. In contrast to the situation in traditional lion haunts, scarcity of non-buffalo prey in the Tsavo makes the Tsavo lions more aggressive.
  • c.The Tsavo lion is considered to be less evolved than the Serengeti variety.
  • d. Tsavo lions have been observed to attack vehicles as well as humans.
  • e.Not Attempted

44. The sentence which concludes the first paragraph, "Now they knew better", implies that:

  • a. The two scientists were struck by wonder on seeing maneless lions for the first time.
  • b. Though Craig was an expert on the Serengeti lion, now he also knew about the Tsavo lions.
  • c. Earlier, Craig and West thought that amateur observers had been mistaken.
  • d. Craig was now able to confirm that darkening of the noses as lions aged applied to Tsavo lions as well.
  • e.Not Attempted

45. Which of the following, if true, would weaken the hypothesis advanced by Gnoske and Peterhans most?

  • a. Craig and Peyton develop even more serious doubts about the idea that Tsavo lions are primitive.
  • b. The maneless Tsavo East lions are shown to be closer to the cave lions.
  • c. Pleistocene cave lions are shown to be far less violent than believed.
  • d. The morphological variations in body and skull size between the cave and Tsavo lions are found to be insignificant.
  • e.Not Attempted

Sub-Section III-B


Number of Questions : 5

DIRECTIONS for Questions 46 and 47:
The sentences given in each question, when properly sequenced, form a coherentparagraph. Each sentence is labeled with a letter. Choose the most logical order of sentences from among the given choices to construct a coherent paragraph.

46. A. Experts such as Larry Burns, head of research at GM, reckon that only such a full hearted leap will allow the world to cope with the mass motorisation that will one day come to China or India.
B. But once hydrogen is being produced from biomass or extracted from underground coal or made from water, using nuclear or renewable electricity, the way will be open for a huge reduction in carbon emissions from the whole system.
C. In theory, once all the bugs have been sorted out, fuel cells should deliver better total fuel economy than any existing engines.
D. That is twice as good as the internal combustion engine, but only five percentage points better than a diesel hybrid.
E. Allowing for the resources needed to extract hydrogen from hydrocarbon, oil, coal or gas, the fuel cell has an efficiency of 30%.

  • a. CEDBA
  • b. CEBDA
  • c. AEDBC
  • d. ACEBD
  • e.Not Attempted

47. A. But this does not mean that death was the Egyptians' only preoccupation.
B. Even papyri come mainly from pyramid temples.
C. Most of our traditional sources of information about the Old Kingdom are monuments of the rich like pyramids and tombs.
D. Houses in which ordinary Egyptians lived have not been preserved, and when most people died they were buried in simple graves.
E. We know infinitely more about the wealthy people of Egypt than we do about the ordinary people, as most monuments were made for the rich.

  • a. CDBEA
  • b. ECDAB
  • c. EDCBA
  • d. DECAB
  • e.Not Attempted

DIRECTIONS for Questions 48 to 50:
Four alternative summaries are given below each text. Choose the option that best captures the essence of the text.

48.Although almost all climate scientists agree that the Earth is gradually warming, they have long been of two minds about the process of rapid climate shifts within larger periods of change. Some have speculated that the process works like a giant oven or freezer, warming or cooling the whole planet at the same time. Others think that shifts occur on opposing schedules in the Northern and Southern Hemispheres, like exaggerated seasons. Recent research in Germany examining climate patterns in the Southern Hemisphere at the end of the last Ice Age strengthens the idea that warming and cooling occurs at alternate times in the two hemispheres. A more definitive answer to this debate will allow scientists to better predict when and how quickly the next climate shift will happen.

  • a. Scientists have been unsure whether rapid shifts in the Earth's climate happen all at once or on opposing schedules in different hemispheres; research will help find a definitive answer and better predict climate shifts in future.
  • b. Scientists have been unsure whether rapid shifts in the Earth's climate happen all at once or on opposing schedules in different hemispheres; finding a definitive answer will help them better predict climate shifts in future.
  • c. Research in Germany will help scientists find a definitive answer about warming and cooling of the Earth and predict climate shifts in the future in a better manner.
  • d. More research rather than debates on warming or cooling of the Earth and exaggerated "seasons in its hemispheres will help scientists in Germany predict climate changes better in future.
  • e.Not Attempted

49. Modern bourgeois society, said Nietzsche, was decadent and enfeebled - a victim of the excessive development of the rational faculties at the expense of will and instinct. Against the liberal-rationalist stress on the intellect, Nietzsche urged recognition of the dark mysterious world of instinctual desires -the true forces of life. Smother the will with excessive intellectualizing and you destroy the spontaneity that sparks cultural creativity and ignites a zest for living. The critical and theoretical outlook destroyed the creative instincts. For man's manifold potential to be realized, he must forego relying on the intellect and nurture again the instinctual roots of human existence.

  • a. Nietzsche urges the decadent and enfeebled modern society to forego intellect and give importance to creative instincts.
  • b. Nietzsche urges the decadent and enfeebled modern society to smother the will with excessive intellectualising and ignite a zest for living.
  • c. Nietzsche criticizes the intellectuals for enfeebling the modern bourgeois society by not nurturing man's creative instincts.
  • d. Nietzsche blames excessive intellectualization for the decline of modern society and suggests nurturing creative instincts instead.
  • e.Not Attempted

50. Local communities have often come in conflict with agents trying to exploit resources, at a faster pace, for an expanding commercial-industrial economy. More often than not, such agents of resource-intensification are given preferential treatment by the state, through the grant of generous long leases over mineral or fish stocks, for example, or the provision of raw material at an enormously subsidized price. With the injustice so compounded, local communities at the receiving end of this process have no recourse except direct action, resisting both the state and outside exploiters through a variety of protest techniques. These struggles might perhaps be seen as a manifestation of a new kind of class conflict.

  • a. A new kind of class conflict arises from preferential treatment given to agents of resource-intensification by the state which the local community sees as unfair.
  • b. The grant of long leases to agents of resource-intensification for an expanding commercial-industrial economy leads to direct protests from the local community, which sees it as unfair.
  • c. Preferential treatment given by the state to agents of resource-intensification for an expanding commercial-industrial economy exacerbates injustice to local communities and leads to direct protests from them, resulting in a new type of class conflict.
  • d. Local communities have no option but to protest against agents of resource-intensification and create a new type of class conflict when they are given raw material at subsidized prices for an expanding commercial-industrial economy.
  • e.Not Attempted

CAT2004 SECTION - DI

Instructions-CAT04
1.This test has three sections which examine various abilities. In all there are 123 questions. You will be given two hours to complete the test. In distributing the time over the three sections, please bear in mind that you need to demonstrate your competence in all three sections.
2. Directions for answering the questions are given before each group of questions. Read these directions carefully and answer the questions by darkening the appropriate circles on the Answer Sheet. There is only one correct answer to each question.
3. Each section carries 50 marks. Each section is divided into two sub-sections, A and B. For example, Section I is divided into two sub-sections. Sub-section I-A and Sub-section I-B. All questions in Sub-sections I-A and II-A carry one mark each. All questions in Sub-sections I-B, II-B and III-B carry two marks each. In Sub-section III-A, a group of 10 questions carries half a mark for each question; the remaining questions in Sub-section III-A carry one mark each. Wrong answers carry negative marks.
4. Do your rough work only on the Test Booklet and NOT on the Answer Sheet.
5. Follow the instructions of the invigilator. Candidates found violating the instructions will be disqualified.

SECTION I

Sub-Section I-A


Number of Questions : 26

DIRECTIONS for Questions 1 to 4:
Answer the questions on the basis of the information given below. Prof. Singh has been tracking the number of visitors to his homepage. His service provider has provided him with the following data on the country of origin of the visitors and the university they belong to:

1. University 1 can belong to

  • a. UK
  • b. Canada
  • c. Netherlands
  • d. USA
  • e.Not Attempted

2.To which country does University 5 belong?

  • a. India or Netherlands but not USA
  • b. India or USA but not Netherlands
  • c. Netherlands or USA but not India
  • d. India or USA but not UK
  • e.Not Attempted

3.Visitors from how many universities from UK visited Prof. Singh's homepage in the three days?

  • a. 1
  • b. 2
  • c. 3
  • d. 4
  • e.Not Attempted

4. Which among the listed countries can possibly host three of the eight listed universities?

  • a. None
  • b. Only UK
  • c. Only India
  • d. Both India and UK
  • e.Not Attempted

DIRECTIONS for Questions 5 to 8 :
Answer the questions on the basis of the information given below.

A study was conducted to ascertain the relative importance that employees in five different countries assigned to five different traits in their Chief Executive Officers. The traits were compassion (C), decisiveness (D), negotiation skills (N), public visibility (P), and vision (V). The level of dissimilarity between two countries is the maximum difference in the ranks allotted by the two countries to any of the five traits. The following table indicates the rank order of the five traits for each country

5. Three of the following four pairs of countries have identical levels of dissimilarity. Which pair is the odd one out?

  • a. Malaysia & China
  • b. China & Thailand
  • c. Thailand & Japan
  • d. Japan & Malaysia
  • e.Not Attempted

6. Which amongst the following countries is most dissimilar to India?

  • a. China
  • b. Japan
  • c. Malaysia
  • d. Thailand
  • e.Not Attempted

7. Which of the following countries is least dissimilar to India?

  • a. China
  • b. Japan
  • c. Malaysia
  • d. Thailand
  • e.Not Attempted

8. Which of the following pairs of countries are most dissimilar?

  • a. China & Japan
  • b. India & China
  • c. Malaysia & Japan
  • d. Thailand & Japan
  • e.Not Attempted

DIRECTIONS for Questions 9 to 12:
Answer the questions on the basis of the information given below.

The data points in the figure below represent monthly income and expenditure data of individual members of the Ahuja family (?), the Bose family ( ), the Coomar family ( ), and the Dubey family ( ). For these questions, savings is defined as

9. Which family has the highest average expenditure?

  • a. Ahuja
  • b. Bose
  • c. Coomar
  • d. Dubey
  • e.Not Attempted

10. Which family has the lowest average income?

  • a. Ahuja
  • b. Bose
  • c. Coomar
  • d. Dubey
  • e.Not Attempted

11. The highest amount of savings accrues to a member of which family?

  • a. Ahuja
  • b. Bose
  • c. Coomar
  • d. Dubey
  • e.Not Attempted

12. Which family has the lowest average savings?

  • a. Ahuja
  • b. Bose
  • c. Coomar
  • d. Dubey
  • e.Not Attempted

DIRECTIONS for Questions 13 to 16:
Answer the questions on the basis of the information given below.
The Dean's office recently scanned student results into the central computer system. When their character reading software cannot read something, it leaves that space blank. The scanner output reads as follows:

In the grading system, A, B, C, D, and F grades fetch 6, 4, 3, 2, and 0 grade points respectively. The Grade Point Average (GPA) is the arithmetic mean of the grade points obtained in the five subjects. For example Nisha's GPA is (6 + 2 + 4 + 6 + 0)/5 = 3.6.
Some additional facts are also known about the students' grades. These are
(a) Vipul obtained the same grade in Marketing as Apama obtained in Finance and Strategy.
(b) Fazal obtained the same grade in Strategy as Utkarsh did in Marketing.
(c) Tara received the same grade in exactly three courses.

13. In Operations, Tara could have received the same grade as

  • a. Ismet
  • b. Hari
  • c. Jagdeep
  • d. Manab
  • e.Not Attempted

14. What grade did Preeti obtain in Statistics?

  • a. A
  • b. B
  • c. C
  • d. D
  • e.Not Attempted

15. What grade did Utkarsh obtain in Finance?

  • a. B
  • b. C
  • c. D
  • d. F
  • e. Not Attempted

16. In Strategy, Gowri's grade point was higher than that obtained by

  • a. Fazal
  • b. Hari
  • c. Nisha
  • d. Rahul
  • e.Not Attempted

DIRECTIONS for Questions 17 to 20:
Answer the questions on the basis of the information given below.
Purana and Naya are two brands of kitchen mixer-grinders available in the local market. Purana is an old brand that was introduced in 1990, while Naya was introduced in 1997. For both these brands, 20% of the mixergrinders bought in a particular year are disposed off as junk exactly two years later. It is known that 10 Purana mixer-grinders were disposed off in 1997. The following figures show the number of Purana and Naya mixer-grinders in operation from 1995 to 2000, as at the end of the year.

17. How many Naya mixer-grinders were disposed off by the end of 2000?

  • a. 10
  • b. 16
  • c. 22
  • d. Cannot be determined from the data
  • e.Not Attempted

18. How many Naya mixer-grinders were purchased in 1999?

  • a. 44
  • b. 50
  • c. 55
  • d. 64
  • e.Not Attempted

19. How many Purana mixer-grinders were purchased in 1999?

  • a. 20
  • b. 23
  • c. 50
  • d. Cannot be determined from the data
  • e.Not Attempted

20. How many Purana mixer-grinders were disposed off in 2000?

  • a. 0
  • b. 5
  • c. 6
  • d. Cannot be determined from the data
  • e.Not Attempted

DIRECTIONS for Questions 21 to 26:
Each question is followed by two statements, A and B. Answer each question using the following instructions

Choose A if the question can be answered by using one of the statements alone but not by using the other statement alone.
Choose B if the question can be answered by using either of the statements alone.
Choose C if the question can be answered by using both statements together but not by either statement alone.
Choose D if the question cannot be answered on the basis of the two statements.

21. Ravi spent less than Rs.75 to buy one kilogram each of potato, onion, and gourd. Which one of the three vegetables bought was the costliest?
A: 2 kg potato and 1 kg gourd cost less than 1 kg potato and 2 kg gourd.
B: 1 kg potato and 2 kg onion together cost the same as 1 kg onion and 2 kg gourd.

  • a. A
  • b. B
  • c. C
  • d.D
  • e.Not Attempted

22. Tarak is standing 2 steps to the left of a red mark and 3 steps to the right of a blue mark. He tosses a coin. If it comes up heads, he moves one step to the right; otherwise he moves one step to the left. He keeps doing this until he reaches one of the two marks, and then he stops. At which mark does he stop?
A: He stops after 21 coin tosses.
B: He obtains three more tails than heads.

  • a. A
  • b. B
  • c. C
  • d. D
  • e.Not Attempted

23. Nandini paid for an article using currency notes of denominations Re.l, Rs.2, Rs.5, and Rs.10 using at least one note of each denomination. The total number of five and ten rupee notes used was one more than the total number of one and two rupee notes used. What was the price of the article?
A: Nandini used a total of 13 currency notes.
B: The price of the article was a multiple of Rs. 10.

  • a. A
  • b. B
  • c. C
  • d. D
  • e.Not Attempted

24. Four candidates for an award obtain distinct scores in a test. Each of the four casts a vote to choose the winner of the award.The candidate who gets the largest number of votes wins the award. In case of a tie in the voting process, the candidate with the highest score wins the award. Who wins the award?
A: The candidates with top three scores each vote for the top scorer amongst the other three.
B: The candidate with the lowest score votes for the player with the second highest score.

  • a. A
  • b. B
  • c. C
  • d. D
  • e.Not Attempted
25. In a class of 30 students, Rashmi secured the third rank among the girls, while her brother Kumar studyipg in the same class secured the sixth rank in the whole class. Between the two, who had a better overall rank?
A: Kumar was among the top 25% of the boys merit list in the class in which 60% were boys.
B: There were three boys among the top five rank holders, and three girls among the top ten rank holders.
  • a. A
  • b. B
  • c. C
  • d. D
  • e.Not Attempted

26. Zakib spends 30% of his income on his children’s education, 20% on recreation and 10% on healthcare. The corresponding percentages for Supriyo are 40%, 25%, and 13%. Who spends more on children’s education?
A: Zakib spends more on recreation than Supriyo.
B: Supriyo spends more on healthcare than Zakib.

  • a. A
  • b. B
  • c. C
  • d. D
  • e.Not Attempted

SECTION I

Sub-Section I-B


Number of Questions : 12

DIRECTIONS for Questions 27 to 30:
Answer the questions on the basis of the information given below.

Coach John sat with the score cards of Indian players from the 3 games in a one-day cricket tournament where the same set of players played for India and all the major batsmen got out. John summarized the batting performance through three diagrams, one for each game. In each diagram, the three outer triangles communicate the number of runs scored by the three top scorers from India, where K, R, S, V, and Y represent Kaif, Rahul, Saurav, Virender, and Yuvraj respectively. The middle triangle in each diagram denotes the percentage of total score that was scored by the top three Indian scorers in that game. No two players score the same number of runs in the same game. John also calculated two batting indices for each player based on his scores in the tournament; the R-index of a batsman is the difference between his highest and lowest scores in the 3 games while the M-index is the middle number, if his scores are arranged in a non-increasing order.

27. Which of the players had the best M-index from the tournament?

  • a. Rahul
  • b. Saurav
  • c. Virender
  • d. Yuvraj
  • e.Not Attempted

28. Among the players mentioned, who can have the lowest R-index from the tournament?

  • a. Only Kaif, Rahul or Yuvraj
  • b. Only Kaif or Rahul
  • c. Only Kaif or Yuvraj
  • d. Only Kaif
  • e.Not Attempted

29. For how many Indian players is it possible to calculate the exact M-index?

  • a. 0
  • b. 1
  • c. 2
  • d. More than 2
  • e.Not Attempted

30. How many players among those listed definitely scored less than Yuvraj in the tournament?

  • a. 0
  • b. 1
  • c. 2
  • d. More than 2
  • e.Not Attempted

DIRECTIONS for Questions 31 to 34:
Answer the questions on the basis of the information given below.


Twenty one participants from four continents (Africa, Americas, Australasia, and Europe) attended a United Nations conference. Each participant was an expert in one of four fields, labour, health, population studies, and refugee relocation. The following five facts about the participants are given.
(a) The number of labour experts in the camp was exactly half the number of experts in each of the three other categories.
(b) Africa did not send any labour expert. Otherwise, every continent, including Africa, sent at least one expert for each category.
(c) None of the continents sent more than three experts in any category.
(d) If there had been one less Australasian expert, then the Americas would have had twice as many experts as each of the other continents.
(e) Mike and Alfanso are leading experts of population studies who attended the conference. They are from Australasia.

31. Which of the following numbers cannot be determined from the information given?

  • a. Number of labour experts from the Americas.
  • b. Number of health experts from Europe.
  • c. Number of health experts from Australasia.
  • d. Number of experts in refugee relocation from Africa.
  • e.Not Attempted

32. Which of the following combinations is NOT possible?

  • a. 2 experts in population studies from the Americas and 2 health experts from Africa attended the conference.
  • b. 2 experts in population studies from the Americas and 1 health expert from Africa attended the conference.
  • c. 3 experts in refugee relocation from the Americas and 1 health expert from Africa attended the conference.
  • d. Africa and America each had 1 expert in population studies attending the conference.
  • e.Not Attempted

33. If Ramos is the lone American expert in population studies, which of the following is NOT true about the numbers of experts in the conference from the four continents?

  • a. There is one expert in health from Africa.
  • b. There is one expert in refugee relocation from Africa.
  • c. There are two experts in health from the Americas.
  • d. There are three experts in refugee relocation from the Americas.
  • e.Not Attempted

34. Alex, an American expert in refugee relocation, was the first keynote speaker in the conference. What can be inferred about the number of American experts in refugee relocation in the conference, excluding Alex?
i. At least one
ii. At most two

  • a. Only i and not ii
  • b. Only ii and not i
  • c. Both i and ii
  • d. Neither i nor ii
  • e.Not Attempted

DIRECTIONS for Questions 35 to 38:
Answer the questions on the basis of the information given below.

The year was 2006. All six teams in Pool A of World Cup hockey, play each other exactly once. Each win earns a team three points, a draw earns one point and a loss earns zero points. The two teams with the highest points qualify for the semifinals. In case of a tie, the team with the highest goal difference (Goal For - Goals Against) qualifies.
In the opening match, Spain lost to Germany. After the second round (after each team played two matches), the pool table looked as shown below.

In the third round, Spain played Pakistan, Argentina played Germany, and New Zealand played South Africa. All the third round matches were drawn. The following are some results from the fourth and fifth round matches (a) Spain won both the fourth and fifth round matches.
(b) Both Argentina and Germany won their fifth round matches by 3 goals to 0.
(c) Pakistan won both the fourth and fifth round matches by 1 goal to 0.

35. Which one of the following statements is true about matches played in the first two rounds?

  • a. Pakistan beat South Africa by 2 goals to 1.
  • b. Argentina beat Pakistan by 1 goal to 0.
  • c. Germany beat Pakistan by 2 goals to 1.
  • d. Germany beat Spain by 2 goals to 1.
  • e. Not Attempted

36. Which one of the following statements is true about matches played in the first two rounds?

  • a. Germany beat New Zealand by 1 goal to 0.
  • b. Spain beat New Zealand by 4 goals to 0.
  • c. Spain beat South Africa by 2 goals to 0.
  • d. Germany beat South Africa by 2 goals to 1.
  • e.Not Attempted

37. If Pakistan qualified as one of the two teams from Pool A, which was the other team that qualified?

  • a. Argentina
  • b. Germany
  • c. Spain
  • d. Cannot be determined
  • e.Not Attempted

38. Which team finished at the top of the pool after five rounds of matches?

  • a. Argentina
  • b. Germany
  • c. Spain
  • d. Cannot be determined
  • e.Not Attempted

CAT2005 SECTION - DI

Instructions
1. The test comprises of 50 questions. You should complete the test within 40 minutes.
2. There is only one correct answer to each question.
3. All questions carry four marks each.
4. Each wrong answer will attract a penalty of one mark.

SECTION III

Sub-Section III-A

Number of Questions : 10

Note: Questions 1 to 10 carry one mark each.

DIRECTIONS:
Answer questions 1 to 4 on the basis of the information given below.

A management institute was established on January 1, 2000 with 3, 4, 5, and 6 faculty members in the Marketing, Organisational Behaviour (OB), Finance, and Operations Management (OM) areas respectively, to start with. No faculty member retired or joined the institute in the first three months of the year 2000. In the next four years, the institute recruited one faculty member in each of the four areas. AH these new faculty members, who joined the institute subsequently over the years, were 25 years old at the time of their joining the institute. All of them joined the institute on April 1. During these four years, one of the faculty members retired at the age of 60. The following diagram gives the area-wise average age (in terms of number of completed years) of faculty members as on April 1 of 2000, 2001, 2002, and 2003

1. From which area did the faculty member retire?

  • a. Finance
  • b. Marketing
  • c. OB
  • d. OM
  • e.Not Attempted

2. Professors Naresh and Devesh, two faculty members in the Marketing area, who have been with the Institute since its inception, share a birthday, which falls on 20th November. One was born in 1947 and the other one in 1950. On April 1 2005, what was the age of the third faculty member who has been in the same area since inception?

  • a. 47
  • b. 50
  • c. 51
  • d. 52
  • e.Not Attempted

3. In which year did the new faculty member join the Finance area?

  • a. 2000
  • b. 2001
  • c. 2002
  • d. 2003
  • e.Not Attempted

4. What was the age of the new faculty member, who joined the OM area, as on April 1, 2003?

  • a. 25
  • b. 26
  • c. 27
  • d. 28
  • e.Not Attempted

DIRECTIONS for Q.5 to Q.7:
The table below reports annual statistics related to rice production in select states of India for a particular year.

5. Which two states account for the highest productivity of rice (tons produced per hectare of rice cultivation)?

  • a. Haryana and Punjab
  • b. Punjab and Andhra Pradesh
  • c. Andhra Pradesh and Haryana
  • d. Uttar Pradesh and Haryana
  • e.Not Attempted

6. How many states have a per capita production of rice (defined as total rice production divided by its population) greater than Gujarat?

  • a. 3
  • b. 4
  • c. 5
  • d. 6
  • e.Not Attempted

7. An intensive rice producing state is defined as one whose annual rice production per million of population is at least 400,000 tons. How many states are intensive rice producing states?

  • a. 5
  • b. 6
  • c. 7
  • d. 8
  • e.Not Attempted

DIRECTIONS for Q.8 to Q.10:
The table below reports the gender, designation and age-group of the employees in an organization. It also provides information on their commitment to projects coming up in the months of January (Jan), February (Feb), March (Mar) and April (Apr), as well as their interest in attending workshops on: Business Opportunities (BO), Communication Skills (CS), and E-Governance (EG).

M=Male, F= Female; Exe=Executive, Mgr=Manager, Dir=Director; Y=Young, I=In-between, O=Old For each workshop, exactly four employees are to be sent, of which at least two should be Females and at least one should be Young. No employee can be sent to a workshop in which he/she is not interested in. An employee cannot attend the workshop on

Communication Skills, if he/she is committed to internal projects in the month of January;
Business Opportunities, if he/she is committed to internal projects in the month of February;
E-governance, if he/she is committed to internal projects in the month of March.

8. Assuming that Parul and Hari arc attending the workshop on Communication Skills (CS), then which of the following employees can possibly attend the CS workshop?

  • a. Rahul and Yamini
  • b. Dinesh and Lavanya
  • c. Anshul and Yamini
  • d. Fatima and Zeena
  • e.Not Attempted

9. How many Executives (Exe) cannot attend more than one workshop?

  • a. 2
  • b. 3
  • c. 15
  • d. 16
  • e.Not Attempted

10. Which set of employees cannot attend any of the workshops?

  • a. Anshul, Charu, Eashwaran and Lavanya
  • b. Anshul, Bushkant, Gayatri and Urvashi
  • c. Charu, Urvashi, Bushkant and Mandeep
  • d. Anshul, Gayatri, Eashwaran and Mandeep
  • e.Not Attempted

SECTION III

Sub-Section III-B

Number of Questions : 20

Note: Questions 11 to 30 carry two marks each.

DIRECTIONS:
Answer questions 11 to 14 on the basis of the information given below.

In the table below is the listing of players, seeded from highest (#1) to lowest (#32), who are due to play in an Association of Tennis Players (ATP) tournament for women. This tournament has four knockout rounds before the final, i.e., first round, second round, quarterfinals, and semi-finals. In the first round, the highest seeded player plays the lowest seeded player (seed # 32) which is designated match No. 1 of first round; the 2nd seeded player plays the 31st seeded player which is designated match No. 2 of the first round, and so on. Thus, for instance, match No. 16 of first round is to be played between 16lh seeded player and the 17th seeded player. In the second round, the winner of match No. 1 of-first round plays the winner of match No. 16 of first round and is designated match No. I of second round. Similarly, the winner of match No, 2 of first round plays the winner of match No. 15 of first round, and is designated match No. 2 of second round. Thus, for instance, match No. 8 of the second round is to be played between the winner of match No. 8 of first round and the winner of match No. 9 of first round. The same pattern is followed for later rounds as well.

11. If there are no upsets (a lower seeded player beating a higher seeded player) in the first round, and only match Nos. 6, 7, and 8 of the second round result in upsets, then who would meet Lindsay Davenport in quarter finals, in case Davenport reaches quarter finals?

  • a. Justine Henin
  • b. Nadia Petrova
  • c. Patty Schnyder
  • d. Venus Williams
  • e.Not Attempted

12. If Elena Dementieva and Serena Williams lose in the second round, while Justine Henin and Nadia Petrova make it to the semifinals, then who would play Maria Sharapova in the quarterfinals, in the event Sharapova reaches quarterfinals?

  • a. Dinara Safina
  • b. Justine Henin
  • c. Nadia Petrova
  • d. Patty Schnyder
  • e.Not Attempted

13. If, in the first round, all even numbered matches (and none of the odd numbered ones) result in upsets, and there are no upsets in the second round, then who could be the lowest seeded player facing Maria Sharapova in semi-finals?

  • a. Anastasia Myskina
  • b. Flavia Pennetta
  • c. Nadia Petrova
  • d. Svetlana Kuznetsova
  • e.Not Attempted

14. If the top eight seeds make it to the quarterfinals, then who, amongst the players listed below, would definitely not play against Maria Sharapova in the final, in case Sharapova reaches the final?

  • a. Amelie Mauresmo
  • b. Elena Dementieva
  • c. Kim Clijsters
  • d. Lindsay Davenport
  • e.Not Attempted

DIRECTIONS:
Answer questions 15 to 18 on the basis of the information given below.

Venkat, a stockbroker, invested a part of his money in the stock of four companies - A, B, C and D. Each of these companies belonged to different industries, viz., Cement, Information Technology (IT), Auto, and Steel, in no particular order. At the time of investment, the price of each stock was Rs.100. Venkat purchased only one stock of each of these companies. He was expecting returns of 20%, 10%, 30%, and 40% from the stock of companies A, B, C and D, respectively. Returns arc defined as the change in the value of the stock after one year, expressed as a percentage of the initial value. During the year, two of these companies announced extraordinarily good results. One of these two companies belonged to the Cement or the IT industry, while the other one belonged to either the Steel or the Auto industry. As a result, the returns on the stocks of these two companies were higher than the initially expected returns. For the company belonging to the Cement or the IT industry with extraordinarily good results, the returns were twice that of the initially expected returns. For the company belonging to the Steel or the Auto industry, the returns on announcement of extraordinarily good results were only one and a half times that of the initially expected returns. For the remaining two companies, which did not announce extraordinarily good results, the returns realized during the year were the same as initially expected.

15. What is the minimum average return Venkat would have earned during the year?

  • a. 30%
  • b. 31.25%
  • c. 32.5%
  • d. Cannot be determined
  • e. Not Attempted

16. If Venkat earned a 35% return on average during the year, then which of these statements would necessarily be true?
I . Company A belonged either to Auto or to Steel Industry.
II. Company B did not announce extraordinarily good results.
III. Company A announced extraordinarily good results.
IV. Company D did not announce extraordinarily good results.

  • a. I and II only
  • b. II and III only
  • c. III and IV only
  • d. II and IV only
  • e.Not Attempted

17. If Venkat earned a 38.75% return on average during the year, then which of these statement(s) would necessarily be true?
I . Company C belonged either to Auto or to Steel Industry.
II. Company D belonged either to Auto or to Steel Industry.
III. Company A announced extraordinarily good results.
IV. Company B did not announce extraordinarily good results.

  • a. I and II only
  • b. II and III only
  • c. I and IV only
  • d. II and IV only
  • e.Not Attempted

18. If Company C belonged to the Cement or the IT industry and did announce extraordinarily good results, then which of these statement(s) would necessarily be true?
I . Venkat earned not more than 36.25% return on average.
II. Venkat earned not less than 33.75% return on average.
III. If Venkat earned 33.75% return on average, Company A announced extraordinarily good results.
IV. If Venkat earned 33.75% return on average, Company B belonged either to Auto or to Steel Industry.

  • a. I and II only
  • b. II and IV only
  • c. II and III only
  • d. III and IV only
  • e.Not Attempted
DIRECTIONS:
Answer questions 19 to 22 on the basis of the information given below.

The year is 2089. Beijing, London, New York, and Paris are in contention lo host the 2096 Olympics. The eventual winner is determined through several rounds of voting by members of the IOC with each member representing a different city. All the four cities in contention are also represented in IOC.
In any round of voting, the city receiving the lowest number of votes in that round gets eliminated. The survivor after the last round of voting gets to host the event.
A member is allowed to east votes for at most two different cities in all rounds of voting combined. (Hence, a member becomes ineligible to cast a vote in a given round if both the cities (s)he voted for in earlier rounds are out of contention in that round of voting.)
A member is also ineligible to cast a vote in a round if the city (s)he represents is in contention in that round of voting.
As long as the member is eligible, (s)he must vote and vote for only one candidate city in any round of voting.

The following incomplete table shows the information on cities that received the maximum and minimum votes in different rounds, the number of votes cast in their favour, and the total votes that were cast in those rounds.

It is also known that All those who voted for London and Paris in round 1, continued to vote for the same cities in subsequent rounds as long as these cities were in contention. 75% of those who voted for Beijing in round 1, voted for Beijing in round 2 as well.
Those who voted for New York in round 1, voted either for Beijing or Paris in round 2.
The difference in votes cast for the two contending cities in the last round was 1.
50% of those who voted for Beijing in round 1, voted for Paris in round 3.

19. What percentage of members from among those who voted for New York in round I, voted for Beijing in round 2?

  • a. 33.33
  • b. 50
  • c. 67.7
  • d. 75
  • e.Not Attempted

20. What is the number of votes cast for Paris in round 1?

  • a. 16
  • b. 18
  • c. 22
  • d. 24
  • e.Not Attempted

21. What percentage of members from among those who voted for Beijing in round 2 and were eligible to vote in round 3, voted for London?

  • a. 33.33
  • b. 38.10
  • c. 50
  • d. 66.67
  • e.Not Attempted

22. Which of the following statements must be true?
a. IOC member from New York must have voted for Paris in round 2.
b. IOC member from Beijing voted for London in round 3.

  • a. Only a
  • b. Only b
  • c. Both a and b
  • d. Neither a nor b
  • e.Not Attempted

DIRECTIONS for Q.23 to Q.26:
The table below presents the revenue (in million rupees) of four firms in three states. These firms, Honest Ltd., Aggressive Ltd., Truthful Ltd. and Profitable Ltd. arc disguised in the table as A, B, C and D, in no particular order.


Further, it is known that
In the state of MP, Truthful Ltd. has the highest market share.
Aggressive Ltd.'s aggregate revenue differs from Honest Ltd.'s by Rs.5 million.

23. What can be said regarding the following two statements?
Statement 1: Profitable Ltd. has the lowest share in MP market.
Statement 2: Honest Ltd.'s total revenue is more than Profitable Ltd.

  • a. If Statement 1 is true then Statement 2 is necessarily true.
  • b. If Statement 1 is true then Statement 2 is necessarily false.
  • c. Both Statement 1 and Statement 2 are true.
  • d. Neither Statement 1 nor Statement 2 is true.
  • e.Not Attempted

24. What can be said regarding the following two statements?
Statement 1: Aggressive Ltd.'s lowest revenues are from MP.
Statement 2: Honest Ltd.'s lowest revenues are from Bihar.

  • a. If Statement 2 is true then Statement 1 is necessarily false.
  • b. If Statement 1 is false then Statement 2 is necessarily true.
  • c. If Statement 1 is true then Statement 2 is necessarily true.
  • d. None of the above.
  • e.Not Attempted
25. What can be said regarding the following two statements?
Statement 1: Honest Ltd. has the highest share in the UP market.
Statement 2: Aggressive Ltd. has the highest share in the Bihar market
  • a. Both statements could be true.
  • b. At least one of the statements must be true.
  • c. At most one of the statements is true.
  • d. None of the above
  • e.Not Attempted

26. If Profitable Ltd.'s lowest revenue is from UP, then which of the following is true?

  • a. Truthful Ltd.'s lowest revenues are from MP.
  • b. Truthful Ltd.'s lowest revenues are from Bihar.
  • c. Truthful Ltd.'s lowest revenues are from UP.
  • d. No definite conclusion is possible.
  • e.Not Attempted

DIRECTIONS for Q.27 to Q.30:
Help Distress (HD) is an NGO involved in providing assistance to people suffering from natural disasters. Currently, it has 37 volunteers. They are involved in three projects: Tsunami Relief (TR) in Tamil Nadu, Flood Relief (FR) in Maharashtra, and Earthquake Relief (ER) in Gujarat. Each volunteer working with Help Distress has to be involved in at least one relief work project.
A Maximum number of volunteers are involved in the FR project. Among them, the number of volunteers involved in FR project alone is equal to the volunteers having additional involvement in the ER project.
The number of volunteers involved in the ER project alone is double the number of volunteers involved in all the three projects.
17 volunteers are involved in the TR project.
The number of volunteers involved in the TR project alone is one less than the number of volunteers involved in ER project alone.
Ten volunteers involved in the TR project are also involved in at least one more project.

27. Based on the information given above, the minimum number of volunteers involved in both FR and TR projects, but not in the ER project is:

  • a. 1
  • b. 3
  • c. 4
  • d. 5
  • e.Not Attempted

28. Which of the following additional information would enable to find the exact number of volunteers involved in various projects?

  • a. Twenty volunteers are involved in FR.
  • b. Four volunteers are involved in all the three projects.
  • c. Twenty three volunteers are involved in exactly one project.
  • d. No need for any additional information.
  • e.Not Attempted

29. After some time, the volunteers who were involved in all the three projects were asked to withdraw from one project. As a result, one of the volunteers opted out of the TR project, and one opted out of the ER project, while the remaining ones involved in all the three projects opted out of the FR project. Which of the following statements, then, necessarily follows?.

  • a. The lowest number of volunteers is now in TR project.
  • b. More volunteers are now in FR project as compared to ER project.
  • c. More volunteers are now in TR project as compared to ER project.
  • d. None of the above
  • e.Not Attempted

30. After the withdrawal of volunteers, as indicated in Question 89, some new volunteers joined the NGO. Each one of them was allotted only one project in a manner such that, the number of volunteers working in one project alone for each of the three projects became identical. At that point, it was also found that the number of volunteers involved in FR and ER projects was the same as the number of volunteers involved in TR and ER projects. Which of the projects now has the highest number of volunteers?

  • a. ER
  • b. FR
  • c. TR
  • d. Cannot be determined
  • e.Not Attempted

CAT2005 SECTION - MATH

Instructions
1. The test comprises of 30 questions. You should complete the test within 40 minutes.
2. There is only one correct answer to each question.
3. All questions carry four marks each.
4. Each wrong answer will attract a penalty of one mark.

SECTION I

Sub-Section I-A

Number of Questions : 10

Note: Questions 1 to 10 carry one mark each.

DIRECTIONS for Q1 to Q5: Answer the questions independently of each other.

1. If x = (163 + 173 + 183 + 193), then x divided by 70 leaves a remainder of

  • a. 0
  • b. 1
  • c. 69
  • d. 35
  • e.Not Attempted

2. A chemical plant has four tanks (A, B, C and D), each containing 1000 litres of a chemical. The chemical is being pumped from one tank to another as follows:
From A to B @ 20 litres/minute
From C to A @ 90 litres/minute
From A to D @ 10 litres/minute
From C to D @ 50 litres/minute
From B to C @ 100 litres/minute
From D to B @ 110 litres/minute
Which tank gets emptied first and how long does it take (in minutes) to get empty after pumping starts?

  • a. A, 16.66
  • b. C, 20
  • c. D, 20
  • d. D, 25
  • e.Not Attempted

3. Two identical circles intersect so that their centres, and the points at which they intersect, form a square of side 1 cm. The area in sq. cm of the portion that is common to the two circles is

  • a. π/4
  • b. π/2 - 1
  • c. π/5
  • d. √2 - 1
  • e.Not Attempted

4. A jogging park has two identical circular tracks touching each other, and a rectangular track enclosing the two circles. The edges of the rectangles are tangential to the circles. Two friends, A and B, start jogging simultaneously from the point where one of the circular tracks touches the smaller side of the rectangular track. A jogs along the rectangular track, while B jogs along the two circular tracks in a figure of eight. Approximately, how much faster than A does B have to run, so that they take the same time to return to their starting point?

  • a. 3.88%
  • b. 4.22%
  • c. 4.44%
  • d. 4.72%
  • e.Not Attempted

5. In a chess competition involving some boys and girls of a school, every student had to play exactly one game with every other student. It was found that in 45 games both the players were girls, and in 190 games both were boys. The number of games in which one player was a boy and the other was a girl is

  • a. 200
  • b. 216
  • c. 235
  • d. 256
  • e.Not Attempted

DIRECTIONS for Q.6 & Q.7:
Answer the questions on the basis of the information given below.

Ram and Shyam run a race between points A and B, 5 km apart. Ram starts at 9 a.m. from A at a speed of 5 km/ hr, reaches B, and returns to A at the same speed. Shyam starts at 9:45 a.m. from A at a speed of 10 km/hr, reaches B and comes back to A at the same speed

6. At what time do Ram and Shyam first meet each other?

  • a. 10 a.m.
  • b. 10:10 a.m.
  • c. 10:20 a.m.
  • d. 10:30 a.m.
  • e.Not Attempted

7. At what time does Shyam overtake Ram?

  • a. 10:20 a.m.
  • b. 10:30 a.m.
  • c. 10:40 a.m.
  • d. 10:50 a.m.
  • e.Not Attempted

DIRECTIONS for Q.8 to Q.10:
Answer the questions independently of each other

8. if R=(3065 - 2965)/(3064 + 2964)

  • a. 0 < R ≤ 0.1
  • b. 0.1 < R ≤ 0.5
  • c. 0.5 < R ≤ 1.0
  • d. R > 1.0
  • e.Not Attempted

9. What is the distance in cm between two parallel chords of lengths 32 cm and 24 cm in a circle of radius 20 cm?

  • a. 1 or 7
  • b. 2 or 14
  • c. 3 or 21
  • d. 4 or 28
  • e.Not Attempted

10. For which value of k does the following pair of equations yield a unique solution for x such that the solution is positive? x2 - y2 = 0
(x - k)2 + y2 = 1

  • a. 2
  • b. 0
  • c. √2
  • d. -√2
  • e.Not Attempted

SECTION I

Sub-Section I-B

Number of Questions : 20

Note: Questions 11 to 30 carry two marks each.

Directions for Questions 11 to 30: Answer the questions independently of each other.

11. Let n! = 1 x 2 x 3 x ... x n for integer n π 1. If p = 1! (2 x 2!) + (3 x 3!) + . . . + (10 x 10!), then p + 2 when divided by 11! leaves a remainder of

  • a. 10
  • b. 0
  • c. 7
  • d. 1
  • e.Not Attempted

12. Consider a triangle drawn on the X-Y plane with its three vertices at (41, 0), (0, 41) and (0, 0), each vertex being represented by its (X, Y) coordinates. The number of points with integer coordinates inside the triangle (excluding all the points on the boundary) is

  • a. 780
  • b. 800
  • c. 820
  • d. 741
  • e.Not Attempted

13. The digits of a three-digit number A are written in the reverse order to form another three-digit number B. If B > A and B-A is perfectly divisible by 7, then which of the following is necessarily true?

  • a. 100 < A < 299
  • b. 106 < A < 305
  • c. 112 < A < 311
  • d. 118 < A < 317
  • e.Not Attempted

14. If a1 = 1 and an+1 - 3an + 2 = 4n for every positive integer n, then a100 equals

  • a. 399 - 200
  • b. 399 + 200
  • c. 3100 - 200
  • d. 3100 + 200
  • e.Not Attempted

15. Let S be the set of five-digit numbers formed by the digits 1, 2, 3, 4 and 5, using each digit exactly once such that exactly two odd positions are occupied by odd digits. What is the sum of the digits in the rightmost position of the numbers in S?

  • a. 228
  • b. 216
  • c. 294
  • d. 192
  • e. Not Attempted

16. The rightmost non-zero digit of the number 302720 is

  • a. 1
  • b. 3
  • c. 7
  • d. 9
  • e.Not Attempted

17. Four points A, B, C and D lie on a straight line in the X-Y plane, such that AB = BC = CD, and the length of AB is 1 metre. An ant at A wants to reach a sugar particle at D. But there are insect repellents kept at points B and C. The ant would not go within one metre of any insect repellent. The minimum distance in metres the ant must traverse to reach the sugar particle is

  • a. 3 √2
  • b. 1 + π
  • c. 4π/ 3
  • d. 5
  • e.Not Attempted

18. If x ≥ y and y > 1, then the value of the expression logx(x/y)+logy(y/x) can never be

  • a. -1
  • b. -0.5
  • c. 0
  • d. 1
  • e.Not Attempted

19. For a positive integer n, let pn denote the product of the digits of n, and sn denote the sum of the digits of n. The number of integers between 10 and 1000 for which pn + sn = n is

  • a. 81
  • b. 16
  • c. 18
  • d. 9
  • e.Not Attempted

20. Rectangular tiles each of size 70 cm by 30 cm must be laid horizontally on a rectangular floor of size 110 cm by 130 cm, such that the tiles do not overlap. A tile can be placed in any orientation so long as its edges are parallel to the edges of the floor. No tile should overshoot any edge of the floor. The maximum number of tiles that can be accommodated on the floor is

  • a. 4
  • b. 5
  • c. 6
  • d. 7
  • e.Not Attempted

21. In the X-Y plane, the area of the region bounded by the graph of |x + y| + |x - y| = 4 is

  • a. 8
  • b. 12
  • c. 16
  • d. 20
  • e.Not Attempted

22. In the following figure, the diameter of the circle is 3 cm. AB and MN are two diameters such that MN is perpendicular to AB. In addition, CG is perpendicular to AB such that AE:EB = 1:2, and DF is perpendicular to MN such that NL:LM = 1:2. The length of DH in cm is

  • a. 2√2-1
  • b. (2√2-1)/2
  • c. (3√2-1)/2
  • d. (2√2-1)/3
  • e.Not Attempted

23. Consider the triangle ABC shown in the following figure where BC = 12 cm, DB = 9 cm, CD = 6 cm and ÐBCD = ÐBAC.

What is the ratio of the perimeter of the triangle ADC to that of the triangle BDC?

  • a. 7/9
  • b. 8/9
  • c. 6/9
  • d. 5/9
  • e.Not Attempted

24. P, Q, S, and R are points on the circumference of a circle of radius r, such that PQR is an equilateral triangle and PS is a diameter of the circle. What is the perimeter of the quadrilateral PQSR?

  • a. 2r(1+√ 3)
  • b. 2r(2+√ 3)
  • c. r(1+√ 5)
  • d. 2r+√ 3)
  • e.Not Attempted
25. Let S be a set of positive integers such that every element n of S satisfies the conditions
a) 1000 = n = 1200
b) every digit in n is odd
Then how many elements of S are divisible by 3?
  • a. 9
  • b. 10
  • c. 11
  • d. 12
  • e.Not Attempted

26. Let . Then x equals

  • a. 3
  • b. (√13-1)/2
  • c. (√13+1)/2
  • d. √13
  • e.Not Attempted

27. Let g(x) be a function such that g(x + l) + g(x - l) = g(x) for every real x. Then for what value of p is the relation g(x + p) = g(x) necessarily true for every real x?

  • a. 5
  • b. 3
  • c. 2
  • d. 6
  • e.Not Attempted

28. A telecom service provider engages male and female operators for answering 1000 calls per day. A male operator can handle 40 calls per day whereas a female operator can handle 50 calls per day. The male and the female operators get a fixed wage of Rs.250 and Rs.300 per day respectively. In addition, a male operator gets Rs.15 per call he answers and a female operator gets Rs.10 per call she answers. To minimize the total cost, how many male operators should the service provider employ assuming he has to employ more than 7 of the 12 female operators available for the job?

  • a. 15
  • b. 14
  • c. 12
  • d. 10
  • e.Not Attempted

29. Three Englishmen and three Frenchmen work for the same company. Each of them knows a secret not known to others. They need to exchange these secrets over person-to-person phone calls so that eventually each person knows all six secrets. None of the Frenchmen knows English, and only one Englishman knows French. What is the minimum number of phone calls needed for the above purpose?.

  • a. 5
  • b. 10
  • c. 9
  • d. 15
  • e.Not Attempted

30. A rectangular floor is fully covered with square tiles of identical size. The tiles on the edges are white and the tiles in the interior are red. The number of white tiles is the same as the number of red tiles. A possible value of the number of tiles along one edge of the floor is

  • a. 10
  • b. 12
  • c. 14
  • d. 16
  • e.Not Attempted

Your performance in CAT2005 MATH-SECTION

Q1. "; if($_POST["answer1"] == 'e1') { $score1=0; echo "You have not attempted this question. " ; echo "
The correct answer is: $ans1
" ; } else { $attempted=$attempted+1; if($time1 == $_POST["answer1"]) { $score1 = 4;//CAT equvalent mark 4 $correct=$correct+1; echo "You have correctly answered this question. " ; } else { $score1= -1;//CAT equivalent mark -1 echo "Your answer was wrong for this question. " ; echo "
The correct answer is: $ans1
" ; } } //Ansewr 2// echo "

Q2. "; if($_POST["answer2"] == 'e2') { $score2=0; echo "You have not attempted this question. " ; echo "
The correct answer is: $ans2
" ; } else { $attempted=$attempted+1; if($time2 == $_POST["answer2"]) { $score2 = 4; // CAT equivalent mark 1 $correct=$correct+1; echo "You have correctly answered this question " ; } else { $score2 = -1; //CAT equivalent mark -0.33 echo "Your answer was wrong for this question" ; echo "
The correct answer is: $ans2
" ; } } //Ansewr 3// echo "

Q3. "; if($_POST["answer3"] == 'e3') { $score3=0; echo "You have not attempted this question. " ; echo "
The correct answer is: $ans3
" ; } else { $attempted=$attempted+1; if($time3 == $_POST["answer3"]) { $score3 = 4;//CAT equivalt 0.5 marks $correct=$correct+1; echo "You have correctly answered this question " ; } else { $score3= -1;//CAT equivalent score -0.166 echo "Your answer was wrong for this question" ; echo "
The correct answer is: $ans3
" ; } } //Ansewr 4// echo "

Q4. "; if($_POST["answer4"] == 'e4') { $score4=0; echo "You have not attempted this question. " ; echo "
The correct answer is: $ans4
" ; } else { $attempted=$attempted+1; if($time4 == $_POST["answer4"]) { $score4 = 4;//CAT equvalent mark 2 $correct=$correct+1; echo "You have correctly answered this question. " ; } else { $score4= -1;//CAT equivalent mark -0.66 echo "Your answer was wrong for this question. " ; echo "
The correct answer is: $ans4
" ; } } //Ansewr 5// echo "

Q5. "; if($_POST["answer5"] == 'e5') { $score5=0; echo "You have not attempted this question. " ; echo "
The correct answer is: $ans5
" ; } else { $attempted=$attempted+1; if($time5 == $_POST["answer5"]) { $score5 = 4; // CAT equivalent mark 1 $correct=$correct+1; echo "You have correctly answered this question " ; } else { $score5 = -1; //CAT equivalent mark -0.33 echo "Your answer was wrong for this question" ; echo "
The correct answer is: $ans5
" ; } } //Ansewr 6// echo "

Q6. "; if($_POST["answer6"] == 'e6') { $score6=0; echo "You have not attempted this question. " ; echo "
The correct answer is: $ans6
" ; } else { $attempted=$attempted+1; if($time6 == $_POST["answer6"]) { $score6 = 4;//CAT equivalt 0.5 marks $correct=$correct+1; echo "You have correctly answered this question " ; } else { $score6= -1;//CAT equivalent score -0.166 echo "Your answer was wrong for this question" ; echo "
The correct answer is: $ans6
" ; } } //Ansewr 7// echo "

Q7. "; if($_POST["answer7"] == 'e7') { $score7=0; echo "You have not attempted this question. " ; echo "
The correct answer is: $ans7
" ; } else { $attempted=$attempted+1; if($time7 == $_POST["answer7"]) { $score7 = 4;//CAT equvalent mark 2 $correct=$correct+1; echo "You have correctly answered this question. " ; } else { $score7= -1;//CAT equivalent mark -0.66 echo "Your answer was wrong for this question. " ; echo "
The correct answer is: $ans7
" ; } } //Ansewr 8// echo "

Q8. "; if($_POST["answer8"] == 'e8') { $score8=0; echo "You have not attempted this question. " ; echo "
The correct answer is: $ans8
" ; } else { $attempted=$attempted+1; if($time8 == $_POST["answer8"]) { $score8 = 4; // CAT equivalent mark 1 $correct=$correct+1; echo "You have correctly answered this question " ; } else { $score8 = -1; //CAT equivalent mark -0.33 echo "Your answer was wrong for this question" ; echo "
The correct answer is: $ans8
" ; } } //Ansewr 9// echo "

Q9. "; if($_POST["answer9"] == 'e9') { $score9=0; echo "You have not attempted this question. " ; echo "
The correct answer is: $ans9
" ; } else { $attempted=$attempted+1; if($time9 == $_POST["answer9"]) { $correct=$correct+1; $score9 = 4;//CAT equivalt 0.5 marks echo "You have correctly answered this question " ; } else { $score9= -1;//CAT equivalent score -0.166 echo "Your answer was wrong for this question" ; echo "
The correct answer is: $ans9
" ; } } //Ansewr 10// echo "

Q10. "; if($_POST["answer10"] == 'e10') { $score10=0; echo "You have not attempted this question. " ; echo "
The correct answer is: $ans10
" ; } else { $attempted=$attempted+1; if($time10 == $_POST["answer10"]) { $score10 = 4; // CAT equivalent mark 1 $correct=$correct+1; echo "You have correctly answered this question " ; } else { $score10 = -1; //CAT equivalent mark -0.33 echo "Your answer was wrong for this question" ; echo "
The correct answer is: $ans10
" ; } } //Ansewr 11// echo "

Q11. "; if($_POST["answer11"] == 'e11') { $score11=0; echo "You have not attempted this question. " ; echo "
The correct answer is: $ans11
" ; } else { $attempted=$attempted+1; if($time11 == $_POST["answer11"]) { $score11 = 8; // CAT equivalent mark 1 $correct=$correct+1; echo "You have correctly answered this question " ; } else { $score11 = -2; //CAT equivalent mark -0.33 echo "Your answer was wrong for this question" ; echo "
The correct answer is: $ans11
" ; } } //Ansewr 12// echo "

Q12. "; if($_POST["answer12"] == 'e12') { $score12=0; echo "You have not attempted this question. " ; echo "
The correct answer is: $ans12
" ; } else { $attempted=$attempted+1; if($time12 == $_POST["answer12"]) { $score12 = 8;//CAT equivalt 0.5 marks $correct=$correct+1; echo "You have correctly answered this question " ; } else { $score12= -2;//CAT equivalent score -0.166 echo "Your answer was wrong for this question" ; echo "
The correct answer is: $ans12
" ; } } //Ansewr 13 echo "

Q13. "; if($_POST["answer13"] == 'e13') { $score13=0; echo "You have not attempted this question. " ; echo "
The correct answer is: $ans13
" ; } else { $attempted=$attempted+1; if($time13 == $_POST["answer13"]) { $score13 = 8;//CAT equvalent mark 2 $correct=$correct+1; echo "You have correctly answered this question. " ; } else { $score13= -2;//CAT equivalent mark -0.66 echo "Your answer was wrong for this question. " ; echo "
The correct answer is: $ans13
" ; } } //Ansewr 14// echo "

Q14. "; if($_POST["answer14"] == 'e14') { $score14=0; echo "You have not attempted this question. " ; echo "
The correct answer is: $ans14
" ; } else { $attempted=$attempted+1; if($time14 == $_POST["answer14"]) { $score14 = 8; // CAT equivalent mark 1 $correct=$correct+1; echo "You have correctly answered this question " ; } else { $score14 = -2; //CAT equivalent mark -0.33 echo "Your answer was wrong for this question" ; echo "
The correct answer is: $ans14
" ; } } //Ansewr 15// echo "

Q15. "; if($_POST["answer15"] == 'e15') { $score15=0; echo "You have not attempted this question. " ; echo "
The correct answer is: $ans15
" ; } else { $attempted=$attempted+1; if($time15 == $_POST["answer15"]) { $score15 = 8;//CAT equivalt 0.5 marks $correct=$correct+1; echo "You have correctly answered this question " ; } else { $score15= -2;//CAT equivalent score -0.166 echo "Your answer was wrong for this question" ; echo "
The correct answer is: $ans15
" ; } } //Ansewr 16// echo "

Q16. "; if($_POST["answer16"] == 'e16') { $score16=0; echo "You have not attempted this question. " ; echo "
The correct answer is: $ans16
" ; } else { $attempted=$attempted+1; if($time16 == $_POST["answer16"]) { $score16 = 8;//CAT equvalent mark 2 $correct=$correct+1; echo "You have correctly answered this question. " ; } else { $score16= -2;//CAT equivalent mark -0.66 echo "Your answer was wrong for this question. " ; echo "
The correct answer is: $ans16
" ; } } //Ansewr 17// echo "

Q17. "; if($_POST["answer17"] == 'e17') { $score17=0; echo "You have not attempted this question. " ; echo "
The correct answer is: $ans17
" ; } else { $attempted=$attempted+1; if($time17 == $_POST["answer17"]) { $score17 = 8; // CAT equivalent mark 1 $correct=$correct+1; echo "You have correctly answered this question " ; } else { $score17 = -2; //CAT equivalent mark -0.33 echo "Your answer was wrong for this question" ; echo "
The correct answer is: $ans17
" ; } } //Ansewr 18// echo "

Q18. "; if($_POST["answer18"] == 'e18') { $score18=0; echo "You have not attempted this question. " ; echo "
The correct answer is: $ans18
" ; } else { $attempted=$attempted+1; if($time18 == $_POST["answer18"]) { $score18 = 8;//CAT equivalt 0.5 marks $correct=$correct+1; echo "You have correctly answered this question " ; } else { $score18= -2;//CAT equivalent score -0.166 echo "Your answer was wrong for this question" ; echo "
The correct answer is: $ans18
" ; } } //Ansewr 19// echo "

Q19. "; if($_POST["answer19"] == 'e19') { $score19=0; echo "You have not attempted this question. " ; echo "
The correct answer is: $ans19
" ; } else { $attempted=$attempted+1; if($time19 == $_POST["answer19"]) { $score19 = 8;//CAT equvalent mark 2 $correct=$correct+1; echo "You have correctly answered this question. " ; } else { $score19= -2;//CAT equivalent mark -0.66 echo "Your answer was wrong for this question. " ; echo "
The correct answer is: $ans19
" ; } } //Ansewr 20// echo "

Q20. "; if($_POST["answer20"] == 'e20') { $score20=0; echo "You have not attempted this question. " ; echo "
The correct answer is: $ans20
" ; } else { $attempted=$attempted+1; if($time20 == $_POST["answer20"]) { $score20 = 8; // CAT equivalent mark 1 $correct=$correct+1; echo "You have correctly answered this question " ; } else { $score20 = -2; //CAT equivalent mark -0.33 echo "Your answer was wrong for this question" ; echo "
The correct answer is: $ans20
" ; } } //Ansewr 21// echo "

Q21. "; if($_POST["answer21"] == 'e21') { $score21=0; echo "You have not attempted this question. " ; echo "
The correct answer is: $ans21
" ; } else { $attempted=$attempted+1; if($time21 == $_POST["answer21"]) { $score21 = 8; // CAT equivalent mark 1 $correct=$correct+1; echo "You have correctly answered this question " ; } else { $score21 = -2; //CAT equivalent mark -0.33 echo "Your answer was wrong for this question" ; echo "
The correct answer is: $ans21
" ; } } //Ansewr 22// echo "

Q22. "; if($_POST["answer22"] == 'e22') { $score22=0; echo "You have not attempted this question. " ; echo "
The correct answer is: $ans22
" ; } else { $attempted=$attempted+1; if($time22 == $_POST["answer22"]) { $score22 = 8;//CAT equivalt 0.5 marks $correct=$correct+1; echo "You have correctly answered this question " ; } else { $score22= -2;//CAT equivalent score -0.166 echo "Your answer was wrong for this question" ; echo "
The correct answer is: $ans22
" ; } } //Ansewr 23 echo "

Q23. "; if($_POST["answer23"] == 'e23') { $score23=0; echo "You have not attempted this question. " ; echo "
The correct answer is: $ans23
" ; } else { $attempted=$attempted+1; if($time23 == $_POST["answer23"]) { $score23 = 8;//CAT equvalent mark 2 $correct=$correct+1; echo "You have correctly answered this question. " ; } else { $score23= -2;//CAT equivalent mark -0.66 echo "Your answer was wrong for this question. " ; echo "
The correct answer is: $ans23
" ; } } //Ansewr 24// echo "

Q24. "; if($_POST["answer24"] == 'e24') { $score24=0; echo "You have not attempted this question. " ; echo "
The correct answer is: $ans24
" ; } else { $attempted=$attempted+1; if($time24 == $_POST["answer24"]) { $score24 = 8; // CAT equivalent mark 1 $correct=$correct+1; echo "You have correctly answered this question " ; } else { $score24 = -2; //CAT equivalent mark -0.33 echo "Your answer was wrong for this question" ; echo "
The correct answer is: $ans24
" ; } } //Ansewr 25// echo "

Q25. "; if($_POST["answer25"] == 'e25') { $score25=0; echo "You have not attempted this question. " ; echo "
The correct answer is: $ans25
" ; } else { $attempted=$attempted+1; if($time25 == $_POST["answer25"]) { $score25 = 8;//CAT equivalt 0.5 marks $correct=$correct+1; echo "You have correctly answered this question " ; } else { $score25= -2;//CAT equivalent score -0.166 echo "Your answer was wrong for this question" ; echo "
The correct answer is: $ans25
" ; } } //Ansewr 26// echo "

Q26. "; if($_POST["answer26"] == 'e26') { $score26=0; echo "You have not attempted this question. " ; echo "
The correct answer is: $ans26
" ; } else { $attempted=$attempted+1; if($time26 == $_POST["answer26"]) { $score26 = 8;//CAT equvalent mark 2 $correct=$correct+1; echo "You have correctly answered this question. " ; } else { $score26= -2;//CAT equivalent mark -0.66 echo "Your answer was wrong for this question. " ; echo "
The correct answer is: $ans26
" ; } } //Ansewr 27// echo "

Q27. "; if($_POST["answer27"] == 'e27') { $score27=0; echo "You have not attempted this question. " ; echo "
The correct answer is: $ans27
" ; } else { $attempted=$attempted+1; if($time27 == $_POST["answer27"]) { $score27 = 8; // CAT equivalent mark 1 $correct=$correct+1; echo "You have correctly answered this question " ; } else { $score27 = -2; //CAT equivalent mark -0.33 echo "Your answer was wrong for this question" ; echo "
The correct answer is: $ans27
" ; } } //Ansewr 28// echo "

Q28. "; if($_POST["answer28"] == 'e28') { $score28=0; echo "You have not attempted this question. " ; echo "
The correct answer is: $ans28
" ; } else { $attempted=$attempted+1; if($time28 == $_POST["answer28"]) { $score28 = 8;//CAT equivalt 0.5 marks $correct=$correct+1; echo "You have correctly answered this question " ; } else { $score28= -2;//CAT equivalent score -0.166 echo "Your answer was wrong for this question" ; echo "
The correct answer is: $ans28
" ; } } //Ansewr 29// echo "

Q29. "; if($_POST["answer29"] == 'e29') { $score29=0; echo "You have not attempted this question. " ; echo "
The correct answer is: $ans29
" ; } else { $attempted=$attempted+1; if($time29 == $_POST["answer29"]) { $score29 = 8;//CAT equvalent mark 2 $correct=$correct+1; echo "You have correctly answered this question. " ; } else { $score29= -2;//CAT equivalent mark -0.66 echo "Your answer was wrong for this question. " ; echo "
The correct answer is: $ans29
" ; } } //Ansewr 30// echo "

Q30. "; if($_POST["answer30"] == 'e30') { $score30=0; echo "You have not attempted this question. " ; echo "
The correct answer is: $ans30
" ; } else { $attempted=$attempted+1; if($time30 == $_POST["answer30"]) { $score30 = 8; // CAT equivalent mark 1 $correct=$correct+1; echo "You have correctly answered this question " ; } else { $score30 = -2; //CAT equivalent mark -0.33 echo "Your answer was wrong for this question" ; echo "
The correct answer is: $ans30
" ; } } echo "

You have attempted $attempted number of questions out of which $correct were correct" ; /** ALL SCORES ARE CALCULATED **/ $scored1= $score1 + $score2 + $score3 + $score4 + $score5 +$score6 + $score7 + $score8 + $score9 + $score10 + $score11 + $score12 + $score13 + $score14 + $score15 + $score16 + $score17 + $score18 + $score19 + $score20 + $score21 + $score22 + $score23 + $score24 + $score25 +$score26 + $score27 + $score28 + $score29 + $score30; $scored=($scored1/4); if($scored<8) { echo "

Your Overall Score is $scored. This is a below average score. Please give a better try in next test

" ; } if(($scored>7)&&($scored<14)) { echo "

Your Overall Score is $scored. This is an average score.Try to improve it in next test.

" ; } if(($scored>13)&&($scored<20)) { echo "

Congrats!!! Your Overall Score is $scored. This is an above average score and with this performance in real CAT/XAT/MAT/IRMA/IIFT you can expect to cross the cutoff. Try to score more than 20 in next test.

" ; } if(($scored>19)) { echo "

Congrats!!! Your Overall Score is $scored. This is an excellent score and with this performance in real CAT/XAT/MAT/IRMA/IIFT you can expect to score more than 99 percentile . Try to maintain this score in all the upcoming tests.

" ; } echo "

Thanx For your Time
" ; ?>

CAT2005 SECTION - EU

Instructions
1. The test comprises of 50 questions. You should complete the test within 40 minutes.
2. There is only one correct answer to each question.
3. All questions carry four marks each.
4. Each wrong answer will attract a penalty of one mark.

SECTION II

Sub-Section II-A

Number of Questions : 10

Note: Questions 1 to 10 carry one mark each.

DIRECTIONS for Q.1 to Q4:
The passage given below is followed by a set of four questions. Choose the best answer to each question.

A game of strategy, as currently conceived in game theory, is a situation in which two or more "players" make choices among available alternatives (moves). The totality of choices determines the outcomes of the game, and it is assumed that the rank order of preferences for the outcomes is different for different players. Thus the "interests" of the players are generally in conflict. Whether these interests are diametrically opposed or only partially opposed depends on the type of game.

Psychologically, most interesting situations arise when the interests of the players are partly coincident and partly opposed, because then one can postulate not only a conflict among the players but also inner conflicts within the players. Each is torn between a tendency to cooperate, so as to promote the common interests, and a tendency to compete, so as to enhance his own individual interests.

Internal conflicts are always psychologically interesting. What we vaguely call "interesting" psychology is in very great measure the psychology of inner conflict. Inner conflict is also held to be an important component of serious literature as distinguished from less serious genres. The classical tragedy, as well as the serious novel, reveals the inner conflict of central figures. The superficial adventure story, on the other hand, depicts only external conflict; that is, the threats to the person with whom the reader (or viewer) identifies stem in these stories exclusively from external obstacles and from the adversaries who create them. On the most primitive level this sort of external conflict is psychologically empty. In the fisticuffs between the protagonists of good and evil, no psychological problems are involved or, at any rate, none are depicted in juvenile representations of conflict.

The detective story, the "adult" analogue of a juvenile adventure tale, has at times been described as a glorification of intellectualized conflict. However, a great deal of the interest in the plots of these stories is sustained by withholding the unraveling of a solution to a problem. The effort of solving the problem is in itself not a conflict if the adversary (the unknown criminal) remains passive, like Nature, whose secrets the scientist supposedly unravels by deduction. If the adversary actively puts obstacles in the detective's path toward the solution, there is genuine conflict. But the conflict is psychologically interesting only to the extent that it contains irrational components such as a tactical error on the criminal's part or the detective's insight into some psychological quirk of the criminal or something of this sort. Conflict conducted in a perfectly rational manner is psychologically no more interesting than a standard Western. For example, Tic-tac-toe, played perfectly by both players, is completely devoid of psychological interest. Chess may be psychologically interesting but only to the extent that it is played not quite rationally. Played completely rationally, chess would not be different from Tic-tac-toe.

In short, a pure conflict of interest (what is called a zero-sum game) although it offers a wealth of interesting conceptual problems, is not interesting psychologically, except to the extent that its conduct departs from rational norms.

1. According to the passage, internal conflicts are psychologically more interesting than external conflicts because

  • a. internal conflicts, rather than external conflicts, form an important component of serious literature as distinguished from less serious genres.
  • b. only juveniles or very few "adults" actually experience external conflict, while internal conflict is more widely prevalent in society.
  • c. in situations of internal conflict, individuals experience a dilemma in resolving their own preferences for different outcomes.
  • d. there are no threats to the reader (or viewer) in case of external conflicts.
  • e.Not Attempted

2.Which, according to the author, would qualify as interesting psychology?

  • a. A statistician's dilemma over choosing the best method to solve an optimisation problem.
  • b. A chess player's predicament over adopting a defensive strategy against an aggressive opponent.
  • c. A mountaineer's choice of the best path to Mt. Everest from the base camp.
  • d. A finance manager's quandary over the best way of raising money from the market.
  • e.Not Attempted

3. According to the passage, which of the following options about me application of game theory to a conflict-of-interest situation is true?

  • a. Assuming that the rank order of preferences for options is different for different players.
  • b. Accepting that the interests of different players are often in conflict.
  • c. Not assuming that the interests are in complete disagreement.
  • d. All of the above.
  • e.Not Attempted

4.The problem solving process of a scientist is different from that of a detective because

  • a. scientists study inanimate objects, while detectives deal with living criminals or law offenders.
  • b. scientists study known objects, while detectives have to deal with unknown criminals or law offenders.
  • c. scientists study phenomena that are not actively altered, while detectives deal with phenomena that have been deliberately influenced to mislead.
  • d. scientists study psychologically interesting phenomena, while detectives deal with "adult" analogues of juvenile adventure tales.
  • e.Not Attempted

DIRECTIONS for Q.5 to Q.7:
The sentences given in each question, when properly sequenced, form a coherent paragraph. Each sentence is labeled with a letter. Choose the most logical order of sentences from among the given choices to construct a coherent paragraph.

5. (A) Similarly, turning to caste, even though being lower caste is undoubtedly a separate cause of disparity, its impact is all the greater when the lower-caste families also happen to be poor.
(B) Belonging to a privileged class can help a woman to overcome many barriers that obstruct women from less thriving classes.
(C) It is the interactive presence of these two kinds of deprivation - being low class and being female - that massively impoverishes women from the less privileged classes.
(D) A congruence of class deprivation and gender discrimination can blight the lives of poorer women very severely.
(E) Gender is certainly a contributor to societal inequality, but it does not act independently of class.

  • a. EABDC
  • b. EBDCA
  • c. DAEBC
  • d. BECDA
  • e.Not Attempted

6. (A) When identity is thus 'defined by contrast', divergence with the West becomes central. (B) Indian religious literature such as the Bhagavad Gita or the Tantric texts, which are identified as differing from secular writings seen as 'western', elicits much greater interest in the West than do other Indian writings, including India's long history of heterodoxy. (C) There is a similar neglect of Indian writing on non-religious subjects, from mathematics, epistemology and natural science to economics and linguistics. (D) Through selective emphasis that point up differences with the West, other civilizations can, in this way, be redefined in alien terms, which can be exotic and charming, or else bizarre and terrifying, or simply strange and engaging. (E) The exception is the Kamasutra in which western readers have managed to cultivate an interest

  • a. BDACE
  • b. DEABC
  • c. BDECA
  • d. BCEDA
  • e.Not Attempted

7. (A) This is now orthodoxy to which I subscribe - up to a point. (B) It emerged from the mathematics of chance and statistics. (C) Therefore the risk is measurable and manageable. (D) The fundamental concept: Prices are not predictable, but the mathematical laws of chance can describe their fluctuations. (E) This is how what business schools now call modern finance was born.

  • a. ADCBE
  • b. EBDCA
  • c. ABDCE
  • d. DCBEA
  • e.Not Attempted

DIRECTIONS for Q.8 to Q.10:
In each question, the word at the top of the table is used in four different ways, numbered A to D.
Choose the option in which the usage of the word is incorrect or inappropriate.

8. NEAR

  • a. I got there just after you left - a near miss!
  • b. She and her near friend left early.
  • c. The war led to a near doubling of oil prices.
  • d. They came near to tears seeing the plight of the victims.
  • e.Not Attempted

9. HAND

  • a. I have my hand full, I cannot do it today.
  • b. The minister visited the jail to see the breach at first hand.
  • c. The situation is getting out of hand here!
  • d. When the roof of my house was blown away, he was willing to lend me a hand.
  • e.Not Attempted

10. FOR

  • a. He has a great eye for detail.
  • b. We are waiting for the day.
  • c. I can’t bear for her to be angry.
  • d. It couldn’t be done for ever.
  • e.Not Attempted

SECTION II

Sub-Section II-B

Number of Questions : 20

Note: Questions 11 to 30 carry two marks each.

Directions for Questions 11 to 18:
Each of the two passages given below is followed by a set of four questions. Choose the best answer to each question.

PASSAGE I

Crinoline and croquet are out. As yet, no political activists have thrown themselves in front of the royal horse on Derby Day. Even so, some historians can spot the parallels. It is a time of rapid technological change. It is a period when the dominance of the world's superpower is coming under threat. It is an epoch when prosperity masks underlying economic strain. And, crucially, it is a time when policy-makers are confident that all is for the best in the best of all possible worlds. Welcome to the Edwardian Summer of the second age of globalisation.

Spare a moment to take stock of what's been happening in the past few months. Let's start with the oil price, which has rocketed to more than $65 a barrel, more than double its level 18 months ago. The accepted wisdom is that we shouldn't worry our little heads about that, because the incentives are there for business to build new production and refining capacity, which will effortlessly bring demand and supply back into balance and bring crude prices back to S25 a barrel. As Tommy Cooper used to say, 'just like that'. Then there is the result of the French referendum on the European Constitution, seen as thick-headed luddites railing vainly against the modern world. What the French needed to realise, the argument went, was that there was no alternative to the reforms that would make the country more flexible, more competitive, more dynamic. Just the sort of reforms that allowed Gate Gourmet to sack hundreds of its staff at Heathrow after the sort of ultimatum that used to be handed out by Victorian mill owners. An alternative way of looking at the French non" is that our neighbours translate "flexibility" as "you're fired".

Finally, take a squinl at the United States. Just like Britain a century ago, a period of unquestioned superiority is drawing to a close. China is still a long way from matching America's wealth, but it is growing at a stupendous rate and economic strength brings geopolitical clout. Already, there is evidence of a new scramble for Africa as Washington and Beijing compete for oil stocks. Moreover, beneath the surface of the US economy, all is not well. Growth looks healthy enough, but the competition from China and elsewhere has meant the world's biggest economy now imports far more than it exports. The US is living beyond its means, but in this time of studied complacency a current account deficit worth 6 percent of gross domestic product is seen as a sign of strength, not weakness. In this new Edwardian summer, comfort is taken from the fact that dearer oil has not had the savage inflationary consequences of 1973-74, when a fourfold increase in the cost of crude brought an abrupt end to a postwar boom that had gone on uninterrupted for a quarter of a century. True, the cost of living has been affected by higher transport costs, but we are talking of inflation at 2.3 per cent and not 27 per cent. Yet the idea that higher oil prices are of little consequence is fanciful. If people are paying more to fill up their cars it leaves them with less to spend on everything else, but there is a reluctance to consume less. In the 1970s unions were strong and able to negotiate large, compensatory pay deals that served to intensify inflationary pressure. In 2005, that avenue is pretty much closed off, but the abolition of all the controls on credit that existed in the 1970s means that households are invited to borrow more rather than consume less. The knock-on effects of higher oil prices are thus felt in different ways - through high levels of indebtedness, in inflated asset prices, and in balance of payments deficits.

There are those who point out, rightly, that modern industrial capitalism has proved mightily resilient these past 250 years, and that a sign of the enduring strength of the system has been the way it apparently shrugged off everything - a stock market crash, 9/11, rising oil prices - that have been thrown at it in the half decade since the millennium. Even so, there are at least three reasons for concern. First, we have been here before. In terms of political economy, the first era of globalisation mirrored our own. There was a belief in unfettered capital flows, in free trade, and in the power of the market. It was a time of massive income inequality and unprecedented migration. Eventually, though, there was a backlash, manifested in a struggle between free traders and protectionists, and in rising labour militancy.

Second, the world is traditionally at its most fragile at times when the global balance of power is in flux. By the end of the nineteenth century, Britain's role as the hegemonic power was being challenged by the rise of the United States, Germany, and Japan while the Ottoman and Hapsburg empires were clearly in rapid decline, Looking ahead from 2005, it is clear that over the next two or three decades, both China and India - which together account for half the world's population - will flex their muscles. Finally, there is the question of what rising oil prices tell us. The emergence of China and India means global demand for crude is likely to remain high at a time when experts say production is about to top out. If supply constraints start to bite, any declines in the price are likely to be short-term cyclical affairs punctuating a long upward trend

11. By the expression 'Edwardian Summer', the author refers to a period in which there is

  • a. unparalleled luxury and opulence.
  • b. a sense of complacency among people because of all-round prosperity.
  • c. a culmination of all-round economic prosperity.
  • d. an imminent danger lurking behind economic prosperity.
  • e.Not Attempted

12. What, according to the author, has resulted in a widespread belief in the resilience of modern capitalism?

  • a. Growth in the economies of Western countries despite shocks in the form of increase in levels of indebtedness and inflated asset prices.
  • b. Increase in the prosperity of Western countries and China despite rising oil prices
  • c. Continued growth of Western economies despite a rise in terrorism, an increase in oil prices and other similar shocks.
  • d. The success of continued reforms aimed at making Western economies more dynamic, competitive and efficient.
  • e.Not Attempted

13. Which of the following best represents the key argument made by the author?

  • a. The rise in oil prices, the flux in the global balance of power and historical precedents should make us question our belief that the global economic prosperity would continue.
  • b. The belief that modern industrial capitalism is highly resilient and capable of overcoming shocks will be belied soon.
  • c. Widespread prosperity leads to neglect of early signs of underlying economic weakness, manifested in higher oil prices and a flux in the global balance of power.
  • d. A crisis is imminent in the West given the growth of countries like China and India and the increase in oil prices.
  • e.Not Attempted

14. What can be inferred about the author's view when he states, 'As Tommy Cooper used to say "just like that'"?

  • a. Industry has incentive to build new production and refining capacity and therefore oil prices would reduce.
  • b. There would be a correction in the price levels of oil once new production capacity is added.
  • c. The decline in oil prices is likely to be short-term in nature.
  • d. It is not necessary that oil prices would go down to earlier levels.
  • e.Not Attempted

PASSAGE II

While complex in the extreme, Derrida's work has proven to be a particularly influential approach to the analysis of the ways in which language structures our understanding of ourselves and the world we inhabit, an approach he termed deconstruction. In its simplest formulation, deconstruction can be taken to refer to a methodological strategy which seeks to uncover layers of hidden meaning in a text that have been denied or suppressed. The term 'text', in this respect, does not refer simply to a written form of communication, however. Rather, texts are something we all produce and reproduce constantly in our everyday social relations, be they spoken, written or embedded in the construction of material artifacts. At the heart of Derrida's deconstructive approach is his critique of what he perceives to be the totalitarian impulse of the Enlightenment pursuit to bring all that exists in the world under the domain of a representative language, a pursuit he refers to as logocentrism. Logocentrism is the search for a rational language that is able to know and represent the world and all its aspects perfectly and accurately. Its totalitarian dimension, for Derrida at least, lies primarily in its tendency to marginalize or dismiss all that does not neatly comply with its particular linguistic representations, a tendency that, throughout history, has all too frequently been manifested in the form of authoritarian institutions. Thus logocentrism has, in its search for the truth of absolute representation, subsumed difference and oppressed that which it designates as its alien 'other'. For Derrida, western civilization has been built upon such a systematic assault on alien cultures and ways of life, typically in the name of reason and progress.

In response to logocentrism, deconstruction posits the idea that the mechanism by which this process of marginalization and the ordering of truth occurs is through establishing systems of binary opposition. Oppositional linguistic dualisms, such as rational/ irrational, culture/nature and good/bad are not, however, construed as equal partners as they are in, say, the semiological structuralism of Saussure. Rather, they exist, for Derrida, in a series of hierarchical relationships with the first term normally occupying a superior position. Derrida defines the relationship between such oppositional terms using the neologism differance. This refers to the realization that in any statement, oppositional terms differ from each other (for instance, the difference between rationality and irrationality is constructed through oppositional usage), and at the same time, a hierarchical relationship is maintained by the deference of one term to the other (in the positing of rationality over irrationality, for instance). It is this latter point which is perhaps the key to understanding Derrida's approach to deconstruction,

For the fact that at any given time one term must defer to its oppositional 'other', means that the two terms are constantly in a state of interdependence. The presence of one is dependent upon the absence or 'absent-presence' of the 'other', such as in the case of good and evil, whereby to understand the nature of one, we must constantly relate it to the absent term in order to grasp its meaning. That is, to do good, we must understand that our act is not evil for without that comparison the term becomes meaningless. Put simply, deconstruction represents an attempt to demonstrate the absent-presence of this oppositional 'other', to show that what we say or write is in itself not expressive simply of what is present, but also of what is absent. Thus, deconstruction seeks to reveal the interdependence of apparently dichotomous terms and their meanings relative to their textual context; that is, within the linguistic power relations which structure dichotomous terms hierarchically. In Derrida's own words, a deconstructive reading "must always aim at a certain relationship, unperceived by the writer, between what he commands and what he does not command of the patterns of a language that he uses. . . .[It] attempts to make the not-seen accessible to sight."

Meaning, then, is never fixed or stable, whatever the intention of the author of a text. For Derrida, language is a system of relations that are dynamic, in that all meanings we ascribe to the world are dependent not only on what we believe to be present but also on what is absent. Thus, any act of interpretation must refer not only to what the author of a text intends, but also to what is absent from his or her intention. This insight leads, once again, to Derrida's further rejection of the idea of the definitive authority of the intentional agent or subject. The subject is decentred; it is conceived as the outcome of relations of differance. As author of its own biography, the subject thus becomes the ideological fiction of modernity and its logocentric philosophy, one that depends upon the formation of hierarchical dualisms, which repress and deny the presence of the absent 'other'. No meaning can, therefore, ever be definitive, but is merely an outcome of a particular interpretation.

15. According to the passage, Derrida believes that

  • a. Reality can be construed only through the use of rational analysis.
  • b. Language limits our construction of reality
  • c. A universal language will facilitate a common understanding of reality.
  • d. We need to uncover the hidden meaning in a system of relations expressed by language.
  • e. Not Attempted

16. To Derrida, 'logocentrism' does not imply

  • a. A totalitarian impulse.
  • b. A domain of representative language.
  • c. Interdependence of the meanings of dichotomous terms.
  • d. A strategy that seeks to suppress hidden meanings in a text.
  • e.Not Attempted

17. According to the passage, Derrida believes that the system of binary opposition

  • a. represents a prioritization or hierarchy.
  • b. reconciles contradictions and dualities.
  • c. weakens the process of marginalization and ordering of truth.
  • d. deconstructs reality.
  • e.Not Attempted

18. Derrida rejects the idea of 'definitive authority of the subject' because

  • a. interpretation of the text may not make the unseen visible
  • b. the meaning of the text is based on binary opposites.
  • c. the implicit power relationship is often ignored.
  • d. any act of interpretation must refer to what the author intends.
  • e.Not Attempted

DIRECTIONS for Q.19 to Q.22:
Each of the following questions has a paragraph from which the last sentence has been deleted. From the given options, choose the one that completes the paragraph in the most appropriate way.

19. The audiences for crosswords and sudoku, understandably, overlap greatly, but there are differences, too. A crossword attracts a more literary person, while sudoku appeals to a keenly logical mind. Some crossword enthusiasts turn up their noses at sudoku because they feel it lacks depth. A good crossword requires vocabulary, knowledge, mental flexibility and sometimes even a sense of humor to complete. It touches numerous areas of life and provides an "Aha!" or two along the way

  • a. Sudoku, on the other hand, is just a logical exercise, each one similar to the last
  • b. Sudoku, incidentally, is growing faster in popularity than crosswords, even among the literati.
  • c. Sudoku, on the other hand, can be attempted and enjoyed even by children.
  • d. Sudoku, however, is not exciting in any sense of the term.
  • e.Not Attempted

20. Most firms consider expert individuals to be too elitist, temperamental, egocentric, and difficult to work with. Force such people to collaborate on a high-stakes project and they just might come to fisticuffs. Even the very notion of managing such a group seems unimaginable. So most organizations fall into default mode, setting up project teams of people who get along nicely.

  • a. The result, however, is disastrous.
  • b. The result is mediocrity.
  • c. The result is creation of experts who then become elitists.
  • d. Naturally, they drive innovations.
  • e.Not Attempted

21. Federer's fifth grand slam win prompted a reporter to ask whether he was the best ever. Federer is certainly not lacking in confidence, but he wasn't about to proclaim himself the best ever. "The best player of this generation, yes", he said, "But nowhere close to ever. Just look at the records that some guys have. I'm a minnow,"

  • a. His win against Agassi, a genius from the previous generation, contradicts that.
  • b. Sampras, the king of an earlier generation, was as humble.
  • c. He is more than a minnow to his contemporaries.
  • d. The difference between 'the best of this generation' and 'the best ever' is a matter of perception.
  • e.Not Attempted

22. Thus the end of knowledge and the closing of the frontier that it symbolizes is not a looming crisis at all, but merely one of many embarrassing fits of hubris in civilization's long industry. In the end, it will pass away and be forgotten. Ours is not the first generation to struggle to understand the organizational laws of the frontier, deceive itself that it has succeeded, and go to its grave having failed.

  • a. One would be wise to be humble.
  • b. But we might be the first generation to actually reach the frontier.
  • c. But we might be the first generation to deal with the crisis,
  • d. However, this time the success is not illusory.
  • e.Not Attempted

DIRECTIONS for Q.23 to Q.26:
Each question consists of four sentences on a topic. Some sentences are grammatically incorrect or inappropriate. Select the option that indicates the grammatically correct and appropriate sentence(s).

23. A. When virtuoso teams begin their work, individuals are in and group consensus is out. B. As project progresses, however, the individual stars harness themselves to the product of the group. C. Sooner or later, the members break through their own egocentrism and become a plurality with single-minded focus on the goal. D. In short, they morph into a powerful team with a shared identity.

  • a. A & C
  • b. A & D
  • c. B & D
  • d. A, C & D
  • e.Not Attempted

24. A. Large reductions in the ozone layer, which sits about 15-30 km above the Earth, take place each-winter over the polar regions, especially the Antarctic, as low temperatures allow the formation of stratospheric clouds that assist chemical reactions breaking down ozone. B. Industrial chemicals containing chlorine and bromine have been blamed for thinning the layer because they attack the ozone molecules, making them to break apart. C. Many an offending chemicals have now been banned. D. It will still take several decades before these substances have disappeared from the atmosphere.

  • a. D
  • b. B & D
  • c. A & D
  • d. A & C
  • e.Not Attempted
25. A. The balance of power will shift to the East as China and India evolve. B. Rarely the economic ascent of two still relatively poor nations has been watched with such a mixture of awe, opportunism, and trepidation. C. Postwar era witnessed economic miracles in Japan and South Korea, but neither was populous enough to power worldwide growth or change the game in a complete spectrum of industries. D. China and India, by contrast, possess the weight and dynamism to transform the 21st-century global economy.
  • a. A, B & C
  • b. A & D
  • c. C
  • d. C & D
  • e.Not Attempted

26. A. People have good reason to care about the welfare of animals. B. Ever since Enlightenment, their treatment has been seen as a measure of mankind's humanity. C. It is no coincidence that William Wilberforce and Sir Thomas Foxwell Buxton, two leaders of the movement to abolish the slave trade, helped found the Royal Society for the Prevention of Cruelty to Animals in 1820s. D. An increasing number of people go further: mankind has a duty not to cause pain to animals that have the capacity to suffer.

  • a. A & D
  • b. B
  • c. A & C
  • d. C & D
  • e.Not Attempted

DIRECTIONS for Q.27 to Q.30: Each of the following questions has a paragraph with one italicized word that docs not make sense. Choose the most appropriate replacement for that word from the options given below the paragraph.

27. Intelligent design derives from an early 19th-century explanation of the natural world given by an English clergyman, William Paley. Paley was the populariser of the famous watchmaker analogy. Proponents of intelligent design are crupping Paley's argument with a new gloss from molecular biology.

  • a. destroying
  • b. testing
  • c. resurrecting
  • d. questioning
  • e.Not Attempted

28. Women squat, heads covered, beside huge piles of limp fodder and blunk oil lamps, and just about all the cows in the three towns converge upon this spot. Sinners, supplicants and yes, even scallywags hand over a few coins for a crack at redemption and a handful of grass.

  • a. shining
  • b. bright
  • c. sputtering
  • d. effulgent
  • e.Not Attempted

29. It is klang to a sensitive traveler who walks through this great town, when he sees the streets, the roads, and cabin doors crowded with beggars, mostly women, followed by three, four, or six children, all in rags and importuning every passenger for alms.

  • a. amusing
  • b. irritating
  • c. disgusting
  • d. distressing
  • e.Not Attempted

30. Or there is the most fingummy diplomatic note on record: when Philip of Macedon wrote to the Spartans that, if he came within their borders, he would leave not one stone of their city, they wrote back the one word - "If"

  • a. witty
  • b. rude
  • c. simple
  • d. terse
  • e.Not Attempted